Property MC Flashcards

ALA Quizzes, Kaplan Quizzes, Kaplan Qbank, Kaplan BAR Prep, MOST LIKELY/MAYBE CLS Quizzes

1
Q

A miller runs a mill that processes only hardwood. Over the years, pine has remained plentiful in the area, but hardwood has become more difficult to find. The miller is informed that a developer plans to build a condominium development in a stand of hardwoods near the miller’s mill. The miller speaks with the developer and learns that he would like to have most of the stand cleared out because it is located in the center of his construction area. During their conversation, the developer agrees that the miller can clear the entire 20-acre stand and remove the trees to his mill. The miller has an unreliable truck that he has used for years, but he buys a new one to haul the hardwood from the developer’s property.

After the developer obtains all necessary permits, but before he breaks ground on the condominium project, he sells the property to an investor, who knows of the agreement between the miller and the developer and who intends to build a commercial building on the site. When the miller arrives to begin harvesting the trees, the investor prohibits him from entering the land, telling him that she intends to sell the hardwood to a dealer.

If the miller files suit seeking an order compelling the investor to allow him to harvest the hardwood, what is the most likely outcome?

The investor will prevail, because the miller and the developer did not have a written agreement that the miller could enter the land.

The investor will prevail, because the license is freely revocable.

The miller will prevail, because the investor cannot prevent him from entering the land under an estoppel theory.

The miller will prevail, because the easement is enforceable against the investor.

A

The correct answer is: The miller will prevail, because the investor cannot prevent him from entering the land under an estoppel theory.

Discussion of correct answer: A license is a right to enter the land of another. A license coupled with an interest is the right to enter land on which personalty in which the licensee has an interest is located. Here, the miller holds a license coupled with an interest because the developer (the licensor) has granted the miller (the licensee) the right to enter his property to remove personalty (the hardwood). A license is generally revocable at the will of the licensor. However, a license may become irrevocable in two situations: (1) a license coupled with an interest, which exists when one person owns personal property on the land of another and has a privilege incidental to such personal property to come on the land to use or recover the personalty; and (2) an executed license, which is based on estoppel and often involves substantial expenditure of funds in reliance on the promisor’s promise to allow the promisee to use the land. An executed license is sometimes deemed to be the equivalent of an easement. Because the miller expended substantial funds on a new truck to haul the wood from the investor’s property, the investor is estopped from revoking the miller’s license to enter the land.

How well did you know this?
1
Not at all
2
3
4
5
Perfectly
2
Q

A businessman divided his time between his three estates. On Tuesday, the businessman signed a valid purchase-and-sale agreement selling one of his estates to his partner. The deal was to close the following Monday. On Thursday, the businessman died while enjoying his mistress’s company at one of the estates not for sale. His will left his real property to his mistress for life, and the remainder of his estate was left to his wife. Wishing only to show his respect for the dead, the partner notified the businessman’s executor that he was willing to postpone the closing for several weeks, as long as he could move in immediately. The executor replied that the partner would now have to renegotiate the sale of the property with the mistress and with the remaindermen. The partner became livid at this information and filed suit against the businessman’s estate for specific performance.

Will the partner prevail, and why or why not?

Yes, because equitable title passed to the partner upon the execution of the purchase-and-sale agreement.

Yes, because the contract did not terminate with the businessman’s death.

No, because the property was now part of the mistress’ life estate.

No, because, as the businessman can no longer sign the deed, the contract has become impossible to perform.

A

The correct answer is: Yes, because equitable title passed to the partner upon the execution of the purchase-and-sale agreement.

Discussion of correct answer: Under the doctrine of equitable conversion, for the period of time between the signing of a valid purchase-and-sale agreement and the closing on the property, the seller retains legal title to the subject property, and the buyer acquires equitable title to the subject property. Therefore, when the purchase-and-sale agreement was executed, the partner acquired an equitable and enforceable interest in the estate.

How well did you know this?
1
Not at all
2
3
4
5
Perfectly
3
Q

A literary agent and a talk show host entered into a month-to-month lease agreement regarding a house owned by the agent. The bathrooms had not been renovated in 15 years, and the sinks, toilets and showers were rust stained and moldy. Without the agent’s permission, the talk show host undertook a renovation of the master bathroom. He installed new sinks, shower stalls and hi-tech self cleaning toilets with seat warmers. The lease agreement contained no provision regarding repairs and renovations. The jurisdiction follows the common law rule.

If the literary agent sues the talk show host, who will prevail?

A The agent, because the talk show host is not a life tenant.
B The agent, because the renovations constitute permissive waste.
C The talk show host, because the renovations increase the market value of the property.
D The talk show host, because the renovations constitute ameliorative waste.

A

A The agent, because the talk show host is not a life tenant.

Discussion of correct answer: Ameliorative waste is an act that increases the market value of the premises by permanently altering them. The renovations in this example constitute ameliorative waste. While a life tenant may commit ameliorative waste, tenants under regular lease agreements, such as the month-to-month lease in this example, may not commit ameliorative waste. Therefore, under the facts presented, the talk show host violated his duties by renovating the apartment and the agent will prevail.

How well did you know this?
1
Not at all
2
3
4
5
Perfectly
4
Q

A teacher moved to another state for a job and bought a house by taking out a mortgage with a bank. Among the various provisions in the mortgage note were: the loan amount, and the interest rate, and an acceleration clause providing that, in the event of default, the mortgagee could declare the entire balance due and that the mortgagor would forfeit his right of redemption in the event that a foreclosure sale was consummated.

Two years later, the teacher stopped making his payments. He defaulted on the loan and the mortgagee began proceedings for a judicial foreclosure. During this period, the teacher rented the house to a businessman but did not use any of the rent to pay the mortgage. The mortgage company held a foreclosure sale. Two months after the foreclosure sale, the teacher won a large amount of money from a scratch off lottery ticket and went to redeem his home but was prevented by the terms of his contract from redeeming the home after the foreclosure sale. This jurisdiction recognizes both equitable and statutory rights of redemption.

If the teacher brings an action to recover his home, what is the likely outcome?

A The teacher will not prevail, because he failed to redeem the home with the rent he received from the businessman.
B The teacher will not prevail, because a foreclosure sale terminates the mortgagor’s interest in the mortgaged real estate.
C The teacher will prevail, because the right of redemption may not be waived in the original purchase document.
D The teacher will prevail, because acceleration clauses are generally considered unconscionable.

A

C The teacher will prevail, because the right of redemption may not be waived in the original purchase document.

How well did you know this?
1
Not at all
2
3
4
5
Perfectly
5
Q

A man inherited a large estate with a log cabin that had been constructed many generations ago using wood from the surrounding old-growth forest. For over a century, the family had been exploiting the timber on the property by permitting companies to enter and remove trees at a sustainable rate, although never from a five-acre area around the log cabin. Not wanting to give up his life in the city, the man leased the log cabin and the five acres around it to a tenant. In the third year of the lease, a serious rainstorm caused part of the roof and walls of the cabin to collapse. The tenant, wanting to retain the locally sourced nature of the cabin, cut down several trees nearby to use in making repairs. While the repairs kept the log cabin from becoming unusable, the fact that old-growth trees were used for the repairs caused a net loss in the value of the property due to how long it would take for them to be replaced. When the man learned of this, he sued the tenant for the loss in value to the property and the value of the trees that had been cut down.

Will the tenant be required to reimburse the man?

A No, because the tenant used the trees to repair the property.

B No, because of the open mines doctrine.

C Yes, because the tenant committed voluntary waste.

D Yes, because the tenant committed ameliorative waste.

A

The correct answer is:No, because the tenant used the trees to repair the property.

Discussion of correct answer:A tenant is not allowed to do anything that adversely affects the future interest that follows the present possession; doing so is called waste. In the case of a lease, this means that the tenant cannot adversely affect the landlord’s interest in the property. When a tenant voluntarily commits an act that has more than a trivial injurious effect on or change in the property, the tenant has committed voluntary waste. However, despite the prohibition on voluntary waste, natural resources may be consumed in three situations: (1) for the repair and maintenance of the property; (2) with permission of the grantor; or (3) under the open mines doctrine. Because the tenant cut down the trees in order to repair and maintain the property, he will not be required to reimburse the man for their value.

Discussion of incorrect answers:

Incorrect. No, because of the open mines doctrine. Under the open mines doctrine, if the grantor or landlord was exploiting the natural resources of the property, it is presumed that the grantee or tenant has the right to continue that exploitation. In this situation, the removal of natural resources by the tenant will not be considered voluntary waste. However, this doctrine is not applicable on these facts. While the man’s family had been selling timber off the property for over a century, it had never allowed the resources on the land surrounding the log cabin to be exploited, and that property is what was leased to the tenant. Instead, the tenant will not be liable for the decreased value of the property because he removed the trees in order to repair and maintain the property, which is a permissible use and not considered voluntary waste.

Incorrect. Yes, because the tenant committed voluntary waste. When a tenant voluntarily commits an act that has more than a trivial injurious effect on or change in the property, the tenant has committed voluntary waste. However, despite the prohibition on voluntary waste, natural resources may be consumed in three situations: (1) for the repair and maintenance of the property; (2) with permission of the grantor; or (3) under the open mines doctrine. Because the tenant cut down the trees in order to repair and maintain the property, he will not be required to reimburse the man for their value.

Incorrect. Yes, because the tenant committed ameliorative waste. Ameliorative waste occurs when a tenant commits an act that increased the value of the premises by permanently altering it. Traditionally, ameliorative waste was prohibited, but under modern law, a tenant is allowed to commit ameliorative waste under certain situations. However, the tenant here did not commit ameliorative waste; the tenant committed voluntary waste, if anything. Further, the tenant will be allowed to use the resources in this way, and will not be liable, because he cut down the trees to repair and maintain the property.

How well did you know this?
1
Not at all
2
3
4
5
Perfectly
6
Q

A city installed several hundred new public restrooms on its sidewalks. Each restroom was a self-contained unit; insertion of a quarter opened the locked door. The units were selfsanitizing; when activated, powerful jets of hot water and disinfectant cleansed the interior. The cleaning cycle could only be activated by a switch contained in a locked recess in the rear of the unit; only specially designated municipal employees carried the key to the recess. These employees patrolled the various units, inspecting them for damage and, after insuring that no one was inside, unlocking the cleaning switch and activating the cleansing cycle. A man brought an action against the city for personal injuries. At trial, the above facts concerning the public restrooms were established. The man introduced additional proof that he had deposited a quarter and entered one of the units early one weekday morning when no one was in sight on the city sidewalk, that while he was inside the cleansing cycle was activated, causing him severe injury, that after his screams attracted a passing police officer who pulled him from the unit, the recess containing the activation switch was locked, and that subsequent tests established that the unit was functioning properly.

Should the man recover for his injuries suffered in the public restroom?

A. No, because the man failed to introduce any evidence of negligence on the part of the city.

B. No, because the man was a licensee and there was no evidence that the public restroom unit had malfunctioned.

C. Yes, because the city is strictly liable for operating what amounted to an ultrahazardous activity.

D. Yes, because an employee of the city must have negligently activated the cleansing cycle of the unit without checking to see if anyone was inside.

A

D. Yes, because an employee of the city must have negligently activated the cleansing cycle of the unit without checking to see if anyone was inside.

How well did you know this?
1
Not at all
2
3
4
5
Perfectly
7
Q

A chemist owned a house that he leased to a tenant pursuant to a residential month-to-month lease agreement. Five months after executing the lease agreement with the tenant, the chemist assigned his interest in the lease to his mother. The written lease agreement did not address assignments.

What is the status of the lease?

(A) The chemist is the landlord under the terms of the lease, because the assignment is invalid.

(B) The chemist’s mother is the landlord under the terms of the lease, because a landlord under either a residential or a commercial periodic lease may assign his lease interest.

(C) The chemist’s mother is the landlord under the terms of the lease, because the lease was a residential lease.

(D) Neither the chemist nor his mother is the landlord under the terms of the lease, because the lease terminated upon the chemist’s attempted assignment.

A

(B) The chemist’s mother is the landlord under the terms of the lease, because a landlord under either a residential or a commercial periodic lease may assign his lease interest.

How well did you know this?
1
Not at all
2
3
4
5
Perfectly
8
Q

On July 1, a college senior entered into an oral agreement with a landlord to lease an apartment near her college campus for $750 per month for a period of one year, starting immediately. At the end of the year, having found a job nearby following her graduation, the college senior began looking for a new apartment. Because she did not find one right away, however, she remained in the apartment through the month of July. She did not, however, pay any rent to the landlord.

What is the relationship between the parties as of August 1?

A A month-to-month periodic tenancy has been formed.

B Because the original lease agreement was not in writing, the Statute of Frauds will operate to invalidate the agreement, resulting in an at-will tenancy.

C The college senior is a tenant at sufferance, and the landlord may immediately evict her.

D The landlord cannot evict the college senior without providing at least 30 days’ notice, but may collect rent from the senior according to the lease terms.

A

C The college senior is a tenant at sufferance, and the landlord may immediately evict her.

Discussion of correct answer: A tenancy at sufferance occurs when a tenant remains in possession of the leased premises (“holds over”) after the end of the lease term. A tenancy at sufferance is not a true tenancy. In most states, if a residential tenant holds over, the landlord may recover possession of the premises and receive the reasonable rental value for the holdover period. Here, the college senior remained in possession of the apartment after the end of the lease term. If the college senior had paid her rent for the month of July on time and the landlord had accepted it, a month-to-month periodic tenancy would have been formed, requiring the landlord to provide notice before terminating the tenancy. However, because she did not do so, she remains on the property as a tenant at sufferance, and the landlord may immediately evict her and collect the reasonable rental value of the apartment for the month of July.

Discussion of incorrect answers:

Incorrect. A month-to-month periodic tenancy has been formed. A lease for a term of years has a definite beginning and end date (e.g., the lease expires on a set date or remains in effect only for a set number of months or years). No notice is required to terminate a lease for a term of years at the end of the specified duration. A periodic tenancy may be created expressly or by implication, as in the case where a holdover tenant pays rent after the end of the lease agreement and the landlord accepts the payment. Here, the one-year oral lease agreement, running from July 1 through June 30, was a lease for term of years, which ended on June 30. The college senior then became a holdover tenant on July 1. Because there was no rent payment made and accepted for the month of July, the college senior will remain a tenant at sufferance, permitting the landlord to immediately evict her and collect the reasonable rental value of the apartment for the month of July.

Incorrect. Because the original lease agreement was not in writing, the Statute of Frauds will operate to invalidate the agreement, resulting in an at-will tenancy. A lease may be created expressly, either orally or in writing. A writing is generally required by the Statute of Frauds for leases with a term of more than one year. An implied lease may be also created, as when a holdover tenant pays rent and the landlord accepts the payment. Here, the term was for one year. However, the lease had already been fully performed, because the one-year period had elapsed. As of the following July 1, the college senior became a holdover tenant. Thus, the Statute of Frauds is inapplicable.

Incorrect. The landlord cannot evict the college senior without providing at least 30 days’ notice, but may collect rent from the senior according to the lease terms. Had the college senior paid rent for the month of July after holding over and the landlord accepted the payment, a month-to-month periodic tenancy would have been created. Once a periodic tenancy existed, the landlord would have been required to provide notice to the college senior before terminating the lease. The notice would need to have been given at least one rental payment period prior to the termination date. However, no periodic tenancy was created here; instead, the college senior remains as a tenant at sufferance. As such, the landlord is entitled to immediately evict the college senior. Additionally, when the tenant is a holdover tenant, the landlord is entitled to collect the reasonable rental value of the property for the holdover period, and is not limited to the rent payment specified in the previous lease.

How well did you know this?
1
Not at all
2
3
4
5
Perfectly
9
Q

A patron checked into a hotel. The hotel’s outer structure had been built as a replica of the classic Roman style of architecture with Ionic columns and marble frescoes draped over the entrance. The elaborate entryway led inside to a variety of shops and restaurants that adjoined the hotel lobby. The hotel management knew that pigeons often nested inside the overhead sculptures. This resulted in pigeon droppings frequently falling outside the main entryway. Consequently, the hotel management instructed its employees to clean the area around the front entrance on a daily basis. One afternoon, one of the patron’s friends came by the hotel to pay a visit. As the friend walked under the columns to enter the hotel, he slipped on some pigeon droppings that had not been wiped up. On this particular day, the hotel employees had failed to clean the entrance area. Injuring his leg, the friend now brings suit against the hotel.

In his personal injury action, will the friend recover?

(A) Yes, because the hotel employees knew of the dangerous condition and failed to remedy it in a timely manner.

(B) Yes, because the hotel employees had not cleaned the entryway.

(C) No, because the friend was not a paying guest of the hotel.

(D) No, because the hotel is only responsible for natural conditions on the land.

A

(A) Yes, because the hotel employees knew of the dangerous condition and failed to remedy it in a timely manner.

How well did you know this?
1
Not at all
2
3
4
5
Perfectly
10
Q

A landlord and a doctor entered into a written lease agreement for the lease of an apartment owned by the landlord. Under the terms of the lease agreement, the doctor was to rent the apartment from the landlord for a period of one year. On the day the lease commenced, the doctor found he could not gain access to the apartment, because the landlord had changed the locks.

Has the landlord breached his duty to deliver possession of the premises to the doctor?

(A) Yes, under the American rule, because the landlord has not delivered actual possession of the premises to the doctor.

(B) Yes, under the minority rule, because the landlord has constructively evicted the doctor.

(C) No, under the minority (American) rule, because the doctor has a legal right to possess the premises.

(D) No, under the majority (English) rule, because a lease for a term of years may be terminated at will.

A

(C) No, under the minority (American) rule, because the doctor has a legal right to possess the premises.

How well did you know this?
1
Not at all
2
3
4
5
Perfectly
11
Q

A great-uncle conveyed his farm to his grandniece and grandnephew as joint tenants. Neither the grandniece nor the grandnephew worked on the land, but they hired a local farmer to plant crops. Four years later, the grandniece granted a mortgage interest in her joint tenancy property to a bank to secure a debt. The term of the mortgage was 10 years. The grandniece paid on the debt for seven years, then died, leaving a will devising her real property to her son. Assume that the jurisdiction follows the lien theory.

If the grandnephew brings an action to quiet title to the land, how will the court find?

The grandnephew owns the land.

The grandnephew and the grandniece’s son hold the land as tenants in common because the mortgage severed the joint tenancy.

The grandnephew and the grandniece’s son hold the land as tenants in common, subject to the bank’s mortgage.

The grandnephew owns the land, subject to the mortgage.

A

The correct answer is: The grandnephew owns the land.

Discussion of correct answer: The grandniece’s joint tenancy interest in the land passed automatically by right of survivorship to the grandnephew when she died. The grandniece’s mortgaging of her joint tenancy interest in the land did not sever the joint tenancy because the land is in a lien theory jurisdiction. Under the lien theory, a mortgage constitutes a lien on the property (rather than a severance of title as in a title theory jurisdiction), and the joint tenancy is not severed.

How well did you know this?
1
Not at all
2
3
4
5
Perfectly
12
Q

On February 1, the owner of a parcel delivered a deed to a woodcutter granting him a profit in gross in the land. As holder of the profit, the woodcutter was entitled to enter the land for the purpose of cutting and taking any timber growing on the property. On April 1, the owner entered into a contract to sell the land to a buyer. On April 15, in exchange for $50,000, the owner delivered a warranty deed for the land to the buyer. On April 30, the woodcutter properly recorded his interest in the land. The buyer did not discover the woodcutter’s interest in the land until May 2, when the buyer happened to notice the woodcutter leaving the land driving a truck loaded with cut timber. That same day, the buyer recorded his deed to the land.

Under which type of recording act will the buyer prevail?

A notice recording act.

A race recording act.

A race-notice recording act.

All of the above, because the woodcutter’s profit in gross cannot be protected by a recording act.

A

The correct answer is:A notice recording act.

Discussion of correct answer:Under the common law, priority is generally awarded to the grantee who was first to receive an interest from the grantor (this is known as “first-in-time, first-in-right”). A recording act may alter the outcome, depending on whether it is a race, notice, or race-notice type of act. Under a notice-type recording act, a subsequent bona fide purchaser who takes for value and without notice of the prior conveyance will prevail over a grantee who failed to record his interest prior to the date of the conveyance to the subsequent purchaser. The buyer purchased the land without notice of the woodcutter’s interest, because the woodcutter did not record the profit until after the conveyance. Thus, the buyer will prevail under a notice recording act, despite the fact that he recorded after the woodcutter did so.

How well did you know this?
1
Not at all
2
3
4
5
Perfectly
13
Q

A man bought a house from a woman. The house was encumbered by a mortgage that was recorded. The man bought the house subject to the mortgage and took over the mortgage payments. The man subsequently sold the house to an investor who was aware of the mortgage. The investor bought the house by an installment land-sale contract that made no mention of the mortgage. After receiving just two of the 20 agreed upon payments under the installment land-sale contract, the man stopped making payments on the mortgage. The mortgagee-initiated foreclosure proceedings.

What will happen to the house and the debt?

(D) The mortgagee may foreclose on the property and hold the woman personally liable for any debt remaining after the foreclosure sale.

(C) The mortgagee may foreclose on the property and hold the man personally liable for any debt remaining after the foreclosure sale.

(B) The mortgagee may foreclose on the property and hold the investor personally liable for any debt remaining after the foreclosure sale.

(A) The mortgagee is estopped from foreclosing on the property, because the last purchaser is a bona fide purchaser.

A

(D) The mortgagee may foreclose on the property and hold the woman personally liable for any debt remaining after the foreclosure sale.

How well did you know this?
1
Not at all
2
3
4
5
Perfectly
14
Q

In 2005, an owner conveyed his real property to his son and daughter as joint tenants with rights of survivorship. Two years later, the son executed a mortgage on the property to a bank to secure a loan. A couple of months after this, the son died intestate, leaving one child as his only heir. At the time of the son’s death, the mortgage had not been paid to the bank. The jurisdiction in which the property is located recognizes a title theory of mortgages.

In an appropriate action, the court should determine that title to the property is vested in whom?

The daughter, with the entire interest subject to the mortgage.

The daughter, free and clear of the mortgage.

Half in the daughter and half in the child, with both subject to the mortgage.

Half in the daughter, free of the mortgage, and half in the child, subject to the mortgage.

A

The correct answer is: Half in the daughter, free of the mortgage, and half in the child, subject to the mortgage.

Discussion of correct answer: In a title theory state, the execution of a mortgage by one joint tenant causes the legal interest of that co-tenant to be transferred to the mortgagee. As a result, the mortgage severs the joint tenancy, because the unities (of title and interest) have been destroyed. This severance results in the mortgaging co-tenant having a tenancy in common with the remaining co-tenant(s). (As a note, if there is more than one remaining co-tenant, their interest remains a joint tenancy with respect to each other, and a tenancy in common with respect to the mortgaging co-tenant.) Furthermore, because a tenancy in common has no unity of interest, the daughter’s undivided one-half interest in the property was not encumbered by the son’s mortgage. The daughter will take free of the mortgage, whereas the child will take subject to the mortgage.

How well did you know this?
1
Not at all
2
3
4
5
Perfectly
15
Q

A young man executed a written deed conveying his vineyard to his handyman, which the handyman properly recorded. The young man had a successful ice skating career, but lost a breach of contract case when he refused to perform a show. The backers of the show, who had no actual notice of the earlier conveyance of the property and did not make a search of the title records, promptly recorded a judgment lien against the vineyard. The state recording act reads, “A conveyance of an estate in land shall not be valid as against any judgment creditor, except such persons having notice of it, unless the conveyance is properly recorded.”

Will the court enforce the judgment lien against the vineyard?

(A) No, because the backers are not purchasers for value.

(B) No, because the handyman properly recorded the deed to the property.

(C) Yes, because as a donee, the handyman was not a purchaser for value, mortgagee, or judgment creditor protected by the state’s recording act.

(D) Yes, because the prompt recording of the judgment lien revoked the earlier deed of the property to the handyman.

A

(B) No, because the handyman properly recorded the deed to the property.

How well did you know this?
1
Not at all
2
3
4
5
Perfectly
16
Q

Two brothers–a chef and a golf pro–lived on a 50-acre parcel of land that their late father had bequeathed to them as tenants in common. They occupied a two-bedroom cabin located near the eastern boundary of the parcel. To the west was a larger parcel owned by a writer. Two years after their father’s death, the chef and the golf pro had a disagreement; the golf pro moved out of the cabin and into an apartment. A year after the golf pro left, the writer telephoned the chef and said that he wanted to construct a chainlink fence on their boundary line. Unfortunately, the writer said, the terrain was so rugged that a survey would add several thousand dollars to the cost of the fence. The writer therefore proposed that they obtain a map, agree on a reasonable location for the fence, and let that serve as the boundary between the two parcels. The chef agreed, and the writer subsequently built a chain-link fence along the line they selected.

Three years later, the chef died intestate, leaving the golf pro as his only heir. The golf pro returned to the property, had a survey performed, and learned that the chain-link fence significantly encroached upon his property. He brought an action to seek redress for this encroachment.

Who should prevail in this action?

(A) The golf pro, because the agreement between his brother and the writer was not in writing.

(B) The golf pro, because he had no notice of the agreement between his brother and the writer.

(C) The writer, because the chef was entitled to establish the boundary between their two parcels by agreement.

(D) The writer, because the golf pro was “ousted” from the tenancy in common when he moved to his apartment.

A

(C) The writer, because the chef was entitled to establish the boundary between their two parcels by agreement.

How well did you know this?
1
Not at all
2
3
4
5
Perfectly
17
Q

A purchaser wished to construct an industrial complex. On August 15, a seller and the purchaser entered into a written agreement that provided, among other things, as follows: “The seller hereby agrees to sell the seller’s unimproved property consisting of 40 acres to the purchaser for the price of $400,000. The seller shall convey good and marketable title. Closing of escrow is to occur no later than December 1. This agreement will be void if the seller cannot close and convey good and marketable title on December 1.”

On October 20, the seller discovered a 20-year-old easement running through the center of the property that a former owner had granted to the city when a mass transit rail line had been in the planning stages. The rail system had been abandoned when voters refused to support an increase in a local tax to finance construction. The seller immediately notified the purchaser of the easement and that she had begun negotiations with the city manager and the city planning commission to repurchase the easement and thereby extinguish it. On November 20, the purchaser telephoned the seller and asked about progress on the easement repurchase. The seller stated that a few holdovers on the planning commission were arguing to retain the easement, but that she had the votes to swing the commission her way, given enough time. “Do your best,” was the purchaser’s only response. Due to the Thanksgiving holiday, the planning commission did not vote on the easement repurchase issue until November 30, but voted to sell the easement back to the seller. The city manager’s office could not process the paperwork necessary to effectuate the sale of easement rights to the seller until December 2, on which day the seller deposited into escrow a warranty deed conveying good and marketable title to the subject property to the purchaser. The purchaser refused to pay the purchase price and declared that the contract had been rescinded. The seller sought specific performance in an appropriate legal action.

Assuming the seller prevails, which of the following is the best support for the court’s decision?

Time was not of the essence in the agreement between the seller and the purchaser.

Delayed performance can never be considered a total breach of a land sale contract.

The purchaser led the seller to believe that strict adherence to the time of the essence clause would not be insisted upon.

The purchaser would be unjustly enriched if permitted to rely upon one day’s delay in performance as justification for rescinding the contract.

A

The correct answer is: The purchaser led the seller to believe that strict adherence to the time of the essence clause would not be insisted upon.

Discussion of correct answer: Ordinarily, failure to perform a contractual obligation by the time specified in the contract is a non- material breach of the agreement. It permits the aggrieved party to recover damages caused by the breach, but is not so serious to be considered a material or total breach that the aggrieved party is excused from performing, and that party may not properly regard the contract as terminated. So long as the “late” party tenders performance within a reasonable time, the breach is cured, and if the aggrieved party refuses to perform, the late party may obtain specific performance. This situation is reversed if the contract is “time is of the essence.” Where a contract states that “time is of the essence,” late performance is a total breach, and the aggrieved party is excused from performance. The late party may not compel specific performance by tardily tendering his performance. Merely establishing a date for performance is not sufficient to make time of the essence; there must be some additional language such as “time is of the essence” or “this agreement is null and void if performance is not tendered upon the date specified,” indicating that timely performance will be critical. Even where time is of the essence, the courts are often hostile to a strict enforcement of such clauses, frequently finding waiver of the importance of timely performance in written or oral statements or from the circumstances. In this problem, the purchaser’s response to the seller’s November 20 progress report regarding negotiations with the city planning commission (“Do your best”) could be construed as a waiver of the right to demand performance by December 1. Thus, this is the best argument in support of specific performance for the seller.

How well did you know this?
1
Not at all
2
3
4
5
Perfectly
18
Q

A young man spent some time chatting and drinking beer in the front yard with his friends after their softball league game. By the time the friends leave and he enters his house he is drunk, and he accidentally leaves his baseball bat on the sidewalk. The next morning, a woman in a convertible drove by the young man’s house. The woman was checking her makeup in her rear-view mirror, and before she realized it, she had veered off the road and onto the sidewalk. The convertible’s tire struck the baseball bat that the young man had left on the sidewalk, sending it flying into the air. The bat hit the woman in the head, severely injuring her. She filed suit against the young man for negligence.

Will she prevail?

A. No, because the young man was drunk and it was foreseeable that he would leave his bat on the sidewalk.

B. No, because people do not usually drive on the sidewalk.

C. Yes, because the young man should not have left his bat on the sidewalk.

D. Yes, because but for the young man’s act, the driver would not have been injured.

A

B. No, because people do not usually drive on the sidewalk.

How well did you know this?
1
Not at all
2
3
4
5
Perfectly
19
Q

When Grantor retires from his cold Northern state to someplace warmer, he wants his daughter and son to be able to keep the old homestead. He conveys the homestead to them both by stating, “to Daughter and Son as joint tenants, and not as tenants in common, with right of survivorship.” Both Daughter and Son revise their wills to include bequests to their children. Daughter bequeaths her property to Grantor’s two grandsons as tenants in common. Son bequeaths his interest to Grantor’s granddaughter, who is married.

If Daughter predeceases Son, what interest does the granddaughter stand to inherit?

A tenancy in common with the two grandsons.

A joint tenancy with the two grandsons.

A tenancy by the entirety with her spouse.

Complete ownership of the property.

A

The correct answer is: Complete ownership of the property.

Discussion of correct answer: In this example, granddaughter winds up with the entire property. When Daughter predeceases Son, the entire property belongs to Son because Daughter and Son had a joint tenancy with right of survivorship. Although the granddaughter is married, a bequest to her does not automatically include her spouse in a tenancy by the entirety. She therefore would take complete ownership.

How well did you know this?
1
Not at all
2
3
4
5
Perfectly
20
Q

A landlord entered into a lease agreement with a tenant pursuant to which the landlord was required to pump out and empty the house’s small septic tank every six months, on January 1 and July 1. During the second year of the lease, the landlord failed to pump the tank in July despite repeated complaints by the tenant, and as a result, the property was suffused with a foul order for the entire month. The tenant remained in the apartment. When the landlord finally arrived to pump the tank at the beginning of August and demanded the previous month’s rent, the tenant refused to pay. The landlord then sued the tenant for the unpaid rent.

Under the majority rule, what remedy is the tenant entitled to pursue?

A The tenant is entitled to a rent abatement, because the landlord has breached the covenant of quiet enjoyment, but he is not relieved of all liability for rent.

B The tenant may withhold the entire rent, because he has been constructively evicted.

C The tenant may not withhold the entire rent, because he has merely been constructively evicted.

D The tenant may not withhold the entire rent, because he remained in the apartment.

A

D The tenant may not withhold the entire rent, because he remained in the apartment.

Discussion of correct answer:Under the majority rule, a tenant may treat a lease as terminated if the landlord breaches the covenant of quiet enjoyment (express or implied in every lease) by constructive eviction of the tenant. In order for a constructive eviction to exist: (1) the landlord must substantially and permanently interfere with the tenant’s use and enjoyment of the premises; and (2) the tenant must move out. In this case, because the tenant did not move out, he has not been constructively evicted, and therefore may not withhold the rent.

Discussion of incorrect answers:

Incorrect. The tenant is entitled to a rent abatement, because the landlord has breached the covenant of quiet enjoyment, but he is not relieved of all liability for rent. This answer not only makes an erroneous assumption but also applies an incorrect rule of law. If a landlord breaches the covenant of quiet enjoyment, such as by an actual or constructive eviction, a tenant may treat the lease as terminated and withhold rent. Thus, in this case, if the landlord had in fact breached the covenant of quiet enjoyment by constructively evicting the tenant, the tenant would have been entitled to withhold the entire rent, not merely a rent abatement. Moreover, given that the tenant did not move out or abandon even a part of the premises, he was not constructively evicted. Therefore, the landlord will be deemed not to have breached the covenant of quiet enjoyment.

Incorrect. The tenant may withhold the entire rent, because he has been constructively evicted. In order for a tenant to be constructively evicted: (1) the landlord must substantially and permanently interfere with the tenant’s use and enjoyment of the premises; and (2) the tenant must move out. In this case, while the first of these requirements was met, the second was not, because the tenant did not move out. Therefore, the tenant has not been constructively evicted and cannot withhold the entire rent.

Incorrect. The tenant may not withhold the entire rent, because he has merely been constructively evicted. This answer makes an assumption not supported by the facts as well as applies an incorrect rule of law. In order for a constructive eviction to exist: (1) the landlord’s act must substantially and permanently interfere with the tenant’s use and enjoyment of the premises; and (2) the tenant must move out. Here, because the tenant did not move out or abandon even a part of the premises, the tenant was not constructively evicted. Moreover, if the tenant had been constructively evicted, he would have been entitled to withhold the entire rent, or receive an abatement if only partially constructively evicted.

How well did you know this?
1
Not at all
2
3
4
5
Perfectly
21
Q

On December 1, 2050, a businessman executed a written 10-year agreement leasing a business property he owned to a restaurateur. The jurisdiction has a Statute of Frauds requiring that a lease for more than two years be written to be enforceable. On March 1, 2057, the restaurateur assigned the lease to a shoe salesman who was opening a shoe store. Neither the restaurateur nor the shoe salesman gave the businessman any notice of the assignment.

Is the assignment valid?

Yes, if the assignment was in writing and the lease did not specifically prohibit an assignment.

Yes, whether or not the assignment was in writing.

No, because no one gave the businessman notice of the assignment.

No, because the lease contained no clause specifically allowing an assignment.

A

The correct answer is: Yes, if the assignment was in writing and the lease did not specifically prohibit an assignment.

Discussion of correct answer: A lease may be freely assigned unless its terms expressly prohibit it. However, to satisfy the jurisdiction’s Statute of Frauds, the assignment must be in writing, because more than two years remained on the lease.

How well did you know this?
1
Not at all
2
3
4
5
Perfectly
22
Q

A father devised his property “to my daughter for life with remainder to such of her children as should be living at her death, provided, however, that no such child of my daughter shall sell, mortgage, or otherwise transfer his or her interest in the property prior to attaining age 20; and if any such child of my daughter shall violate this provision, then upon such violation, his or her interest shall pass to the remaining children of my daughter then living.” The will devised the residuary of the father’s estate to his wife.

The father died and was survived by his wife, daughter, and the daughter’s three children. The wife brought an action for a declaration of rights, claiming that the attempted gifts to the father’s grandchildren were entirely void and that the interest following the life estate to his daughter passed to the wife absolutely by the residuary clause. Assume that the action was properly brought with all necessary parties and with a guardian ad litem appointed to represent the interests of unborn and infant grandchildren.

How should the court decide?

The attempted gifts to grandchildren are void under the Rule Against Perpetuities.

The attempted gifts to grandchildren are void as unlawful restraints against alienation.

The provisions concerning the grandchildren are valid and will be upheld according to their terms.

Even if the provisions against sale, mortgage, or other transfer by the grandchildren are void, the remainders to grandchildren are otherwise valid and will be given effect.

A

The correct answer is: Even if the provisions against sale, mortgage, or other transfer by the grandchildren are void, the remainders to grandchildren are otherwise valid and will be given effect.

Discussion of correct answer: A restraint against alienation of an inheritable future interest is void where the restraint may continue in effect after the future interest becomes possessory. Similarly, a restraint against alienation applied to an inheritable future interest is void to the same extent that the restraint would be void if the future interest were presently possessory (i.e., if the future interest will result in a fee simple, the restraint is void from its inception). In this question, if the daughter dies with a surviving child under age 20, then the restraint on alienation will continue in effect after that child’s remainder interest becomes possessory, and thus the restraint is void. The effect of voiding the restraint is that the children take free of the restraint. The validity of the gifts to them, therefore, is not affected.

How well did you know this?
1
Not at all
2
3
4
5
Perfectly
23
Q

A man and woman each owned adjoining parcels of land. The man constructed a house on his lot and resided there with his wife and three children. The woman resided in another state. Twenty-five years ago, the man built a tennis court in the backyard of his lot. Unbeknownst to the man, the tennis court extended three feet onto the woman’s lot.

The woman had no knowledge of the encroachment until this year, when she returned for the first time since she purchased it. When she inspected her lot, she ascertained that the man’s tennis court extended beyond the property line. The statutory period for both adverse possession and prescriptive easements is 20 years.

If the woman now brings an action against the man alleging encroachment, for whom should the court rule?

The woman, because the man was unaware that his possession was adverse.

The woman, because the man failed to exercise any acts of dominion that would exclude the woman from the three- foot strip.

The man, because his possession was hostile.

The man, because the woman’s conduct manifests an intent to abandon any claim she might have had to an exclusive right in the three-foot strip.

A

The correct answer is: The man, because his possession was hostile.

Discussion of correct answer: The man’s possession was (1) actual and exclusive; (2) open, visible, and notorious (i.e., not secret or clandestine, but rather, occupying as a true owner); (3) continuous (without abandonment or a physical hiatus in occupancy); (4) hostile and adverse; and (5) for the statutory period. The key element in this question is hostile and adverse. Under the majority view, it is generally stated that the adverse possessor must intend to hold adversely against the whole world, including the rightful owner. The motives of the adverse possessor are irrelevant. Whether he intends a wrongful disseisin or whether he occupies what he believes to be his own, he acquires title, because it is the visible and adverse possession with intent to possess that constitutes its adverse character, not the remote views or mistaken belief of the possessor [Schertz v. Rundles, 363 N.E. 2d 203 (1977)]. It is clear that the man intended to possess the strip under the tennis court, believing that it was his own property. Therefore, because the man’s possession was both adverse and hostile, the woman will not prevail in an action for encroachment. The man has obtained title to the three-foot strip by adverse possession.

How well did you know this?
1
Not at all
2
3
4
5
Perfectly
24
Q

A seller learned that her friend was looking for a home in the area. The friend loved the seller’s home and offered to purchase it. However, the friend had very little money, and offered to purchase the home at a price which was substantially lower than the price the seller hoped to get for the property. Despite the low offer, the seller was eager to sell her house, and she agreed to sell the home to her friend for a reduced price subject to the mortgage that the seller already had on the property. The seller immediately stopped making mortgage payments on the home. Unfortunately, the friend did not make any payments either. The outstanding balance on the home was $200,000, but the market value of the home was only $150,000.

Which of the following is correct?

(A) The mortgagee cannot foreclose upon the property, but the friend is personally liable for the debt.

(B) The mortgagee cannot foreclose upon the property, and the friend is not personally liable for the debt.

(C) The mortgagee may foreclose upon the property, but the friend is not personally liable for the debt.

(D) The mortgagee may foreclose upon the property, and the friend is personally liable for the debt.

A

(C) The mortgagee may foreclose upon the property, but the friend is not personally liable for the debt.

How well did you know this?
1
Not at all
2
3
4
5
Perfectly
25
Q

Tom devised Blackacre to Larry and Dahlia in equal shares. A year later, Larry conveyed his interest in Blackacre to Paul. Larry subsequently died, and Paul brought an action to partition Blackacre. The jurisdiction has no statute directly applicable to the relevant issues.

What is the most likely reason why Dahlia would be declared the sole owner of all of Blackacre?

(A) The estate created in Larry and Dahlia by Tom’s will was a joint tenancy.

(B) The estate created in Larry and Dahlia by Tom’s will was a tenancy by the entirety.

(C) The estate created in Larry and Dahlia by Tom’s will included the right of survivorship.

(D) The estate created in Larry and Dahlia by Tom’s will was a fee rather than a life estate.

A

(B) The estate created in Larry and Dahlia by Tom’s will was a tenancy by the entirety.

How well did you know this?
1
Not at all
2
3
4
5
Perfectly
26
Q

Greenwood was a 500-acre parcel of land which bordered on Rural Highway, a two-lane country road. In 1950, the owner of Greenwood decided to subdivide Greenwood into 50 10-acre lots. He then sold each lot, and placed a restriction in each deed limiting development to residential use. Each deed was properly recorded.

Starting in 2000, a number of changes took place; the property surrounding Greenwood changed from entirely residential to almost entirely commercial, many of the homes in the Greenwood development were abandoned and fell into disrepair, some of the lots in Greenwood had been converted to commercial use, and Rural Highway was widened to three lanes in each direction to accommodate the increasing commercial traffic.

In 2010, the current owner of Lot 28 in the Greenwood subdivision sold the property to a businessman. The businessman desired to tear down the residential home on Lot 28 and build a factory.

What would be the businessman’s strongest argument as to why the restriction limited development to residential use should not be enforced?

Because over 60 years have passed since the restriction was first placed in the deeds, its enforcement would be barred by laches.

There has been a change to the neighborhood which should prevent enforcement of the restriction.

The continued enforcement of the restriction would amount to an unreasonable restraint on alienation.

Any suit to enforce the restriction would be barred by the statute of limitation.

A

The correct answer is: There has been a change to the neighborhood which should prevent enforcement of the restriction.

Discussion of correct answer: Under the “change of neighborhood” doctrine, when the neighborhood in which the burdened land is located has so changed that it would be inequitable to continue to enforce the restriction it will not be enforced. Here, it is clear that such changes have taken place so that the restriction should no longer be enforced.

How well did you know this?
1
Not at all
2
3
4
5
Perfectly
27
Q

A man was the record title owner of Casa de Sol, a beautiful 12-acre hacienda estate, in fee simple absolute. In 2005, the man conveyed Casa de Sol to his friend for life with remainder to the man’s daughter. The friend took possession of the property and, two years later, sold it to a buyer for $200,000. The following year, the daughter died and was survived by her son as sole heir. The buyer has now indicated a desire to sell the property. A dispute has arisen as to legal title to the property. The man, the friend, and the son are still alive.
Which of the following best describes the state of title to Casa de Sol?
(A) The son has a fee simple.

(B) The buyer has a life estate, and the man has a reversion.

(C) The buyer has a life estate for the life of the friend with a remainder in the son.

(D) The buyer has a life estate with a vested remainder in the son, subject to the man’s reversionary interest.

A

(C) The buyer has a life estate for the life of the friend with a remainder in the son.

How well did you know this?
1
Not at all
2
3
4
5
Perfectly
28
Q

A laboratory manufactured nitroglycerin (a chief ingredient in dynamite) at its plant. An artist who specialized in making glass sculptures had a studio two blocks away from the laboratory’s plant. One evening, an explosion at the laboratory’s plant caused the artist’s studio to shake, resulting in the destruction of valuable artwork. The artist brought a tort action against the laboratory to recover damages.

Which of the following, if established, would furnish the laboratory with its best possible defense?

(A) The laboratory used extraordinary care in the manufacture and storage of nitroglycerin and was not guilty of any negligence that was causally connected with the explosion.
(B) The laboratory has a contract with the federal government whereby all the nitroglycerin manufactured at its plant is used in U.S. military weapons systems.
(C) The explosion was caused when lightning (an act of God) struck the plant during an electrical storm.
(D) The harm that the artist suffered would not have resulted but for the abnormal fragility of the artist’s work.

A

(D) The harm that the artist suffered would not have resulted but for the abnormal fragility of the artist’s work.

How well did you know this?
1
Not at all
2
3
4
5
Perfectly
29
Q

A daughter needed to obtain residential care for her father. The daughter’s aunt planned to convey her property to a charity. However, the aunt told the daughter she would give her a life estate in the property so the daughter could use the income for her father’s care. The deed granted the property “to [the daughter], for the life of [the father], then to [the charity].”

What interest, if any, does the daughter have in the property?

(A) A life estate.
(B) A life estate pur autre vie.
(C) A fee simple determinable.
(D) A fee simple subject to a condition subsequent.

A

(B) A life estate pur autre vie.

How well did you know this?
1
Not at all
2
3
4
5
Perfectly
30
Q

A property developer and a real estate broker entered into a contract of sale, with the developer agreeing to sell his cabin to the broker for $100,000. At closing, the developer gave the broker a general warranty deed, and the broker handed the developer a cashier’s check for the full purchase price. The broker promptly recorded the deed, then sold the property to his dentist, also by general warranty deed. Many years later, the dentist was mending the fence that encircles the land when he found an eviction notice from the bank, which was commencing foreclosure proceedings against the developer’s father for defaulting on a mortgage to the property. The dentist then spoke to the bank and discovered that the developer had indeed sold the property to his own father one week before completing the transaction with the real estate broker.
Can the dentist recover from the developer?

(A) Yes, because the dentist is a bona fide purchaser for value.

(B) Yes, because the covenant for quiet enjoyment runs with the land.

(C) No, because only the real estate broker could recover from the developer.

(D) No, because the covenant against encumbrances is a present covenant.

A

(B) Yes, because the covenant for quiet enjoyment runs with the land.

How well did you know this?
1
Not at all
2
3
4
5
Perfectly
31
Q

A homeowner gave a mortgage on a property to a third-party lender for $4 million, $2.5 million to buy the property and $1.5 million to make improvements on it.

Which of the following is true?

(A) This mortgage is not a purchase-money mortgage, because a portion was used to make improvements on the land.

(B) This mortgage is not a purchase-money mortgage, because the mortgage was given as part of the same transaction in which the homeowner acquired title to the property.

(C) This mortgage is a purchase-money mortgage, and as such, it does not have priority over any pre-existing liens on the property.

(D) This mortgage is a purchase-money mortgage, and as such, it has priority over all prior liens that attach to the property through the actions of the homeowner.

A

(D) This mortgage is a purchase-money mortgage, and as such, it has priority over all prior liens that attach to the property through the actions of the homeowner.

How well did you know this?
1
Not at all
2
3
4
5
Perfectly
32
Q

As a tribute to his great body of legal work, a professor of property law established a library on his 10-acre estate. After several years of operating the library at a loss, the professor decided to sell the property. The professor conveyed the property by a duly executed deed “to librarian, but if the premises cease to be used for educational purposes, then to my alma mater.” One night, the librarian served his friend a beer on the premises. The professor’s alma mater filed suit claiming ownership. Assume that all interests satisfy the Rule Against Perpetuities.

Should the alma mater prevail?

No, because the professor holds a fee simple subject to condition subsequent.

No, because the librarian holds a fee simple subject to an executory interest.

Yes, because the alma mater holds a fee simple subject to a shifting executory interest.

Yes, because the estate will be automatically divested in favor of the alma mater.

A

The correct answer is: No, because the librarian holds a fee simple subject to an executory interest.

Discussion of correct answer: A fee simple subject to an executory interest is an estate that is automatically divested to a third person upon the occurrence of a named event. In this case, the professor deeded the property to the librarian, with the condition that the alma mater receives the property if the premises cease to be used for educational purposes. But in this case, the fact that the librarian served his friend a beer does not indicate that the premises have ceased to be used for educational purposes. The librarian will prevail because he holds the estate in fee simple subject to an executory interest and the named event that would cause transfer of the property has not occurred.

How well did you know this?
1
Not at all
2
3
4
5
Perfectly
33
Q

A homeowner sold property to his niece. The niece intended to record her deed, but planned to wait until the next week. She told her cousin about her plans, though, and showed him the deed. The cousin immediately went to the homeowner and convinced him to sell the property to him. The homeowner executed a deed, and the cousin took the deed and recorded it. The niece attempted to record her deed the next week as she had planned. The jurisdiction has a race recording statute.

Does the cousin have a superior claim to the property?

(A) No, because the cousin had actual notice of the conveyance to the niece.

(B) No, because the niece had the first deed.

(C) Yes, because he recorded his deed first.

(D) Yes, because the cousin was a purchaser for value.

A

(A) No, because the cousin had actual notice of the conveyance to the niece.

How well did you know this?
1
Not at all
2
3
4
5
Perfectly
34
Q

A restaurant owner ran a small restaurant in a suburban community, which he opened for dinner at 3:00 p.m., seven days a week, even though business was often slow at that hour. One rainy afternoon, a young couple stopped in for an early dinner just as the restaurant was opening. After finishing his meal, the man left the restaurant and went to pick up the couple’s car, so that his wife would not have to walk to the parking lot in the rain. Just as the man stepped off the curb onto the street, an elderly driver came around a blind corner and hit the man with his vehicle. The man was knocked to the ground and severely injured. Putting on his emergency lights, the driver got out of his car as quickly as possible and raced into the restaurant. Hearing the driver’s story, the woman begged the restaurant owner to permit her to use the telephone to call an ambulance, but the restaurant owner refused, having a strict “no customer phone use” policy. The woman ran to find another open business and an ambulance arrived 15 minutes later. According to the man’s physician, the delay in treatment allowed a blood clot to form in the man’s brain, which seriously complicated his condition.

If the man sues the restaurant owner, what is the restaurant owner’s best defense?

A. The restaurant owner’s acts were a passive contributor to the man’s injuries while the driver’s acts were primary and active.

B. The man was at fault for walking into the street.

C. The man was no longer a business invitee.

D. The man impliedly assumed the risk by walking into the street.

A

C. The man was no longer a business invitee.

How well did you know this?
1
Not at all
2
3
4
5
Perfectly
35
Q

A man owns a farm which he sells to an investor for $200,000. The farm was located in a state with a race-notice statute. The investor was not as concerned with recording and did not promptly record his deed. The man then transfers title of the farm to his brother as a gift. The brother promptly recorded the deed. Some time later, the investor recorded his deed to the farm. The investor then sued the brother to quiet title.

How should the court resolve the dispute over the farm?

A. The court should find in favor of the brother, because he recorded first.

B. The court should find in favor of the brother, because he took without notice of the deed to the investor.

C. The court should find in favor of the investor, because the brother did not pay value for the property.

D. The court should find in favor of the investor, because he had no notice of the deed to the brother.

A

C. The court should find in favor of the investor, because the brother did not pay value for the property.

How well did you know this?
1
Not at all
2
3
4
5
Perfectly
36
Q

A homeowner currently owes $150,000 on her mortgage. Due to recently losing her job, she has stopped making her monthly mortgage payments. The bank that holds the mortgage has taken the appropriate steps to conduct a judicial foreclosure on the house.

Which of the following is a requirement for a judicial foreclosure sale?

(B) The sale must result in a sale price that is at least the fair market value of the property.

(D) The sale must be conducted pursuant to a power-of-sale clause included in the mortgage documents.

(C) The sale must result in a fair sale price that is not necessarily the fair market value of the property.

(A) The sale must first be a private sale, and then after 30 days it must be made public.

A

(C) The sale must result in a fair sale price that is not necessarily the fair market value of the property.

How well did you know this?
1
Not at all
2
3
4
5
Perfectly
37
Q

A grandfather had a beautifully landscaped backyard including prize-winning apple trees. A competitor was upset that the grandfather kept winning the apple pie contests, so she sent several youths to steal apples from the grandfather’s trees during peak ripeness. The grandfather was upset when the apples began disappearing. To prevent further theft, he tied lengths of fishing line between trees, stretching it tightly about five feet off the ground. It was very difficult to see in the daytime and impossible to see at night. A few nights later, the youths entered the grandfather’s backyard. The grandfather heard them in his backyard, so he turned on the light and yelled, “Get out of here!” The youths began to run in all directions. One of them ran into one of the lengths of fishing line. His throat was cut by the line, and he was severely injured.

If the youth sues the grandfather for damages, should he recover?

A. Yes, because the grandfather used a mechanical device to protect his property.

B. Yes, because, without warning known trespassers, the grandfather used more force than was reasonably necessary to protect his property.

C. No, because the youth was injured when he was trespassing.

D. No, because the grandfather may use force to protect his property.

A

B. Yes, because, without warning known trespassers, the grandfather used more force than was reasonably necessary to protect his property.

How well did you know this?
1
Not at all
2
3
4
5
Perfectly
38
Q

A lumberjack and a truck driver entered into a residential lease. The agreement includes a provision that “the lease term shall continue until the lumberjack, at his sole discretion, terminates the lease.” The jurisdiction follows the majority rule.

Which most accurately describes the type of tenancy created?

A An at-will tenancy that may be terminated by the lumberjack.
B An at-will tenancy that may be terminated by either the lumberjack or by the truck driver.
C A life estate held by the truck driver.
D A periodic tenancy between the lumberjack and the truck driver.

A

B An at-will tenancy that may be terminated by either the lumberjack or by the truck driver.

Discussion of correct answer: A tenancy that has no fixed duration and lasts only as long as the landlord and tenant desire is an at-will tenancy. In this case, the lease agreement had no fixed duration and provided that only the landlord would have the power to terminate the lease. However, most courts will interpret this language as creating an at-will tenancy that can be terminated by either party. Therefore, this answer choice is correct.

How well did you know this?
1
Not at all
2
3
4
5
Perfectly
39
Q

A farmer held record title to a 200-acre orchard in fee simple absolute. On February 15, 2020, the farmer deeded the orchard to his daughter. The daughter did not record the deed. The farmer and his daughter agreed that the farmer could continue to live in the small house on the property, although the daughter was now the owner of the orchard. Based on this understanding, the farmer continued to reside on the orchard grounds. On March 1, 2021, the daughter deeded the orchard to a neighbor by quitclaim deed, in exchange for a payment of $15,000. The neighbor immediately and properly recorded the deed. However, the farmer tended to be forgetful about business matters. On April 1, 2021, the farmer, failing to remember that he had already deeded the property to his daughter, conveyed the orchard to a businessman in exchange for a sum that was slightly less than the fair market value of the property. The businessman immediately and properly recorded his deed. The neighbor and the businessman had no actual notice of their conflicting claims to the land. The jurisdiction had a standard tract index of documents affecting the title to land. The recording statute of the jurisdiction provided that, unless recorded, a deed would have no effect as to a subsequent purchaser for value who takes in good faith and who first records his or her deed.

In a suit over title to the property, which party will prevail?

(A) The neighbor, because he paid value for the property.

(B) The neighbor, because he recorded first.

(C) The businessman, because the neighbor took only a quitclaim deed, which contains no guarantees as to title.

(D) The businessman, because he is a bona fide purchaser for value and had no constructive notice of the prior conveyance.

A

(B) The neighbor, because he recorded first.

How well did you know this?
1
Not at all
2
3
4
5
Perfectly
40
Q

A landlord lived with his two tenants as roommates in a townhouse. The lease agreements specified that each tenant had private access to one of the bedrooms and access to the shared common areas of the house. One day, due to poor upkeep, one of the pipes burst under the kitchen sink, flooding the kitchen and rendering the entire room unusable. The tenants salvaged what they could of their property from the kitchen and complained to the landlord to fix it, but despite constant promises that he would get to it, the kitchen was never repaired.

Are the tenants entitled to withhold their next month’s rent payments?

(A) No, because they are entitled only to a rent abatement.

(B) No, because they did not move out of the townhouse.

(C) Yes, because they have been constructively evicted from part of the premises.

(D) Yes, because the landlord failed to maintain the kitchen in good repair.

A

(A) No, because they are entitled only to a rent abatement.

Explanation
The correct answer is: No, because they are entitled only to a rent abatement.

Discussion of correct answer: Where the landlord fails to maintain the common areas used by all tenants such that the tenant breaches the covenant of quiet enjoyment, this will be considered a constructive eviction. When the tenants are only constructively evicted from part of the premises, there has been a partial constructive eviction. Under the majority rule, if a tenant is constructively evicted from part of the premises, the tenant may receive a rent abatement, but is not relieved of all liability for rent. Therefore, the tenants are not entitled to withhold their entire rent payment, but may receive an abatement.

Discussion of incorrect answers:

Incorrect. No, because they did not move out of the townhouse. For constructive eviction to exist: (1) the landlord’s act must substantially and permanently interfere with the tenant’s use and enjoyment of the premises; and (2) the tenant must move out. When there constructive eviction is only partial, though, the tenant need only abandon the part of the premises that he has been constructively evicted from; he does not have to move out of the property entirely.

Incorrect. Yes, because they have been constructively evicted from part of the premises. Under the majority rule, when a tenant is constrictively evicted from only a part of the property, he is entitled only to a rent abatement, but not relieved of all liability to pay rent.

Incorrect. Yes, because the landlord failed to maintain the kitchen in good repair. While this is relevant in a claim that the landlord breached the covenant of quiet enjoyment, and thus partially constructively evicted the tenants from the kitchen common area, it is not a reason for the tenants to be relieved from all liability for rent. They would be entitled only to a rent abatement, and possibly damages for breach of the covenant of quiet enjoyment.

How well did you know this?
1
Not at all
2
3
4
5
Perfectly
41
Q

An owner has a home valued at $100,000 and owes Bank No. 1 $50,000 for a first mortgage lien. The owner owes Bank No. 2 $40,000 for a second mortgage. The owner applies to Bank No. 1 for additional funds to purchase a motor home, and the bank provides the funds. Bank No. 1’s mortgage is increased to $100,000 after it increases the homeowner’s mortgage.

Should Bank No. 2 feel undersecured as the result of Bank No. 1’s increasing the amount added to the homeowner’s mortgage?

Yes, because Bank No. 2 should have been required to authorize the additional loan.

Yes, because the homeowner increased his loan obligation by $50,000.

No, because the homeowner increased the principal amount owed to Bank No. 1.

No, because the homeowner will have larger installment payments as the result of the additional monies loaned.

A

The correct answer is: No, because the homeowner increased the principal amount owed to Bank No. 1.

Discussion of correct answer: The senior mortgage was modified when the homeowner increased the amount owed. Where there is a modification, a junior mortgage holder prevails over the modification if the modification materially prejudices the holder of the junior mortgage. Increasing the amount of the principal is an example of a modification materially prejudicing the holder of a junior mortgage. Since the homeowner increased the amount owed, the modification materially affected Bank No. 2. However, since Bank No. 2 would prevail over the modification, Bank No. 2 should not feel undersecured.

How well did you know this?
1
Not at all
2
3
4
5
Perfectly
42
Q

A husband and wife were walking down a country road when they were frightened by a steer running loose on the road. To avoid the steer, they climbed over a fence to get onto the adjacent property, which was owned by a farmer. After climbing over the fence, the husband and wife damaged some of the farmer’s crops that were growing near the fence, on which was posted a large “No Trespassing” sign. The farmer was walking with his large watchdog when he saw the husband and wife. The farmer approached them, hoping to frighten them with the fierce-looking dog, but the leash broke and the dog bit the wife.

If the wife files a battery claim against the farmer, will she prevail?

(A) No, because the wife made an unauthorized entry onto the farmer’s land.
(B) No, because the farmer did not intend to cause any harmful contact with the wife.
(C) Yes, because the farmer intended for the dog to frighten the wife.
(D) Yes, because the breaking of the leash establishes liability under the doctrine of res ipsa loquitur.

A

(C) Yes, because the farmer intended for the dog to frighten the wife.

How well did you know this?
1
Not at all
2
3
4
5
Perfectly
43
Q

A landlord leased an office space to a tenant for five years. The lease contained covenants whereby the tenant agreed to pay $750 rent per month to the landlord, and the landlord agreed to repaint the office once each year. At the beginning of the second year, the landlord refused to paint the office.

What are the tenant’s remedies?

(A) The tenant may refuse to pay rent due to the landlord’s breach of covenant.

(B) The tenant may treat the lease as terminated if he vacates the premises.

(C) The tenant may paint the office space himself and offset his subsequent rental payments by the cost of the paint and labor.

(D) The tenant may not paint the office space, because this would constitute ameliorative waste.

A

(C) The tenant may paint the office space himself and offset his subsequent rental payments by the cost of the paint and labor.

How well did you know this?
1
Not at all
2
3
4
5
Perfectly
44
Q

A shop owner told his customer that he owned a sizable tract of land. The two then entered into a written land sale contract, under which the customer would pay $75,000 and the shop owner would deliver to the customer a general warranty deed. However, unknown to the customer, the shop owner was in fact only leasing the land from a friend. The following week, the customer entered into a deal with a real estate broker. According to the terms of that contract, on October 1, the broker would pay $100,000 and the customer would deliver a general warranty deed. On September 1, the customer tendered $75,000 to the shop owner in exchange for a purported general warranty deed to the land. Then, on October 1, the broker paid the customer $100,000 in exchange for a purported general warranty deed. Two weeks later, the customer discovered that the shop owner’s friend, not the shop owner, was and remained the owner of the land. The customer then negotiated with the shop owner’s friend, and October 30, the friend gave the customer a general warranty deed in exchange for $85,000. When the broker then discovered that the customer had not held title to the land on October 1, he brought an action to recover damages from the customer. Should the broker recover?

(A) Yes, because the customer conveyed the property by general warranty deed.

(B) Yes, because the customer breached the warranty of marketable title implied in her land sale contract with the broker.

(C) No, because under the doctrine of estoppel by deed, the broker received title to the land on October 30.

(D) No, because the customer is protected from liability by the doctrine of equitable conversion.

A

(A) Yes, because the customer conveyed the property by general warranty deed.

How well did you know this?
1
Not at all
2
3
4
5
Perfectly
45
Q

A university director and a buyer entered into a written contract for the sale of a building owned by the university, constructed of marble, and situated on a half-acre lot. After the contract had been signed but before the date set for closing, a powerful tornado struck the area and totally destroyed the building. The university has no insurance to compensate for the loss of the building. The contract made no provision for the destruction of the building, and the jurisdiction has no applicable statute governing the rights of the parties. When the buyer refused to deposit the purchase price into escrow, the university director brought an appropriate action to enforce the contract via specific performance.

Which of the following best supports a judgment in favor of the university?

The doctrine of equitable conversion means that the buyer is regarded in equity as the owner of the building.

Since the buyer could have avoided loss by insuring his interest in the building, he cannot complain about having to perform under the contract.

Specific performance is available to either party in a land sale contract since each parcel of land is considered unique, the loss of which is not compensable in damages.

Since the university’s action sounds in equity, the buyer is precluded from raising the circumstances of the building’s destruction as a defense.

A

The correct answer is: The doctrine of equitable conversion means that the buyer is regarded in equity as the owner of the building.

Discussion of correct answer: The doctrine of equitable conversion allocates the risk of loss in a land sale contract to the purchaser. Thus, if damage or destruction occurs to the real property after formation but before closing, the purchaser (whom equity considers the owner of the property) bears the loss. In circumstances such as those presented by this question, the buyer is considered the equitable owner of the building from the moment the contract of sale is formed, and the subsequent destruction of the building by the earthquake is the buyer’s loss. The buyer must pay the full purchase price to the university even though the value of the property has been substantially reduced by the loss of the marble structure. A number of jurisdictions have altered the doctrine of equitable conversion so that the risk of loss remains on the seller until closing unless the purchaser takes possession. The Uniform Vendor and Purchaser Risk Act generally places the risk of loss upon the seller until the buyer has either taken possession or obtained legal title, at least as to physical damage to the property. The facts here indicate that this jurisdiction had not adopted any statute on the subject.

How well did you know this?
1
Not at all
2
3
4
5
Perfectly
46
Q

One day, a renter called the telephone company to report that the telephone line outside his house was down. Two telephone repairmen, a technician and a supervisor, responded to the call. The technician went up in the bucket truck while the supervisor remained on the ground below. In the meantime, the renter was across the street in his living room smoking marijuana, indulging in his “afternoon high.” As the technician was working on the line, the supervisor negligently bumped the bucket truck into reverse and hit the accelerator. The technician, in a panic, grabbed onto the top of the telephone pole and was left dangling. As the supervisor scrambled to get the bucket truck back underneath the technician, the renter saw everything from his window. He grabbed his chainsaw from the basement, ran across the street, and cut down the telephone pole. The technician fell to the ground and broke both arms and his collarbone. The technician sued the renter.

Which of the following is correct?

A. The renter should be held liable for negligence, unless he could have saved the technician had he not been high.

B. The renter should not be held liable for negligence, because he had no duty to take affirmative action to save the technician.

C. The renter should be held liable for negligence, because he caused the technician to fall.

D. The renter should not be held liable for negligence, because the supervisor placed the technician in peril and the renter was just trying to help.

A

C. The renter should be held liable for negligence, because he caused the technician to fall.

How well did you know this?
1
Not at all
2
3
4
5
Perfectly
47
Q

A teacher purchases a two-story home in the city and obtains a loan from the bank for $300,000. The bank files a deed of trust on the property and obtains a lien on the property. A few years later, the teacher loses her job and has trouble making payments to the bank. After three months without receiving a payment, the bank initiates foreclosure proceedings. One month later, a date is set for a public auction on the property. A few days prior to the public auction, the teacher’s uncle provides a loan to the teacher, which is more than sufficient for the teacher to pay her mortgage and stop the auction. However, the bank refuses to stop the auction and tells the teacher that she is unable to pay the debt due.

Is the bank correct that the teacher cannot stop the foreclosure proceedings?

Yes, because once the foreclosure proceedings started, the teacher lost her ability to redeem the property.

Yes, because the bank obtained a court order to conduct a public auction on the property.

No, because the teacher has the right to redeem the property before foreclosure of the property.

No, because the teacher has the right to redeem the property up to two years following foreclosure of the property.

A

The correct answer is: No, because the teacher has the right to redeem the property before foreclosure of the property.

Discussion of correct answer: A mortgagor has the right at any time after default, but before foreclosure, to redeem the property by paying the debt due. Here, a few days prior to the public auction, the teacher found the funds to pay the amount due. The bank should have worked with the teacher to assist her with her redemption of the property.

How well did you know this?
1
Not at all
2
3
4
5
Perfectly
48
Q

A lifeguard owned a beautiful sea side cottage. The lifeguard entered into a five-year residential lease agreement for the cottage with a trucker. The lease prohibited all assignments. After occupying the premises for four years, the trucker requested permission to assign the lease to a taxi driver. The lifeguard agreed to a one-time waiver of the provision in the lease which prohibited assignments, and the trucker assigned the lease to the taxi driver. One month later, the taxi driver assigned the lease to a tax attorney without requesting or receiving permission from the lifeguard. The tax attorney failed to pay rent.

Who should the lifeguard sue for the unpaid rent?

A. The tax attorney, because under the Rule in Dumpor’s Case, the assignment of the lease to the tax attorney was valid.

B. The trucker and the tax attorney.

C. The trucker and the taxi driver, because the assignment to the tax attorney was not valid.

D. The trucker, because the authorized assignee vacated the premises.

A

C. The trucker and the taxi driver, because the assignment to the tax attorney was not valid.

Discussion of correct answer: Under the Rule in Dumpor’s Case, once the landlord waives the prohibition on assignments, it is waived forever. However, the majority rule is that a landlord can preserve the validity of the anti-assignment clause by stating that the waiver is one-time only. Because the lifeguard did so here, the clause remained valid after the one-time waiver to the taxi driver. As such, the assignment to the tax attorney was invalid, and he never entered privity of estate with the lifeguard. Therefore, the lifeguard could sue either the trucker, who remains in privity of contract, or the taxi driver, who remains in privity of estate.

How well did you know this?
1
Not at all
2
3
4
5
Perfectly
49
Q

When Mamie died in 2040, she was survived by her two daughters, Sue and Ellie. Sue was a widow with one son, Aaron. Ellie was a childless widow. By her will, Mamie devised Homestead “to Sue and Ellie, as joint tenants with right of survivorship.” Sue died in 2055, leaving a will in which she devised her interest in Homestead to her son Aaron “for life, and then to his heirs and assigns.” At the time of Sue’s death, Aaron had no children. In 2060, Ellie died, leaving a will in which she devised all her real property to her good friend Burt, who had taken care of her in her final illness. In 2061, Burt conveyed Homestead to Ronnie by warranty deed. Ronnie then contracted to convey Homestead to Fern.

Which of the following statements is most accurate?

Title to Homestead is unmarketable, because the conveyance by warranty deed to Ronnie made title unmarketable.

Title to Homestead is unmarketable, because Aaron may have children sometime in the future who can contest the conveyance.

Title to Homestead is marketable, because Aaron and any children he has had could participate in the conveyance to Fern.

Title to Homestead is marketable.

A

The correct answer is: Title to Homestead is marketable.

Discussion of correct answer: When Mamie devised Homestead to Sue and Ellie, they became joint tenants with right of survivorship. When Sue died in 2055, Ellie became the sole owner of Homestead. Ellie then conveyed marketable title to Burt by will. Burt subsequently conveyed marketable title by warranty deed to Ronnie. As such, Ronnie is the sole owner of Homestead and can convey marketable title to Fern.

How well did you know this?
1
Not at all
2
3
4
5
Perfectly
50
Q

A stockbroker leased his apartment to a diplomat pursuant to a lease with a one-year term. Shortly after moving into the apartment, the diplomat installed central air conditioning in the apartment. The lease did not address renovations or improvements such as the diplomat’s installation of air conditioning.

In most jurisdictions, is the diplomat’s action permissible under the lease?

(A) Yes, because the diplomat improved the value of the premises.

(B) Yes, because the lease does not prohibit it.

(C) No, because it constitutes permissive waste.

(D) No, because it constitutes ameliorative waste.

A

(D) No, because it constitutes ameliorative waste.

How well did you know this?
1
Not at all
2
3
4
5
Perfectly
51
Q

A homeowner has a mortgage with ABC Bank. The homeowner loses his job and has no income or ability to make the required monthly payments under the mortgage. The homeowner’s friend is looking for a home to purchase. Without notice to the bank, the homeowner sells the property to the friend, who agrees to make the payments. Three months later, the friend loses his job and is unable to satisfy the monthly mortgage payments.

Is the friend liable to the bank?

A. Yes, because the friend agreed to the terms of sale and, as transferee, the friend owns and possesses the property.

B. Yes, because the friend is in breach of the terms of the mortgage.

C. No, because the bank may not foreclose if the property has been sold to the friend, a third party.

D. No, if the friend agreed to purchase the property subject to the mortgage.

A

D. No, if the friend agreed to purchase the property subject to the mortgage.

How well did you know this?
1
Not at all
2
3
4
5
Perfectly
52
Q

A tenant rented a furnished, two-bedroom apartment from a landlord under a short-term lease. One evening, the tenant came home to find that the wall between the two bedrooms had collapsed, making the apartment structurally unsound. The tenant immediately informed the landlord of the problem. Fearing injury or further damage to his personal property, the tenant moved in with his sister. Two weeks went by and the next month’s rent became due, but the landlord had made no attempt to fix the damage.

Is the tenant required to stay in the lease?

A No, because the landlord constructively evicted the tenant.
B No, because the landlord actually evicted the tenant.
C Yes, and the tenant must repair the damage.
D Yes, and the tenant must allow more time to repair the damage.

A

A No, because the landlord constructively evicted the tenant.

How well did you know this?
1
Not at all
2
3
4
5
Perfectly
53
Q

A train conductor had some beautiful land in the desert. He gave his widowed friend permission to camp there for four months. The friend fell in love with the area and built a small house on the land without telling the conductor and raised his son there. After 25 years there, the friend died and left the land to his son. The conductor died the following year. Seven years later, the friend’s son bought some nearby land, combined it with his own, and built a profitable resort. The hotel, the centerpiece of the resort, was located on the land that had originally belonged to the conductor. When the conductor’s heirs learned about the hotel and resort, they sued to have the hotel declared theirs. The period of time required to acquire title by adverse possession in their jurisdiction is 30 years.

How should the court rule?

(A) The land and the hotel belong to the friend’s son.

(B) The hotel belongs to the friend’s son, but the land belongs to the conductor’s heirs.

(C) The land and the hotel belong to the conductor’s heirs.

(D) The hotel belongs to conductor’s heirs, but the land belongs to the friend’s son.

A

(A) The land and the hotel belong to the friend’s son.

How well did you know this?
1
Not at all
2
3
4
5
Perfectly
54
Q

A lawyer handed his son a deed granting the lawyer’s land “to my only granddaughter.” The lawyer instructed his son not to give the deed to the granddaughter unless and until she survived her parents. In his conversation with his son, the lawyer expressly retained the right to retrieve the deed. The son placed the deed in a desk drawer. Alas, the granddaughter discovered the deed and promptly recorded it in accordance with the state’s recording act. Three months later, the lawyer was involved in an automobile accident with another driver. The driver sued the lawyer and was awarded a $100,000 judgment. Having no actual notice of the conveyance to the granddaughter, the driver recorded a judgment lien against the lawyer’s land and sought enforcement of the lien. The state recording act reads, “Every unrecorded conveyance of real property is void against any subsequent purchaser for consideration or mortgagee of the same property whose conveyance is first duly recorded, and as against any subsequent judgment on record affecting the title.” If the court finds in favor of the driver and enforces the judgment lien against the land, it will most likely be for which of the following reasons?

(A) The recording act expressly protects subsequent judgment creditors such as the driver.

(B) As a donee, the granddaughter is not protected by the recording act, which expressly protects only purchasers for consideration, mortgagees, and judgment creditors.

(C) The conveyance by the lawyer to his granddaughter was not effective.

(D) The driver had no actual notice of the granddaughter’s interest in the property.

A

(C) The conveyance by the lawyer to his granddaughter was not effective.

How well did you know this?
1
Not at all
2
3
4
5
Perfectly
55
Q

A landlord and a business executive entered into a written lease agreement for a penthouse apartment for a period of five years, with rent to be paid monthly. Two years later, the executive was reassigned to an office in another state for one year. The executive leased the apartment to a coworker for the duration of his reassignment. The coworker promptly moved in, but never paid any rent. After nine months, the coworker was fired, and he moved out of the apartment and back in with his parents. Upon returning home from his reassignment, the executive had barely unpacked before he was promptly served with a lawsuit filed by the landlord against both the executive and the coworker for the previous year’s unpaid rent.

Assuming the landlord is successful, how will the court rule regarding damages?

A Against the coworker individually for the full year’s rent, and no judgment against the executive.
B Against the executive for the full year’s rent, and no judgment against the coworker.
C Against the executive individually for three months’ rent, and against the coworker individually for nine months’ rent.
D Against the executive and coworker for the full year’s rent, because they are jointly and severally liable

A

The correct answer is: B. Against the executive for the full year’s rent, and no judgment against the coworker.

Discussion of correct answer: A sublease occurs when a tenant transfers to a third person (the subtenant, or sublessee) less than all of his rights, title, and interest in the leased premises. A subtenant does not come into privity of estate with the original landlord. Therefore, absent an express assumption of the duty to pay rent by the subtenant, the landlord may not sue the subtenant directly for unpaid rent. Here, the coworker was a sublessee, because his lease was for only one of the three years remaining on the lease term. As such, the landlord has no cause of action against the coworker, and must instead recover all of the unpaid rent from the original tenant, the business executive.

Discussion of incorrect answers:

Incorrect. Against the coworker individually for the full year’s rent, and no judgment against the executive. Because the coworker was a sublessee, he was neither in privity of estate nor private of contract with the landlord. As such, the landlord has no cause of action against him. The landlord must instead recover any or all of the unpaid rent from the original tenant.

Incorrect. Against the executive individually for three months’ rent, and against the coworker individually for nine months’ rent. While the fact that the coworker abandoned the premises may affect his liability for unpaid rent to the business executive, the coworker will not be directly liable to the landlord at all, because he was a subtenant, and so never came into privity with the landlord. Instead, the business executive, as the original tenant, will remain liable for the entire unpaid rent.

Incorrect. Against the executive and coworker for the full year’s rent, because they are jointly and severally liable. Because the coworker was a sublessee, he was neither in privity of estate nor private of contract with the landlord. As such, the landlord has no cause of action against him, either jointly or severally. The landlord must instead recover any or all of the unpaid rent from the original tenant alone.

How well did you know this?
1
Not at all
2
3
4
5
Perfectly
56
Q

A landowner owned a vacant lot. The landowner agreed to sell the vacant lot to a purchaser for $30,000. The purchaser intended to build and operate a Christmas tree shop thereon. The written agreement was complete in all respects and provided, among other things, that on the date set for closing, nine months into the future, the landowner would convey “good and marketable title” to the vacant lot by depositing into escrow a general warranty deed to that effect. Four weeks after the contract of sale was signed, the purchaser sought the landowner’s permission to enter into possession of the vacant lot. The landowner orally agreed. Unbeknownst to the landowner, the purchaser subsequently planted 50 pine trees on the vacant lot. The presence of the pine trees, which had cost the purchaser $10,000 to plant, increased the market value of the vacant lot by $8,000. One month before the date set for closing, the purchaser abandoned possession of the vacant lot and informed the landowner that he was rescinding the land sale contract because he was leaving the area in order to join an ongoing menorah-selling business in another city. When the landowner refused to pay the purchaser $10,000 in reimbursement for the cost of planting the pine trees, the purchaser brought an appropriate action to recover that amount from the landowner.

Judgment should be awarded for whom?

The landowner, because his assent to the purchaser’s possession was not in writing.

The landowner, because the purchaser repudiated the land sale contract.

The purchaser, in the amount of $8,000, because the landowner’s permission for the purchaser to take possession of the vacant lot under these circumstances constituted tacit permission to make improvements to the property.

The purchaser, in the amount of $10,000, because he is subrogated to the rights of the contractor who planted the pine trees.

A

The correct answer is: The landowner, because the purchaser repudiated the land sale contract.

Discussion of correct answer: In land sale contracts, where the performance is to pay money rather than render services, the court’s first reaction to a breaching purchaser was to allow the seller to retain any down payment, even if the amount so paid exceeded what the seller might have obtained in damages. There has been a movement to permit a defaulting buyer to obtain restitution of any payments made in advance to the extent that they exceed the actual damages suffered by the aggrieved seller. Even under circumstances where restitution was readily allowed, as in building construction contracts, restitution would be denied if the breach was willful, or for the convenience or financial advantage of the breaching party. In any case, the courts do not uniformly grant restitution to breaching buyers in land-sale contracts. In this question, the purchaser went into possession of the real property and without the knowledge of the landowner made improvements thereon. The purchaser then breached, for his own convenience (to pursue a business opportunity in another city), and now seeks to compel the landowner to pay for something the landowner never bargained for. Under these circumstances, the case for restitution to the purchaser is very weak.

How well did you know this?
1
Not at all
2
3
4
5
Perfectly
57
Q

The owner of a limousine business decided to retire and convey his building to his daughter. The owner hoped that his daughter would take over the business and settle down. The owner conveyed the building by quit claim deed for the sum of one dollar “to my daughter and her heirs for so long as the premises are used for a limousine business.” As soon as the owner retired and moved to California, his daughter converted the building to a dance club and abandoned the limousine business.

Did the estate automatically revert to the owner?

Yes, because the estate was defeasible.

Yes, because the estate was a fee simple determinable with the possibility of reverter.

No, because the estate was a fee simple determinable.

No, because the estate was a fee simple subject to condition subsequent.

A

The correct answer is: Yes, because the estate was a fee simple determinable with the possibility of reverter.

Discussion of correct answer: A determinable estate terminates automatically on the happening of a named future event. A determinable estate is described with the following words: “for so long as,” “during,” “while,” or “until.” The language “to my daughter and her heirs for so long as the premises are used for a limousine business” created a fee simple determinable with the possibility of reverter. The estate automatically reverted to the owner when his daughter ended the limousine business.

How well did you know this?
1
Not at all
2
3
4
5
Perfectly
58
Q

A young man and his friend were cliff jumping from a popular jump site. As the young man approached the edge of the cliff, he said that he saw a person at the bottom. The friend, a more experienced jumper who had introduced the young man to the sport, looked down and said to go jump and that the young man will land nowhere near the person at the bottom. The young man was uncertain, but jumped anyway and lost control of his parachute. A woman was swimming in a quarry near the landing site. The young man released his parachute as he spun over the quarry and landed on top of the woman. The woman lost consciousness and suffered a broken arm as the young man struggled to drag her ashore.
Who bears liability for the accident?

A. The young man only, because his conduct was the legal cause of the accident.

B. Neither the young man nor the friend, because the woman was contributorily negligent by swimming in the quarry.

C. Both the young man and his friend, because they were joint tortfeasors.

D. The friend only, because his encouragement initiated the accident.

A

C. Both the young man and his friend, because they were joint tortfeasors.

How well did you know this?
1
Not at all
2
3
4
5
Perfectly
59
Q

A locksmith and a tennis pro entered into a written lease agreement for the lease of an apartment owned by the locksmith. Under the terms of the lease agreement, the tennis pro was to rent the apartment for $2,000 per month from the locksmith for a period of two years. On the day the lease commenced, the tennis pro found he could not gain access to the apartment because the locksmith had changed the locks and had allowed his girlfriend to move into the apartment.

Has the locksmith breached his duty?

A. No, under the minority rule, because the tennis pro has a legal right to possess the premises.

B. No, under the majority rule, because a lease for a term of years may be terminated at will.

C. Yes, under the minority rule, because the landlord has constructively evicted the tennis pro.

D. Yes, under the American rule, because the locksmith has not delivered actual possession of the premises to the tennis pro.

A

A. No, under the minority rule, because the tennis pro has a legal right to possess the premises.

How well did you know this?
1
Not at all
2
3
4
5
Perfectly
60
Q

A musician landed a lucrative recording contract. Soon thereafter, the musician entered into a contract with a husband and wife to purchase their residence for $1,000,000. The purchase contract required the musician to make a deposit of $10,000 as a down payment but did not contain a liquidated damages provision. Shortly thereafter, the musician’s recording contract was terminated when he was arrested for drug use. The musician notified the husband and wife that he would be unable to proceed with the purchase of their home. He requested his $10,000 deposit back to pay bail, but the husband and wife refused. On the day that the musician was to close on the husband and wife’s home, another buyer purchased the home for $1,100,000.

If the musician brings an action against the husband and wife to recover his $10,000 and wins, it will be because of what reason?

The termination of the musician’s recording contract was wrongful.

The deposit amount was excessive.

Absent a liquidated damages provision, a down payment is always refundable.

The husband and wife were able to sell their property to another buyer for $1,100,000 soon after the musician refused to perform.

A

The correct answer is: The husband and wife were able to sell their property to another buyer for $1,100,000 soon after the musician refused to perform.

Discussion of correct answer: Even without a liquidated damages provision, a seller may elect to retain a down payment in the event of a breach by the buyer but the modern approach limits the amount to no more than 10% of the purchase price and courts may require the seller to return to the buyer the amount that the down payment exceeds the actual damages caused by the breach. In this case, the husband and wife did not suffer any damages as a result of the breach because they were able to sell the house in the same time frame as originally anticipated and for more than the original contract price. A court would likely require the couple to return the $10,000 to the musician for this reason.

How well did you know this?
1
Not at all
2
3
4
5
Perfectly
61
Q

A rancher owned a ranch. In order to protect his livestock from wolves, the rancher erected a fence with electrical wiring around his property. The wiring emitted a shock upon contact with the fence, but the voltage was of such low intensity that it posed no risk of injury to a human being. Consequently, the rancher did not post any notice of the electrical wiring. One day, a cowboy was riding his horse along the rancher’s property when a gust of wind blew his hat onto the fence. The cowboy had a pacemaker. When the cowboy touched the fence to retrieve his hat, the electric current short-circuited his pacemaker, killing him. The cowboy’s estate brought a wrongful death action against the rancher.

Which of the following would the electrical wiring device constitute?

A. The legal cause of the cowboy’s death, but not the cause-in-fact.

B. The cause-in-fact of the cowboy’s death, but not the legal cause.

C. The legal cause and the cause-in-fact of the cowboy’s death.

D. Neither the legal cause nor the cause in-fact of the cowboy’s death.

A

B. The cause-in-fact of the cowboy’s death, but not the legal cause.

How well did you know this?
1
Not at all
2
3
4
5
Perfectly
62
Q

A professor rented a single family home in a small college town. The valid written lease ran from August to August with rental payments of $1,500 due on the first of each month. In December the professor traveled home for the holidays. When he returned he found that the first floor of the house had flooded during a recent storm and the living room carpeting had been damaged. He informed the landlord, but when nothing had been done by the end of January the professor decided to withhold February’s rent. Although the carpeting had dried, it had discolored and buckled in several places.

March came with no word from the landlord, and the professor packed his things and moved out of the house. There is no applicable statute in the jurisdiction.

If the landlord sues the professor for breach of the lease, which of the following is accurate?

A The landlord will win, because he was not given sufficient time to repair the carpeting.
B The landlord will win, because the professor was not constructively evicted.
C The professor will win, because his obligation to pay rent ceased when the landlord failed to repair the carpeting.
D The professor will win, because the landlord breached the implied warranty of habitability.

A

B The landlord will win, because the professor was not constructively evicted.

How well did you know this?
1
Not at all
2
3
4
5
Perfectly
63
Q

A father’s daughter had recently applied for admission to a prestigious college. The father knew that if the daughter were accepted by the college he would be unable to pay her tuition from his savings. The father approached his wealthy uncle, explained the situation to him, and asked if the uncle would be willing to lend him money to cover the daughter’s tuition. The uncle responded that he would be willing to lend up to $10,000 per year, but would require security that the debt be paid. In exchange for the uncle’s promise to lend the tuition money, the father executed a mortgage in favor of the uncle on Whiteacre, a parcel of land owned by the father. The uncle promptly recorded this mortgage.

A month later the daughter received a letter stating that she was not accepted for admission by the college. The daughter then enrolled at a local community college. Since the tuition at the community college was far less than at the prestigious college the father did not need to borrow money from the uncle.

One year later, the father died in an auto accident. The daughter, as the father’s sole heir, inherited all of the father’s estate. She discovered that, six months prior to executing the mortgage in favor of the uncle, the father had issued a mortgage on Whiteacre to a bank in exchange for a loan. Neither the daughter nor the uncle knew of this mortgage, and the bank failed to record it. The jurisdiction has the following recording statute in effect:

“An unrecorded conveyance, mortgage or other instrument is invalid as against a subsequent bona fide purchaser for value and without notice.”

Which mortgage would have top priority?

A The uncle’s mortgage, because at the time of recordation, the uncle had no knowledge of the mortgage to the bank.
B The uncle’s mortgage, because his promise to pay tuition would be sufficient to constitute “paying value.”
C The bank’s mortgage, because recording acts do not operate in favor of mortgagees.
D The bank’s mortgage, because the uncle was not a bona fide purchaser.

A

D The bank’s mortgage, because the uncle was not a bona fide purchaser.

How well did you know this?
1
Not at all
2
3
4
5
Perfectly
64
Q

A property owner borrowed money from a friend in order to make improvements. As security, the owner granted his friend a mortgage “on all the real property I own in [the county]” and promised that the mortgage would be repaid in no more than fifteen years. The friend recorded the mortgage. A few months later, the owner sold some of his property to a buyer. The owner then defaulted on the mortgage, and the friend attempted to foreclose on the buyer’s property. The buyer objected to the foreclosure, arguing that the friend did not have a mortgage on his property.

Does the friend have a mortgage on the buyer’s property?

(A) No, because the buyer was obligated to satisfy the mortgage when the property was sold.

(B) No, because the description of the property was not sufficient.

(C) Yes, because a mortgage deed was executed.

(D) Yes, because there was evidence of an intent to create a security interest.

A

(B) No, because the description of the property was not sufficient.

How well did you know this?
1
Not at all
2
3
4
5
Perfectly
65
Q

A landlord and a tenant entered into a lease for a period of one year for an apartment. Three months into the lease, the floor collapsed under the living room and part of the bedroom due to work that the landlord was doing in the apartment below. When the landlord told the tenant that it would take at least three months to repair the floors, the tenant moved out. When the tenant refused to pay rent the next month, the landlord sued the tenant.

Is the tenant obligated to continue paying rent?

(A) No, because the covenant of quiet enjoyment has been breached.

(B) No, because the tenant has been actually evicted from the property.

(C) Yes, because although the covenant of quiet enjoyment has been breached, the tenant is only entitled to a rent abatement.

(D) Yes, because the landlord is working to repair the apartment.

A

(A) No, because the covenant of quiet enjoyment has been breached.

The correct answer is:No, because the covenant of quiet enjoyment has been breached.

Discussion of correct answer:Every lease contains a covenant of quiet enjoyment (express or implied). A landlord is deemed to have breached this covenant if the landlord actually or constructively evicts a tenant. A constructive eviction occurs when: (1) the landlord’s act substantially and permanently interferes with the tenant’s use and enjoyment of the premises; and (2) the tenant moves out. In this case, both of these factors are present. When a tenant has been constructively evicted, he may treat the lease as terminated and withhold rent.

Discussion of incorrect answers:

Incorrect. No, because the tenant has been actually evicted from the property. The tenant here has been constructively, not actually, evicted from the property. An actual eviction occurs when the landlord, or someone claiming through him, disturbs the landlord in his possession of the property. In this situation, the tenant may treat the lease as terminated and withhold rent. However, when the doesn’t actually evict the tenant, but instead, substantially and permanently interferes with the tenant’s use and enjoyment of the property such that the tenant moves out, the landlord is considered to have constructively evicted the tenant.

Incorrect. Yes, because although the covenant of quiet enjoyment has been breached, the tenant is only entitled to a rent abatement. When the tenant has been constructively evicted from only part of the property, there is still a breach of the covenant of quiet enjoyment, but the tenant will be entitled to receive only a rent abatement, and not to withhold rent entirely. However, when the interference is substantial and the tenant actually moves out, the tenant will be considered to have been constructively evicted from the entire premises. When this occurs, the tenant is entitled to treat the lease as terminated and withhold rent.

Incorrect. Yes, because the landlord is working to repair the apartment. The fact that the landlord is working on repairing the restaurant is not relevant here. The landlord has constructively evicted the tenant from the property, and the tenant has actually moved out. As such, the tenant is entitled to treat the lease as terminated and withhold rent.

How well did you know this?
1
Not at all
2
3
4
5
Perfectly
66
Q

A bicycle helmet was designed to be lightweight. The helmet maker knew that its design would provide protection only for low-impact crashes, so it warned users not to use the helmet when using “any motorized vehicle.” A cyclist owned one of the helmets and decided to wear it hang gliding because a hang glider is not motorized. When the cyclist was running down the hill to take off, another glider negligently crashed into the cyclist. The cyclist hit his head, but the helmet did not protect him. He sustained permanent injuries as a result of his fall.

Will the helmet maker be held strictly liable for these injuries?

(A) No, because the cyclist misused the helmet.
(B) No, because the other glider’s negligence was a superseding cause of the cyclist’s injuries.
(C) Yes, because the helmet had a design defect.
(D) Yes, because the helmet had a manufacturing defect.

A

(C) Yes, because the helmet had a design defect.

How well did you know this?
1
Not at all
2
3
4
5
Perfectly
67
Q

A landlord and a tenant entered into an oral lease for a term of six months. After the commencement of the lease term, the ceiling of the leased apartment caved in as a result of renovations the landlord was making to the apartment above. Unable to remain in the apartment, the tenant moved out.

What are the tenant’s remedies at common law?

(A) Treat the lease as terminated and withhold rent, because the covenant of quiet enjoyment has been breached.

(B) Reduce the rent payments, because although the covenant of quiet enjoyment has been breached, the lease is not terminated.

(C) Unilaterally terminate the lease and discontinue rent payments.

(D) No remedies exist at common law, because an oral lease does not contain a covenant of quiet enjoyment.

A

(A) Treat the lease as terminated and withhold rent, because the covenant of quiet enjoyment has been breached.

How well did you know this?
1
Not at all
2
3
4
5
Perfectly
68
Q

A minister had been counseling a married couple. During one session, the husband accused the wife of having an extramarital affair, which the wife admitted. That Sunday, the minister gave a sermon against adultery. The minister did not mention any names or make any statements that would lead anyone to believe that the sermon was about a particular person. Nevertheless, the wife left the sermon extremely embarrassed, and she hid in her bedroom for several days.

In an action against the minister for intentional infliction of emotional distress, will the wife prevail?

(A) No, because preaching a sermon that talked about adultery was not outrageous conduct.
(B) No, because the wife did not sustain any physical harm as a result of her distress.
(C) Yes, because preaching a sermon that talked about adultery was outrageous conduct in these circumstances.
(D) Yes, because the minister was in a position of authority over the married couple.

A

(A) No, because preaching a sermon that talked about adultery was not outrageous conduct.

How well did you know this?
1
Not at all
2
3
4
5
Perfectly
69
Q

A developer owned a strip mall in a busy area of the city, and leased the storefronts to various businesses pursuant to year-to-year periodic lease agreements. Deciding to retire, the developer sold his interests to a business investor and assigned all of his lease agreements to the investor. None of the written lease agreements contained clauses addressing assignments.

Which of the following is true?

A The developer remains the landlord under the terms of the leases, because the assignments are invalid.

B The investor is now the landlord under the terms of the leases, because a landlord may assign his lease interest.

C The investor is the landlord under the terms of the leases, because the leases were commercial.

D The leases terminated upon the developer’s attempted assignment.

A

B The investor is now the landlord under the terms of the leases, because a landlord may assign his lease interest.

The correct answer is:The investor is now the landlord under the terms of the leases, because a landlord may assign his lease interest.

Discussion of correct answer:Only an at-will tenancy terminates when one of the parties attempts to assign his or her interest. By contrast, periodic leases, such as the year-to-year leases in this case, do not terminate upon a party’s assignment. As such, absent a lease term to the contrary, it was permissible for the developer, as landlord, to assign his lease interests to the business investor, who then became the new landlord under the lease agreements.

Discussion of incorrect answers:

Incorrect. The developer remains the landlord under the terms of the leases, because the assignments are invalid. There is nothing in the facts to suggest that the developer’s assignment of his lease interests to the business investor is invalid. The leases did not prohibit assignments, and the only type of lease that terminates upon a party’s attempted assignment is an at-will lease. Because the year-to-year leases in this question are periodic leases, not at-will leases, it was permissible for the developer to assign his lease interests.

Incorrect. The investor is the landlord under the terms of the leases, because the leases were commercial. The fact that the leases are commercial leases is not relevant to the question of whether the developer’s assignments were valid. The only type of lease that is subject to termination when it is assigned is an at-will lease (although any lease may contain a prohibition against assignments). Because the leases in this question are periodic leases that do not contain any prohibitions against assignments, the developer’s assignment of the leases to the business investor was valid.

Incorrect. The leases terminated upon the developer’s attempted assignment. The lease did not terminate upon the developer’s assignment. Unlike an at-will lease, which terminates if either party attempts to assign his lease interest, periodic leases such as the year-to-year leases in this question do not terminate upon the landlord’s assignment of his lease interest. As such, this answer choice is incorrect.

How well did you know this?
1
Not at all
2
3
4
5
Perfectly
70
Q

A lawyer leased his house to a travel agent for a term of four years. Sixteen months after the lease commenced, the house was destroyed when a gas leak caused a massive explosion. No one was injured. The travel agent found another place to rent and refused to pay the lawyer. The lawyer filed suit to collect rent. The jurisdiction follows the old common law rule, not the modern common law rule.

Who should prevail?

A The travel agent, because the lease is a residential lease.
B The travel agent, because the lease terminated with the destruction of the house.
C The lawyer, because the house was not destroyed by an act of God.
D The lawyer, because the lease term has not expired.

A

D The lawyer, because the lease term has not expired.

Discussion of correct answer: Under the old common law estate theory, a tenant is still required to pay rent if the premises are destroyed during the lease term. In this case, given that the house was destroyed while the lease was still in effect, the travel agent will, under the old common law rule, be required to continue paying rent. Therefore, this is the correct answer choice.

How well did you know this?
1
Not at all
2
3
4
5
Perfectly
71
Q

When Mamie died in 2040, she was survived by her two daughters, Sue and Ellie. Sue was a widow with one son, Aaron. Ellie was a childless widow. By her will, Mamie devised Homestead “to Sue and Ellie, as joint tenants with right of survivorship.” Sue died in 2055, leaving a will in which she devised her interest in Homestead to her son Aaron “for life, and then to his heirs and assigns.” At the time of Sue’s death, Aaron had no children. In 2060, Ellie died, leaving a will in which she devised all her real property to her good friend Burt, who had taken care of her in her final illness. In 2061, Burt conveyed Homestead to Ronnie by warranty deed. Ronnie then contracted to convey Homestead to Fern.

Which of the following statements is most accurate?

(A) Title to Homestead is unmarketable, because the conveyance by warranty deed to Ronnie made title unmarketable.

(B) Title to Homestead is unmarketable, because Aaron may have children sometime in the future who can contest the conveyance.

(C) Title to Homestead is marketable, because Aaron and any children he has had could participate in the conveyance to Fern.

(C) Title to Homestead is marketable.

A

(C) Title to Homestead is marketable.

How well did you know this?
1
Not at all
2
3
4
5
Perfectly
72
Q

A talented florist took great pride in the gardens on his estate, his dress, and his political party membership. He had donated flowers to decorate the White House of every President of his chosen party for many decades. To ensure that his flowers would grace only the homes of those he felt worthy, his will left his real property to his nephew for as long as the nephew donated flowers to Presidents of his chosen party and to no others. The florist also devised all of his personal property to his nephew, except for an account containing $100. The florist’s only other heir was his nonmarital daughter, to whom he left the residue of his estate. Several decades later, the nephew became a generous supporter of a different political party. When that party’s candidate was elected, the nephew was given the honor of supplying the flowers for that party’s President’s inauguration. Upon hearing of the plans, the nonmarital daughter filed suit to quiet title.

If the court finds in the nonmarital daughter’s favor, what is the most likely reason?

(A) The nonmarital daughter held a valid possibility of reverter.

(B) The gift to the nephew violated the Rule Against Perpetuities.

(C) The nonmarital daughter inherited a valid reversionary interest.

(D) The gift to the nephew failed, because the conditions of the gift were void as against public policy.

A

(A) The nonmarital daughter held a valid possibility of reverter.

How well did you know this?
1
Not at all
2
3
4
5
Perfectly
73
Q

A gardener’s neighbor had won the local rose competition every year for the last 11 years. A week before the annual competition, the gardener watched as his neighbor watered his prize-winning roses. The gardener noted how small his own roses were in comparison to the neighbor’s roses. Extremely upset at the prospect of losing the contest to the neighbor yet again, the gardener ran into the neighbor’s yard and started stomping his roses. The neighbor yelled, “Stop, stop! My babies!” However, the gardener continued to damage the neighbor’s rose garden. The neighbor picked up his gardening shovel and swung it at the gardener to stop him from destroying all of the roses. The gardener was hit by the shovel and fell to the ground in pain. Beside himself with anger, the neighbor began hitting the gardener over the head with the shovel, eventually rendering him unconscious. The gardener’s wife saw the gardener being beaten with the shovel and ran outside screaming for the neighbor to stop, but the neighbor would not. The gardener’s wife picked up a pair of weeding shears and stabbed the neighbor in the face with them, injuring the neighbor severely. The neighbor filed suit against the gardener and his wife for trespass to land, battery, and intentional infliction of emotional distress.

For which claims will the neighbor likely prevail at trial?

(A) Trespass to land, battery, and intentional infliction of emotional distress.
(B) Trespass to land and battery, but not for intentional infliction of emotional distress.
(C) Trespass to land only.
(D) Trespass to land and intentional infliction of emotional distress only, but not for a battery.

A

(D) Trespass to land and intentional infliction of emotional distress only, but not for a battery.

How well did you know this?
1
Not at all
2
3
4
5
Perfectly
74
Q

A seller contracted to sell a parcel of property to a buyer by general warranty deed. The contract is enforceable and contains all the elements necessary to satisfy the Statute of Frauds. At the closing, the seller tendered a general warranty deed, but the buyer refused the deed and would not complete the transaction because she had discovered an unpaid mortgage on the property.

The buyer asserts the mortgage renders the property not “free of encumbrances” as required by the contract. The amount of the mortgage is less than the agreed-upon purchase price for the parcel of property. The seller contends that he intends to pay off the mortgage from proceeds of the sale of the property to the buyer. The seller brings a cause of action against the buyer for specific performance.

If a court finds for the seller and compels specific performance, which of the following is the best explanation for the court’s ruling?

The seller hasn’t breached any portion of the contract.

The seller has the implied right to use the proceeds from the sale of the parcel of land to the buyer to satisfy the mortgage and clear title to the property.

Under the doctrine of equitable conversion, title has already passed to the buyer, and she is therefore obligated to pay the agreed-upon purchase price.

The lien of the mortgage will automatically shift away from the parcel of land and attach to the seller’s proceeds from the sale.

A

The correct answer is: The seller has the implied right to use the proceeds from the sale of the parcel of land to the buyer to satisfy the mortgage and clear title to the property.

Discussion of correct answer: The existence of unpaid liens or mortgages will not constitute a breach of the purchase contract if the proceeds from the sale are sufficient and, in fact, used to pay off the encumbrance. The facts state that the proceeds from the sale of the parcel of land will be more than enough for the seller to pay off the mortgage and clear title to the property at closing.

How well did you know this?
1
Not at all
2
3
4
5
Perfectly
75
Q

A seller entered into a written contract with a buyer whereby the buyer was to purchase Brownacre, an undeveloped parcel consisting of 100 acres, for $250,000. The contract provided that, “because of economic uncertainties, time is of the essence.” The contract further provided for a closing date of June 30, specified that all warranties and covenants of title would merge into the deed after closing, and required the buyer to deliver half the purchase price to the seller by June 15 and the balance on the date of closing.

The buyer failed to make the payment of one-half of the purchase price on June 15. On June 20, the buyer tendered a check for $125,000 to the seller, which the seller accepted. On June 30, the buyer tendered the remaining $125,000 and the seller delivered a deed to Brownacre to the buyer. The seller then sued the buyer for damages caused by the delay in tendering the first payment of the purchase price.

What would be the result in that lawsuit?

The seller will prevail, because he tendered the deed to the buyer in a timely fashion.

The seller will prevail, because the acceptance of the late performance did not waive the right to recover damages.

The buyer will prevail, because the short delay did not result in a material breach of the land-sale contract.

The buyer will prevail, because the seller waived any right to damages by accepting the late performance.

A

The correct answer is: The seller will prevail, because the acceptance of the late performance did not waive the right to recover damages.

Discussion of correct answer: As a general rule, time is not of the essence in a real estate contract unless the parties state otherwise. Here, the contract specifically stated that time was of the essence. This means that the buyer’s late performance was a breach of the contract. The seller could have treated the late performance as a material breach of the contract, discharging his obligation to deliver the deed to the buyer. However, by accepting the late performance the seller is electing to treat the late performance as a minor breach, entitling the seller to delay damages.

How well did you know this?
1
Not at all
2
3
4
5
Perfectly
76
Q

Dave and Sally were brother and sister. Their father Eddie died leaving a valid will. In his will, Eddie devised, “my house to both my children Dave and Sally.” Since the house was their childhood home, both Dave and Sally held a sentimental attachment to the property. One week after the funeral, Dave moved into the house. About three months later, Sally decided to visit the house. She was shocked to find out that Dave had rented her old room to her ex-husband Harry. Sally hired an attorney who discovered that Dave had recently taken out a mortgage on the house. Further, Dave already missed a mortgage payment.

Which of the following is correct?

Dave and Sally took the property as joint tenants, hence the tenancy was severed by Dave’s actions.

Dave and Sally took the property as tenants in common, hence the tenancy was not severed by Dave’s actions.

Dave and Sally took the property as joint tenants, hence Dave did not have the right to rent any rooms.

Dave and Sally took the property as tenants in common, hence Dave did not have the right to mortgage the house.

A

The correct answer is: Dave and Sally took the property as tenants in common, hence the tenancy was not severed by Dave’s actions.

Discussion of correct answer: Tenancy in common is a form of concurrent ownership, where each co-tenant owns an undivided interest in the whole of the property with no right of survivorship. No special words are required to create a tenancy in common. Tenancy in common requires unity of possession only. Unity of possession means that each tenant in common has the right to possess the whole of the property. A tenant in common may transfer her interest through a conveyance, lease, mortgage, or other transfer of a present possessory or future property interest. Dave and Sally took as tenants in common because of the language of the deed “my house to both my children Dave and Sally.” Dave has the right to rent a room and to mortgage the house. A co-tenant may seek partition through judicial action; however, the co-tenancy was not severed by Dave’s actions (as opposed to a joint tenancy).

How well did you know this?
1
Not at all
2
3
4
5
Perfectly
77
Q

A linguist and a teacher entered into a five-year residential lease agreement for a house owned by the linguist. The lease prohibited subleases. One year into the lease term, the teacher asked the linguist for permission to sublease the house, despite the provision in the lease prohibiting subleases. The linguist agreed, and the teacher signed a two-year sublease with a sailor. After the termination of this sublease, the teacher asked the linguist for permission to enter into a one-year sublease with a scientist. Even though the scientist’s income and assets indicated that he would easily be able to make the monthly rental payments, the linguist refused to grant permission for the sublease to the scientist. The teacher and the scientist signed the second sublease. The linguist filed suit against the teacher.

Who should prevail?

A. The teacher, because a landlord may not unreasonably withhold approval of a sublease.

B. The teacher, because under the Rule in Dumpor’s Case, a prohibition on assignment of a lease is unenforceable.

C. The linguist, because her oral waiver of the provision prohibiting subleases was ineffective.

D. The linguist, because she withheld her consent to the second sublease.

A

D. The linguist, because she withheld her consent to the second sublease.

How well did you know this?
1
Not at all
2
3
4
5
Perfectly
78
Q

A woman executed a mortgage with a bank for the purchase of a home. For the first few years of her 15-year mortgage, she had no trouble making timely payments on the mortgage note. One year, however, she got behind on her mortgage payments and, for five months, made no payments at all. The woman received notice from the bank indicating that she had defaulted on the loan and advising her of its intent to engage in foreclosure proceedings. Realizing that the home was worth quite a bit more than it was when she purchased it, the woman tried to sell it herself rather than face foreclosure but was unsuccessful. The bank subsequently foreclosed on the home and the sale price was more than sufficient to cover the outstanding debt. The woman contacted the bank to inquire about any remaining proceeds from the sale, and the bank told her that it had retained the remaining proceeds from the foreclosure sale as rent for the months that she lived in the house after she defaulted on the mortgage.

If the woman is successful in recovering the remaining proceeds, which of the following provides the best reason?

(B) She lives in a jurisdiction that adheres to the title theory of mortgage interests.

(D) She lives in a jurisdiction that adheres to the equitable theory of mortgage interests.

(A) She lives in a jurisdiction that adheres to the intermediate theory of mortgage interests.

(C) She lives in a jurisdiction that adheres to the lien theory of mortgage interests.

A

(C) She lives in a jurisdiction that adheres to the lien theory of mortgage interests.

How well did you know this?
1
Not at all
2
3
4
5
Perfectly
79
Q

A skier purchased a parcel of land from a landowner when she was 30 years old. The skier intended to develop the parcel as a ski resort. The next year, however, the skier suffered a head injury and was declared legally incompetent before she could make any improvements to the land. Five years later, a snowboarder purchased the parcel next to the skier’s parcel. The snowboarder built a large ski resort. A few years later, when the injured skier was 40 years old, the snowboarder inadvertently expanded her resort onto the skier’s land. The skier never recovered from her head injury and died at age 61. The skier’s daughter inherited the skier’s estate. Five years after the skier’s estate passed to the skier’s daughter, the snowboarder was killed by a falling gondola. The snowboarder’s children, a brother and a sister, inherited the resort as joint tenants. That same year, the skier’s daughter sought to eject the brother and the sister from that part of their resort that was on the daughter’s land. The statutory period for adverse possession is 20 years.
Will the daughter prevail in her suit against the brother and the sister?

(A) Yes, because the statute period was tolled during the incompetency of the skier.

(B) Yes, because the brother and the sister have not been in possession for 20 years.

(C) No, because the brother and the sister possess color of title to the daughter’s entire parcel.

(D) No, because the brother and the sister’s period of possession will be tacked onto the snowboarder’s period of possession.

A

(A) Yes, because the statute period was tolled during the incompetency of the skier.

How well did you know this?
1
Not at all
2
3
4
5
Perfectly
80
Q

A landlord owned a three-story building on the outskirts of the city. Having decided to retire, the landlord determined to sell the building to the tenants, who had each occupied the building for more than 20 years, as a favor to them. A day-care operator operated a day-care service on the first two floors. The child-care facility was on the first floor and the day-care operator’s business operation was on the second. A photographer operated a photographic supply warehouse on the top floor. The top two floors were served by a stairway attached to the building on its exterior. The landlord deeded the first two floors to the day-care operator, and the third floor to the photographer via a simple, hand-drawn deed. No lawyers were involved, but the deed was sufficient to satisfy the statute of frauds applicable in the jurisdiction.

Several years after the conveyance, an earthquake caused the stairs running from the second to the third floor to collapse. When the photographer hired contractors to repair the stairs, the day-care operator objected, saying that he had no obligation to allow the workers on his premises to repair the stairs, and in fact, he feared that the asbestos that the workers would have to release in repairing the stairs would prevent him from operating his business. The statute of limitations on trespass actions in the jurisdiction is 15 years.

If the photographer brings suit to force the day-care operator to allow the photographer’s workers access to reconstruct the stairs, how is the court most likely to rule?

The photographer has a prescriptive easement to use (and thus to rebuild) the stairs by virtue of his long-standing use of the stairs for access to the third floor.

The photographer has an implied right to rebuild the stairs.

Any rights the photographer may have had were terminated by the destruction of the stairway.

The photographer has no right to rebuild the stairs unless the deed gives him that right.

A

The correct answer is: The photographer has an implied right to rebuild the stairs.

Discussion of correct answer: The building was held by a common grantor, who deeded part of it to one grantee and part of it to another grantee. An easement by necessity for the photographer to use the stairway can be implied, even if the deed does not specifically mention it, because that use of the stairway is necessary for the photographer to have access to his property, the top floor. This should be analyzed as analogous to the photographer’s purchase of a landlocked piece of land.

How well did you know this?
1
Not at all
2
3
4
5
Perfectly
81
Q

An elderly woman who needed to move into assisted housing agreed to sell her home to her neighbor for a below market price. Because the neighbor was notorious for taking poor care of his own property, a coalition of local residents came together in opposition to the deal. The coalition members offered the woman an extra $10,000 for the home, and the woman sold it to them on the spot. They paid her immediately, but they did not record the deed right away. With the elderly woman’s permission, they started planting flowers in the yard and they painted the outside of the house. When the time came for the elderly woman to move, the neighbor visited, told the woman he was disturbed by rumors he had heard that she had sold the property to the neighborhood coalition, and reminded her of their deal. Feeling guilty, the elderly woman lied and said that the deal with the coalition had fallen through, though the elderly woman wasn’t a very convincing liar. The neighbor knew that the coalition had planted flowers and painted the house. However, because he wanted the house, the neighbor gave the woman a cashier’s check in exchange for the deed, which the neighbor recorded immediately. The following day the coalition recorded its deed. Assume this transaction took place in a race-notice jurisdiction.

Who owns the property and why?

A The coalition, because it purchased first.

B The coalition, because it purchased first and the neighbor had notice of its deed.

C The neighbor, because he recorded first.

D The neighbor, because he is a bona fide purchaser.

A

B The coalition, because it purchased first and the neighbor had notice of its deed.

How well did you know this?
1
Not at all
2
3
4
5
Perfectly
82
Q

A debtor borrowed $50,000 from a bank and issued to bank a mortgage on property owned by the debtor. The mortgage instrument stated in part: “In the event the debtor defaults on the debt and the bank forecloses, the debtor agrees that her right to redeem shall last for a maximum of six weeks following a declaration of default by the bank.” A year later the debtor defaulted on the debt and the bank declared a default. A week later the bank initiated foreclosure proceedings. Six weeks later, before a foreclosure sale had taken place, the debtor tendered the full amount due on the debt. The bank refused to accept the tender and proceeded with the foreclosure.

Was the bank’s action proper?

Yes, because the debtor waived her right to redeem in the mortgage instrument.

Yes, because the bank had already initiated foreclosure proceedings.

No, because the debtor properly redeemed.

No, because the bank lost its right to foreclose on the property by attempting to “clog” the right of redemption.

A

The correct answer is: No, because the debtor properly redeemed.

Discussion of correct answer: A mortgagee is not allowed to “clog” the mortgagor’s equitable right of redemption. Any such statement in a mortgage document that attempts to do so will be unenforceable. The debtor’s equitable right of redemption exists until a foreclosure sale has taken place.

How well did you know this?
1
Not at all
2
3
4
5
Perfectly
83
Q

A tenant entered into a one-year lease with a landlord. The terms of the lease required the tenant to repair and maintain the premises during the lease, with the exception of ordinary wear and tear. After six months, the tenant complained to the landlord that the hot water was not hot. When the landlord failed to inspect the problem, the tenant again requested that the landlord repair the water heater. In the eighth month of the lease, the tenant stopped paying rent, but did not vacate the property. After the landlord repaired the water heater in the tenth month, the landlord demanded that the tenant pay the unpaid rent. When the tenant refused, the landlord evicted the tenant and sued to recover the unpaid rent.

Is the tenant liable to the landlord for the unpaid rent?

(A) No, because the landlord breached the covenant of quiet enjoyment by not repairing the water heater.

(B) No, because the landlord evicted the tenant before the end of the lease.

(C) Yes, because the landlord had no duty to repair the water heater without a specific provision in the lease.

(D) Yes, because the tenant had a duty to pay rent while in possession of the premises.

A

(D) Yes, because the tenant had a duty to pay rent while in possession of the premises.

How well did you know this?
1
Not at all
2
3
4
5
Perfectly
84
Q

After a gallery purchased one of her original paintings for an enormous sum of money, an artist decided to sell her studio, which was located in a rather dangerous part of town, and move to a nicer area. As a result of the studio’s less than desirable location, hardly anyone expressed any interest in purchasing the studio. Finally, a young newscaster saw an ad for the studio and came to view the property. The newscaster liked the studio very much, and thought that living in a “high-risk” area might give him access to breaking news stories. Sensing the artist’s eagerness to sell the studio, the newscaster offered to purchase the studio for $120,000, a price $50,000 below market value. The artist accepted the offer, and the newscaster gave the artist a check for $12,000 as a down payment on the studio, and the artist gave the newscaster the deed to the property. The parties agreed that the newscaster would deliver the remainder of the purchase price the following day.

However, the artist’s next-door neighbor overhead the communication between the artist and the newscaster. Less than an hour after the newscaster had left, the neighbor approached the artist with an offer to purchase the studio for the full market value of $170,000. Delighted at the opportunity, the artist immediately accepted the neighbor’s offer. That same night, the neighbor gave the artist a cashier’s check for the full amount of the purchase price, and the artist conveyed the deed to the neighbor. The neighbor did not immediately record the deed. The next day, the newscaster returned to the studio to give the artist a check for the remainder of the purchase price. Too embarrassed to admit that she had sold the studio to someone else, the artist accepted the newscaster’s check. Shortly thereafter, the newscaster recorded his deed to the studio. One week later, the neighbor recorded his deed. The jurisdiction in which the studio was located was a “pure notice” jurisdiction.

Under the jurisdiction’s “pure notice” recording statute, who owns the studio?

A. The newscaster, because he recorded his deed first.

B. The newscaster, because the neighbor had notice of the conveyance to the newscaster.

C. The neighbor, because he took subsequent in time to the newscaster.

D. The neighbor, because he was a bona fide purchaser.

A

B. The newscaster, because the neighbor had notice of the conveyance to the newscaster.

B. The newscaster, because the neighbor had notice of the conveyance to the newscaster.

Discussion of correct answer: Under a “pure notice” recording statute, an unrecorded conveyance or other instrument is invalid as against a subsequent bona fide purchase for value and without notice.

To prevail under a notice statute, a claimant of real property must prove the following three elements:

1) that the claimant took subsequent in time to another person claiming ownership of the real property in question;
2) that the claimant was a bona fide purchaser for value; and
3) that the claimant took the property without actual, constructive, or inquiry notice of a prior claim to the property.

In this case, the neighbor’s purchase of the studio did occur subsequent in time to the transaction between the artist and the newscaster, and the neighbor paid value for the studio. However, given that the neighbor overhead the transaction between the artist and newscaster and knew that the artist had delivered to the newscaster a deed to the studio, the neighbor had actual notice of the newscaster’s prior claim to the property, and will not be protected under the pure notice recording statute. Therefore, the newscaster is the legal owner of the studio.

How well did you know this?
1
Not at all
2
3
4
5
Perfectly
85
Q

A seller entered into a contract to sell Redwood to a buyer for $250,000. On the date agreed upon, the buyer paid the $250,000, and the seller executed a quitclaim deed to the buyer. Two weeks later, the buyer learned that the seller had acquired title to Redwood by adverse possession, and had never pursued an action to perfect title. The buyer brought an action against the seller for damages.

Who should prevail?

The buyer, because the seller failed to convey marketable title.

The buyer, because the seller breached the land-sale contract.

The seller, because he conveyed a quitclaim deed to the buyer.

The seller, because title obtained by adverse possession is marketable title.

A

The correct answer is: The seller, because he conveyed a quitclaim deed to the buyer.

Discussion of correct answer: Unless expressly stated to the contrary, every contract for the sale of real property contains an implied covenant requiring the seller to convey marketable title to the buyer. Marketable title generally means title that is free of all reasonable risk of attack. However, when the deal closes the contractual covenant to convey marketable title is said to “merge” into the deed, and it can no longer be the basis for legal action. Any action by the buyer against the seller after closing would have to be based on covenants of title contained in the deed. Here, the seller conveyed Redwood to the buyer by a quitclaim deed, which contains no covenants of title. Therefore, the buyer would not be able to recover against the seller based upon any failure by him to convey marketable title.

How well did you know this?
1
Not at all
2
3
4
5
Perfectly
86
Q

A father had one daughter. He gave her a signed paper that stated: “I hereby give my daughter my house for the rest of her life as long as she remains unmarried.” The daughter immediately moved into the house and began paying the appropriate property taxes. Later that year, a teenager lost control of his car and crashed into the garage. At that point, the father discovered that the daughter had not been insuring the house. The daughter did not repair the garage but instead put her old jalopy on the front lawn, on cinder blocks, waiting until she had time to repair it. She let weeds grow up around it, and also put several large hunks of rusty metal on the front lawn, which she was planning to make into a sculpture. A year later, the daughter’s boyfriend moved in and put his broken car up on blocks in the front yard, next to the daughter’s nonworking car. They got married the following year.

At what point could the father terminate the daughter’s estate?

A When the garage was damaged, because the damage to the garage impaired the value of the property.

B When the father learned that the daughter had not insured the house.

C When the daughter failed to repair the garage and left debris on the front lawn, because it impaired the value of the property.

D When the daughter married, because that is what the deed stated.

A

The correct answer is: When the daughter married, because that is what the deed stated.

Discussion of correct answer: The father created a determinable life estate for his daughter, to be measured by his daughter’s life, and conditioned upon her remaining unmarried. Her estate will automatically terminate and revert back to the father upon the daughter’s marriage or her death, whichever comes first. While the daughter did commit waste, the father would be limited to monetary damages unless a statute granted him the right to seek forfeiture through a judicial action. Because no such statute was mentioned in the facts, this answer is correct.

How well did you know this?
1
Not at all
2
3
4
5
Perfectly
87
Q

A truck driver worked a 40-hour week driving a semi-truck that transported crushed automobiles. During the month of November, he noticed a flyer advertising moonlighting positions available in the area on the bulletin board in the break room at work. The truck driver’s employer allowed such flyers to be posted as a courtesy to those employees who might need to earn extra money for the holidays. The driver called the number on the flyer and was hired by a farmer as an independent contractor. The truck driver was to collect Christmas trees from the farm and deliver them to several outlets that sold cut trees. After a few weeks of moonlighting, the truck driver was exhausted from driving 14 to 18 hours every day. While delivering trees, he missed a turn and slammed on his brakes. The sudden stop caused some of the trees to fall from the truck, and one of the trees knocked a pedestrian down and hit her in the head, severely injuring her.

If the pedestrian sues the truck driver’s employer and the farmer, from whom is the pedestrian entitled to recover?

A. Both the truck driver’s employer and the farmer.

B. The truck driver’s employer only.

C. The farmer only.

D. Neither the truck driver’s employer nor the farmer.

A

D. Neither the truck driver’s employer nor the farmer.

How well did you know this?
1
Not at all
2
3
4
5
Perfectly
88
Q

An electrician lived with his son and daughter in a house on several acres. When he died, he left his home to his son and daughter as tenants in common. Without any discussion or agreement about how to proceed with respect to ownership of the property, the daughter continued to live there, making all tax and insurance payments, paying for repairs and maintenance, and erecting valuable improvements such as a detached garage, guesthouse, and swimming pool. Upon his father’s death, the son went away to college and then worked in a nearby city as an accountant. The jurisdiction in which the property is located operates under common law, and there is no applicable legislation to the situation. Ten years after their father’s death, the son sought partition of the tenancy in common he shared with his sister via sale of the house and land. In addition to half of any proceeds from the sale, the daughter demanded from her brother an additional portion of the proceeds of sale to compensate her for the improvements she made and for the funds she expended in the upkeep and maintenance of the property.

How should the court rule?

The proceeds should be divided equally, since the daughter voluntarily contributed to the property in terms of maintenance and improvements and is, therefore, deemed to have made a gift to her co-tenant.

The proceeds should be divided equally, since a co-tenant out of possession has no duty to contribute to improvements or maintenance on the property.

The daughter should be given an additional amount of the proceeds attributable to the improvements she made, but only to the extent that the amount she paid exceeds the benefit she derived from use of the property.

The daughter should be given an additional amount of the proceeds attributable to the maintenance and upkeep of the property as well as the improvements she made if they generated increased profits.

A

The correct answer is: The daughter should be given an additional amount of the proceeds attributable to the maintenance and upkeep of the property as well as the improvements she made if they generated increased profits.

Discussion of correct answer: At common law, a co-tenant cannot seek contribution from another co-tenant for repairs performed on the property; however, in a partition suit, the costs of repairs may be credited in favor of the co-tenant who made repairs. Moreover, a co-tenant generally has no right to seek contribution or set-off for improvements made unless the improvements generate rents or profits, in which case the costs of improvements are recoverable via partition action. Finally, a co-tenant who pays a mortgage or taxes may seek contribution through partition; however, a co-tenant in sole possession will receive reimbursement only to the extent that his payment exceeds the market value of the property.

How well did you know this?
1
Not at all
2
3
4
5
Perfectly
89
Q

A man signed a mortgage to purchase a piece of property owned by a bank. The man agreed to make monthly payments for a period of 15 years according to the mortgage agreement. The man was in the process of obtaining the necessary permits to build a restaurant on the land, so he had not yet begun construction. Several months later, the bank conveyed the property to a developer who planned to put a hotel on the property. The developer paid cash to the bank for the property. One week later, the man recorded his interest subject to the mortgage. Two days after that, the hotel developer recorded his conveyance. Assume that this jurisdiction has a race-type recording act. Which party has priority?

(A) The developer, because he had an outright conveyance of the property.

(B) The developer, because the conveyance need not be recorded to be enforceable.

(C) The man, because he recorded his interest first.

(D) The man, because an interest subject to a mortgage does not need to be recorded in a race jurisdiction.

A

(C) The man, because he recorded his interest first.

How well did you know this?
1
Not at all
2
3
4
5
Perfectly
90
Q

A man invited several friends over to his house. While they were inside, freezing rain fell outside, coating everything with a thin layer of ice. As they left, one of the man’s guests slipped on the ice on the front stoop of the man’s house.

Which of the following best describes the duty of care owed by the man to his guest?

A. No duty of care.

B. A duty to inspect the premises for unknown dangers and disclose their existence to others.

C. A duty to warn of any known dangerous condition on the premises.

D. An absolute duty of care.

A

C. A duty to warn of any known dangerous condition on the premises.

How well did you know this?
1
Not at all
2
3
4
5
Perfectly
91
Q

A man owned a granite quarry that had a worldwide reputation for the quality of its stone. He had no offspring to take over when he died. Wanting to ensure the continuation of the quarry as a family business, he asked his nephew to take over. The nephew agreed. The man agreed to sell him the quarry for a nominal amount provided that the nephew agreed to language in the deed stating: “To my nephew and his heirs and assigns, but if my nephew dies without issue, then to his cousin and her heirs and assigns.” At the time that the man conveyed the quarry to his nephew, the nephew was single and had no children. Soon after the nephew took over the quarry, a spring was discovered that produced high-quality water. The nephew ordered the immediate cessation of all quarrying of granite in the area of the source of the spring. The portion of the quarry he closed down was the richest area of production. The nephew began to bottle water to sell in local and regional markets. By the time his cousin learned of the spring water operation on the property, granite production at the quarry had decreased by 30%.

If the cousin seeks to enjoin the nephew from continuing the water extraction and to obtain a judicial accounting, which of the following is the most likely reason that a court might find in favor of the nephew?

The nephew can extract spring water from the property because he holds a fee simple defeasible in the quarry.

The nephew’s extraction of spring water from the property does not constitute bad faith use of the quarry.

The nephew can extract spring water from the property because he has a possessory interest in the quarry.

The cousin cannot prevent the nephew from extracting water from the property because she has no interest in the quarry.

A

The correct answer is: The nephew can extract spring water from the property because he holds a fee simple defeasible in the quarry.

Discussion of correct answer: The nephew holds a fee simple subject to an executory interest (one of the three kinds of defeasible fees) and may use the estate in the same manner as a person who holds a fee simple (that is, subject only to government power to impose land use restrictions or to bring a condemnation action). The language “but if the nephew dies without issue, then to his cousin and her heirs and assigns” is conditional language that limits the duration of the nephew’s present fee simple estate such that it will automatically terminate if he dies without issue and title to the quarry will pass to the cousin. As owner of the present fee simple estate, the nephew can remove spring water from the quarry without consulting his cousin, who holds a future interest in the property.

How well did you know this?
1
Not at all
2
3
4
5
Perfectly
92
Q

A father conveyed a home “to my son and my daughter as co-tenants.” The daughter lived with her husband in another state and did not want to possess the home. The son, who had just graduated from college, moved into the home. Two years later, the daughter asked the son to move out and to find a renter for the home. Another year passed, and the son was still living in the home. The daughter was upset that she was not seeing any income from the property.

Is the daughter entitled to collect rent from the son?

(A) No, because the daughter lives in another state.

(B) No, because the son has not wrongfully ousted the daughter from the property.

(C) Yes, because the son and the daughter are tenants in common.

(D) Yes, because the son is in possession of the home.

A

(B) No, because the son has not wrongfully ousted the daughter from the property.

How well did you know this?
1
Not at all
2
3
4
5
Perfectly
93
Q

A landlord owns several large apartment buildings in a college town, and many of his tenants are students at one of the private colleges or the public university in town. In a period when renting apartments in one particular building had become more difficult due to a high vacancy rate throughout the town, the landlord rented a two-bedroom apartment to a law student pursuant to a two-year lease. The lease contained language prohibiting any assignment “absent the express consent of the lessor.” Ten months later, the law student decided to move to another apartment complex to live with friends, and transferred the remainder of her lease to a friend. The landlord soon learned of the transfer and sued to enforce the assignment prohibition against the friend.

What is the likely outcome of the suit?

(A) The friend will prevail, because the prohibition unreasonably restrains the law student from alienating her property interest.

(B) The friend will prevail, because the landlord’s acceptance of the friend as a tenant fulfills his duty to mitigate.

(C) The landlord will prevail, because the prohibition is enforceable against the friend.

(D) The landlord will prevail, because the law student has surrendered the premises

A

(C) The landlord will prevail, because the prohibition is enforceable against the friend.

How well did you know this?
1
Not at all
2
3
4
5
Perfectly
94
Q

A driver and passenger were driving to work in the driver’s car. As they were driving, the passenger suddenly pointed to an overturned vehicle along the side of the highway. As the driver turned to look at the overturned vehicle, he failed to see an abandoned vehicle about 200 feet in front of him. Seconds later, the driver crashed into the abandoned auto and was injured. The jurisdiction has a comparative negligence statute. The driver asserted a claim against the owner of the abandoned vehicle.

What is the most likely result?

A. The driver will recover all of his damages because the defendant created a dangerous condition.

B. The driver will recover only a portion of damages because the abandoned vehicle was in plain view.

C. The driver will recover nothing because he had the last clear chance to avoid the collision.

(D) The driver will recover nothing because the passenger’s act was a supervening cause.

A

B. The driver will recover only a portion of damages because the abandoned vehicle was in plain view.

How well did you know this?
1
Not at all
2
3
4
5
Perfectly
95
Q

A law student went to a coffee shop to study. Just as the law student was entering the coffee shop, an art student stopped her saying, “This is a place of beauty. We don’t want law students here.” The law student was angered by this remark and said, “Get out of my way.” The art student said, “No. Make me.” The law student took out her full-sized umbrella, intending to frighten the art student, and swung it near her head. However, the umbrella broke and a large piece flew off, striking the art student.

If the art student sues the law student for battery, will she prevail?

(A) No, because the art student provoked the law student with her offensive comments.
(B) No, because the law student did not intend for the umbrella to come in contact with the art student.
(C) Yes, because she was struck by the umbrella.
(D) Yes, because the law student failed to conduct herself as a reasonably prudent person.

A

(C) Yes, because she was struck by the umbrella.

How well did you know this?
1
Not at all
2
3
4
5
Perfectly
96
Q

A rancher granted Greenacre to his son-in-law, the son-in-law’s heirs and assigns, provided that the son-in-law is survived by issue of himself and the rancher’s daughter, but if the son-in-law dies without issue of himself and the rancher’s daughter, then to the rancher’s nephew. The son-in-law has discovered that the mountains on the eastern edge of Greenacre are rich in copper.

May he properly mine copper from Greenacre?

No, because, under the open mines doctrine, the son-in-law is not permitted to open a new mine after he takes possession of Greenacre.

No, because mining at Greenacre would invade the nephew’s future interest in the property.

Yes, because the son-in-law holds a fee simple determinable, so he may use Greenacre as he pleases.

Yes, because the son-in-law holds a fee simple subject to an executory interest, so he may use Greenacre as he pleases.

A

The correct answer is: Yes, because the son-in-law holds a fee simple subject to an executory interest, so he may use Greenacre

as he pleases.

Discussion of correct answer: The son-in-law was granted a fee simple subject to an executory interest in Greenacre. The “heirs and assigns” language creates a fee interest, and the words of condition that require that the son-in-law die survived by children of himself and the rancher’s daughter establish that the fee is defeasible; upon the occurrence of the specified condition (no children of the son-in-law and the rancher’s daughter survive him), the fee interest in Greenacre passes to the rancher’s nephew. The owner of a fee interest is not limited (except as specified in the grant of the fee) in the manner in which he may use the property. Holders of executory interests have no right to complain of “waste,” as do remaindermen who follow a life estate. Therefore, the son-in-law may open a mine on Greenacre without regard for the nephew’s interest.

How well did you know this?
1
Not at all
2
3
4
5
Perfectly
97
Q

Several farmers in a drought-ridden state jointly decided to sell their land to a developer, negotiating a good price and extracting a promise from the developer to build only upscale ranch-style homes on lots no smaller than ten acres each. After the sale was consummated, the developer proceeded to develop and sell large homes on the lots, including in each deed a restrictive covenant by which the grantee promised not to subdivide his or her parcel. Before the developer had completed constructing homes on all of the lots, a son offered to purchase the last five ten-acre lots unimproved. The developer sold the last lots to the son subject to the same restrictive covenant as was contained in the other deeds. The son immediately resold the lots to his father’s construction company. His deed did not contain the restrictive covenant. The construction company then sought and obtained a zoning change and construction permit for the development of 200 condominiums on small lots to be subdivided from the original five lots. A homeowner, the first purchaser of a home from the developer, brought an action against the construction company to prevent the subdivision as a violation of the restrictive covenant.

What will be the likely outcome?

Judgment for the homeowner, because the county could not alter the zoning classification in violation of the vested property rights of the previous purchasers of the developer’s lots.

Judgment for the homeowner, because the restrictive covenant is enforceable against the construction company.

Judgment for the construction company, because enforcement of the restrictive covenant would constitute an unreasonable restraint on alienation.

Judgment for the construction company, because the county may rezone regardless of vested property rights.

A

The correct answer is: Judgment for the homeowner, because the restrictive covenant is enforceable against the construction company.

Discussion of correct answer: This is the best answer because the restrictive covenants appear to meet the requirements for enforceability and running with the land: they touch and concern the land, there was intent and notice that the restrictions would run with the land, and there was horizontal and vertical privity. Even if they are not enforceable as covenants, they would be enforceable as implied reciprocal servitudes, because they were part of a common development plan that put the construction company on notice.

How well did you know this?
1
Not at all
2
3
4
5
Perfectly
98
Q

A gym owner entered a five-year lease of commercial property with a landlord. The gym owner drilled holes into the concrete floor and affixed a large pull-up bar system in the middle of the space. Prior to the end of the lease term, the gym owner removed the pull-up bar system from the space, causing damage to the concrete floor.

Is the gym owner liable to the landlord?

A. Yes, because the removal of the pull-up bar system caused damage to the concrete floor.

B. Yes, because the pull-up bar system was a trade fixture that became part of the realty which the gym owner could not remove.

C. No, because the pull-up bar system was a trade fixture which the gym owner was free to remove.

D. No, because the gym owner removed the pull-up bar system before the end of the lease term.

A

A. Yes, because the removal of the pull-up bar system caused damage to the concrete floor.

How well did you know this?
1
Not at all
2
3
4
5
Perfectly
99
Q

A developer bought a 25-acre parcel of land and obtained the required government permits to subdivide the land and construct single-family homes on quarter-acre lots. He sold the first 50 homes and conveyed each with a deed containing an express restriction that the lot be used only for single-family residences. None of the deeds mentioned any covenants regarding restrictions on other property in the subdivision, but the developer orally promised each buyer that the entire subdivision would be restricted to single- family residence use.

The developer then began work on another project and left the job of selling the remaining lots to an agent. The deed to Lot 51 did not contain the restriction on use that had been contained in the earlier deeds. Lot 51’s buyer decided to open a small business selling lottery tickets and videos on his lot. The buyer of Lot 1 filed an action to enjoin the construction and operation of the business.

Who will prevail?

The Lot 51 buyer, because the deeds to Lots 1-50 are outside his chain of title.

The Lot 51 buyer, because the developer’s oral promise to restrict the remaining lots cannot be enforced by the buyers of Lots 1-50.

The Lot 1 buyer, because the Lot 51 buyer effectively had notice of the restrictions in the other deeds.

The Lot 1 buyer, because the developer orally promised that the whole subdivision would be subject to the single- family use restriction.

A

The correct answer is: The Lot 1 buyer, because the Lot 51 buyer effectively had notice of the restrictions in the other deeds.

Discussion of correct answer: A party has inquiry notice of a deed restriction, even if it is not recorded in the deed. The inquiry notice would come from the appearance of the property. Because all of the nearby property was single-family residential, the Lot 51 buyer should have been on notice to inquire whether businesses would be allowed.

How well did you know this?
1
Not at all
2
3
4
5
Perfectly
100
Q

A buyer purchased two lots for $1 million on a long, flat, sandy coastal island that contained many public beaches, as well as homes further back from the beaches. The buyer intended to build two houses, in line with other houses, giving each house an ocean- front view. However, after the purchase of the two lots, the state enacted land-use legislation protecting the area near the beaches from any further development. The buyer sued the state, alleging that his property had been taken for a public purpose without just compensation.

Is the buyer entitled to damages of $1 million?

No, because the state acted within its police power in adopting the zoning scheme.

No, because the buyer can still camp or picnic on his two lots.

Yes, because the buyer has been denied all reasonable, economically beneficial uses of his land.

Yes, because the state can never rezone an area.

A

The correct answer is: Yes, because the buyer has been denied all reasonable, economically beneficial uses of his land.

Discussion of correct answer: The Fifth Amendment provides that private property shall not be taken for public use without just compensation. This prohibition applies to the states through the Due Process Clause of the Fourteenth Amendment. The U.S. Supreme Court held that a property owner is categorically entitled to compensation for a “taking” when the owner has been deprived of all economically beneficial or productive use of the land [Lucas v. South Carolina Coastal Council, 505 U.S. 1003 (1992)]. The courts do not need to weigh the three prongs for a partial taking by a zoning law when the owner has clearly been deprived of all use of his property. Thus, here, the buyer is entitled to the full $1 million from the state.

How well did you know this?
1
Not at all
2
3
4
5
Perfectly
101
Q

A landlord agreed to lease his townhouse to a tenant for a period of one year. The written lease agreement stated that the lease would automatically renew for successive one-year periods unless either the landlord or the tenant gave notice of termination at least one month (and no more than two months) in advance of the end of the period. One year passed after the landlord and tenant executed the lease agreement, and neither gave notice of termination.

After the one-year anniversary of the execution of the lease agreement, what type of tenancy do the landlord and tenant have?
(A) An implied tenancy.

(B) A term of years.

(C) A periodic tenancy.

(D) An at-will tenancy.

A

(C) A periodic tenancy.

How well did you know this?
1
Not at all
2
3
4
5
Perfectly
102
Q

A farmer leased a large country estate for a period of five years. One bright, sunny day, a year after the farmer had moved onto the estate, the farmer’s neighbor was flying a kite in the neighbor’s backyard. The wind picked up and sent the neighbor’s kite flying onto the farmer’s estate, where it became entangled in the limbs of one of the estate’s large trees. The farmer discovered the kite and called the neighbor about retrieving his kite from her tree. However, unable to afford to pay someone to climb the immense tree to retrieve the kite, the neighbor decided to simply abandon the kite and refused to remove it.

Will the farmer prevail in an action against the neighbor for trespass to land?

(A) Yes, because the neighbor’s kite is a tangible physical object that entered the estate.
(B) Yes, because the neighbor failed to remove the kite.
(C) No, because the farmer doesn’t own the estate.
(D) No, because the neighbor abandoned the kite.

A

(B) Yes, because the neighbor failed to remove the kite.

How well did you know this?
1
Not at all
2
3
4
5
Perfectly
103
Q

A man owned a mansion on 100 acres where he raised ostriches and ran a record company. He purchased the property for $5,000,000. He paid the seller a down payment of $1,000,000 and secured a mortgage from a bank for $4,000,000. The interest rate on the mortgage loan was fixed at 7% for a term of 30 years. In order to secure the loan in a timely manner, he signed documents at closing in which he waived his right to redeem the property in the event of foreclosure. He had an attorney represent him at the closing. For four years, income from the record company and selling ostrich eggs to local eateries was sufficient to pay the $27,000 per month mortgage payment. Due to a downturn in the demand for the man’s type of music, and the loss of numerous ostriches from. disease, he defaulted on the mortgage loan. The bank commenced foreclosure proceedings. The bank had the property appraised at $8,000,000. After hearing about the appraisal, the man borrowed money from his music industry friends. The judge granted the bank’s motion for summary judgment and set a date for a sheriff’s sale. The man tendered a check to the bank in the amount sufficient pay off the mortgage and all debt otherwise due on the loan. The bank refused to accept his payment. His attorney filed a motion to dismiss the foreclosure suit.

Should the motion be granted?

(C) No, because he may only redeem the property after the foreclosure sale.

(A) Yes, because he was entitled to redeem the property even after the foreclosure sale.

(B) Yes, because the waiver of the right to redeem was invalid.

(D) No, because he waived the equitable right of redemption.

A

(B) Yes, because the waiver of the right to redeem was invalid.

How well did you know this?
1
Not at all
2
3
4
5
Perfectly
104
Q

Senior Bank held a $100,000 purchase-money mortgage on a cottage. Since the cottage was too small, the owner took a $50,000 mortgage from Junior Bank to build a new media room extension on the cottage. The initial terms of the first mortgage included a 5% interest rate for a term of 15 years. The owner was having trouble making her payments. After she called Senior Bank to discuss the issue, Senior Bank offered to modify her loan. The modification increased the term to 30 years and lowered the monthly payment accordingly. The owner defaulted on the loan. Senior Bank filed a foreclosure complaint against the owner and Junior Bank. Junior Bank filed an answer claiming that its mortgage had priority over Senior Bank’s mortgage to the extent of the material prejudice against it. The jurisdiction followed the lien theory of mortgages.

Did Junior Bank’s mortgage have priority?

(A) No, because Senior Bank’s mortgage was a purchase-money mortgage.

(B) No, because the loan modification did not materially prejudice Junior Bank’s interest.

(D) Yes, because the loan modification materially prejudiced Junior Bank’s interest.

(C) Yes, because Senior Bank modified its mortgage

A

(B) No, because the loan modification did not materially prejudice Junior Bank’s interest.

How well did you know this?
1
Not at all
2
3
4
5
Perfectly
105
Q

An environmentalist conveyed a tract of land “to the people of the state, but if any portion of the tract shall ever be used for any commercial purpose or otherwise converted from its natural state (with exception for recreational, scientific, or educational purposes), then the grantor or his successors in interest may re-enter as of the grantor’s former estate.”

What is the state’s ownership interest in the tract of land?

(A) Fee simple determinable.

(B) Fee simple subject to condition subsequent.

(C) Easement appurtenant.

(D) Determinable fee subject to an executory interest.

A

(B) Fee simple subject to condition subsequent.

How well did you know this?
1
Not at all
2
3
4
5
Perfectly
106
Q

A manufacturer of component parts supplied pedals to a large bicycle producer. One of the pedals ended up being defective but was still placed on a bicycle because the producer was not aware that the pedal was defective. The bicycle was later sold to a father, who bought it for his son. Two weeks after the purchase of the bicycle, the defective pedal broke when the son was riding his bicycle at a local park. As a result, the boy crashed into an elderly man, who suffered serious injuries. The elderly man filed a strict product liability action against the bicycle producer.

Will the elderly man prevail in his lawsuit?

(A) Yes, because the bicycle producer is in the chain of distribution.
(B) Yes, because of the consumer expectation test standard.
(C) No, because the defect is the fault of the manufacturer of component parts.
(D) No, because the elderly man was not using the defective product.

A

(A) Yes, because the bicycle producer is in the chain of distribution.

How well did you know this?
1
Not at all
2
3
4
5
Perfectly
107
Q

A tattoo parlor had a sign in the window that said “Temporary Tattoos Also Available.” The tattoo artist showed a customer samples of her work. The customer picked out a design, and paid to have the tattoo done. The customer thought that he was getting a temporary tattoo, but did not communicate this to the artist. The artist gave the customer the tattoo. After several weeks, the customer concluded that the tattoo was permanent, not temporary. He brought an action against the tattoo artist for battery.

Will the customer’s suit against the tattoo artist for battery be successful?

(A) No, because the customer gave his express consent to the tattoo.
(B) No, because the customer gave his implied consent to the tattoo.
(C) Yes, because the customer’s consent to the tattoo was obtained by fraud.
(D) Yes, because the customer’s consent was based on a mistake.

A

(A) No, because the customer gave his express consent to the tattoo.

How well did you know this?
1
Not at all
2
3
4
5
Perfectly
108
Q

A tenant entered into a two-year lease with a landlord for an apartment. Six months into the lease, the tenant fell asleep while smoking and caused a fire that destroyed the apartment. As a result, the tenant moved out. The next month, the landlord demanded that the tenant pay rent, but the tenant refused,, citing the destruction of the property. The landlord sued to recover the unpaid rent.

Who will prevail?

(A) The tenant, because the property has been destroyed.

(B) The tenant because he has moved out of the property.

(C) The landlord, because the tenant negligently caused the destruction.

(D) The landlord, because the tenant leased only part of the property.

A

(C) The landlord, because the tenant negligently caused the destruction.

Explanation
The correct answer is:The landlord, because the tenant negligently caused the destruction.

Discussion of correct answer:At modern law, when leased property is destroyed, the tenant is relieved from the duty to pay rent, and there is no longer any distinction between a tenant who leases part or all of a building. However, there is an exception to this rule–where the tenant intentionally or negligently causes the destruction, the tenant will not be relieved from liability. Because the tenant here negligently caused the fire that destroyed the apartment, he will not be excused from paying rent.

Discussion of incorrect answers:

Incorrect. The tenant, because the property has been destroyed. At modern law, when leased property is destroyed, the tenant is relieved from the duty to pay rent, and there is no longer any distinction between a tenant who leases part or all of a building. However, there is an exception to this rule–where the tenant intentionally or negligently causes the destruction, the tenant will not be relieved from liability. Because the tenant here negligently caused the fire that destroyed the apartment, he will not be excused from paying rent.

Incorrect. The tenant, because he has moved out of the property. This answer choice appears to be asserting a constructive eviction defense–that the tenant can no longer use the premises and has in fact moved out. However, the destruction here was caused by the tenant’s negligence, and as a result, the tenant will not be relieved from liability for paying rent.

Incorrect. The landlord, because the tenant leased only part of the property. This answer is incorrect because it is asserting the common law rule, not the modern law rule, and even then, is doing so incorrectly. At common law, a tenant who leased all of the premises was not relieved from liability when the property was destroyed because the tenant would still have possession of the land. However, a tenant who leased only a portion of the building was relieved if the property was destroyed. At modern law, however, there is no distinction between leasing all or part of the property; either way, the destruction of the property will relieve the tenant of his obligations. However, there is an exception to this rule where the tenant intentionally or negligently causes the destruction, which is the case here.

How well did you know this?
1
Not at all
2
3
4
5
Perfectly
109
Q

A rancher had a loan with a bank secured by a mortgage on his ranch. The mortgage deed contained a provision at the time of creation waiving the rancher’s right to redeem in the event of foreclosure. The rancher defaulted on the loan, and the bank foreclosed on the ranch. A buyer bought the ranch for $150,000 at a public foreclosure sale. The amount due on the note was $160,000. Two months after the foreclosure sale, the rancher received a large inheritance. A statute in the jurisdiction provides: “A mortgagor shall have the right to redeem the mortgaged property within six months after a foreclosure sale by matching the foreclosure price.“

Can the rancher redeem the ranch from the buyer for $150,000?

(A) No, because the rancher waived the right to redeem in the event of default and foreclosure.

(B) No, because the rancher did not exercise the right to redeem before the foreclosure sale took place.

(C) Yes, because the buyer was on notice of the rancher’s interest in the ranch.

(D) Yes, because the rancher may redeem the ranch from the buyer within six months of the foreclosure sale.

A

(D) Yes, because the rancher may redeem the ranch from the buyer within six months of the foreclosure sale.

How well did you know this?
1
Not at all
2
3
4
5
Perfectly
110
Q

A six-year-old boy often came onto a neighbor’s property to play basketball in the neighbor’s driveway, shooting at the basket attached to the neighbor’s garage. The neighbor often returned home from work to find the boy playing in the driveway. The neighbor decided to dig a well to provide irrigation for his lawn and garden. As the work was being done, the contractor left a large piece of equipment in the yard every evening. One day, after the contractor’s employees had gone home for the evening, the boy came over to play basketball. Curious, he began to climb around the machine, which was able to be started by simply pushing a button; no key was required. The child began to press each button and turn each handle he could reach. Eventually, he pressed the ignition button and the machine began to move. Scared, the child jumped from the still running machine and landed on a pile of dirt, breaking his ankle. The machine continued on its path and eventually struck a support beam in the neighbor’s deck, causing the deck and a section of the roof to collapse. The child’s parents filed suit on his behalf, naming as defendants both the neighbor and the contractor.

In a pure comparative negligence jurisdiction, who will prevail?

A. The neighbor, because he had no duty to maintain or inspect the equipment.

B. The neighbor, because the child was a trespasser.

C. The child, because the neighbor is strictly liable for injuries due to unusually dangerous activities on his property.

D. The child, because it was reasonably foreseeable that he would be on the neighbor’s property.

A

D. The child, because it was reasonably foreseeable that he would be on the neighbor’s property.

How well did you know this?
1
Not at all
2
3
4
5
Perfectly
111
Q

One morning, a small, frail, elderly man was waiting at a crowded bus stop for the city bus to go to the local senior center. When the bus arrived, a shopper, who was carrying a large handbag, got on in front of him. As the man struggled to manage the steep stairs onto the bus, the shopper’s handbag, which was quite heavy, swung back and hit him, throwing him off balance. The man fell back into the street and broke his hip. It was later revealed that the shopper had been carrying a large handgun in her purse, which was what had made the bag so heavy. While the shopper had a permit for the weapon, the jurisdiction in question had enacted an ordinance prohibiting the carrying of concealed weapons. The man files a lawsuit against the shopper to recover his medical expenses on grounds of negligence per se.

Will the man prevail?

A. No, because the harm was not of the type that the statute was designed to prevent.

B. No, because the man’s injuries were unforeseeably extensive.

C. Yes, because the handbag was heavy due to the gun, and the man was within the class of people the statute was designed to prevent.

D. Yes, because the man can prove causation and damages.

A

A. No, because the harm was not of the type that the statute was designed to prevent.

How well did you know this?
1
Not at all
2
3
4
5
Perfectly
112
Q

A jeweler arranged to purchase a strip mall for $1 million and obtained a loan from a local bank for the full purchase price. Over the course of the next few years, several tenants left the strip mall. In an effort to revive the strip mall, the jeweler arranged to convey the strip mall to one of her regular customers, who was the beneficiary of a trust fund and had recently come into a great deal of money. The customer agreed in writing to assume the jeweler’s mortgage. After several months, however, when business at the strip mall still had not improved, the customer stopped making the mortgage payments on the property.

From which party or parties may the bank pursue payment of the mortgage?

A. The jeweler only, because the jeweler did not obtain a novation.

B. The customer only, because the customer agreed to assume the mortgage.

C. The customer only, because the customer took the strip mall subject to the mortgage.

D. The jeweler or the customer, because both parties are personally liable for the mortgage payments.

A

D. The jeweler or the customer, because both parties are personally liable for the mortgage payments.

Discussion of correct answer: Under the facts presented, the jeweler was the original mortgagor of the strip mall, and the bank was the mortgagee. If a mortgagor transfers the property “subject to the mortgage” and mortgage payments are not made, the mortgagee may foreclose and force the property to be sold, but the transferee does not have personal liability for the debt. In contrast, if the transferee of mortgaged real property “assumes the mortgage” and mortgage payments are not made, the mortgagee may foreclose and force the property to be sold, and, in addition, the transferee is personally liable for any deficiency. Finally, in a novation, the transferee of real property and the mortgagee agree that the transferee will assume the mortgage and the mortgagor will be released from liability. In this case, the customer assumed the mortgage, and is therefore personally liable for payment of the amount still owed to the bank. However, given that the bank never agreed to release the jeweler from liability, no novation occurred, and the jeweler likewise remains personally liable for the remaining mortgage payments. As such, the bank may pursue payment from either party, and this answer is correct.

How well did you know this?
1
Not at all
2
3
4
5
Perfectly
113
Q

A colonel had never forgiven his only son for refusing to follow in his footsteps as a career military officer. When the colonel became too ill to enjoy riding the horses he kept on his ranch, he conveyed it to his lifelong friend and fellow military retiree, “for life, remainder to the children of my son.” At the time of this conveyance, the son was married but had no children.

After the conveyance of the ranch, which of the following best describes the interest that the son’s children hold in the ranch?

A contingent remainder.

A shifting executory interest.

A springing executory interest.

They have no interest in the ranch.

A

The correct answer is: A contingent remainder.

Discussion of correct answer: The future interest created in a third party that follows a life estate (or estate for years) created in a grantee is a remainder. Remainders may be vested or contingent. A vested remainder is one that is: (1) created in an ascertained person; and (2) is not subject to a condition precedent. A remainder not possessing these characteristics is contingent. In this problem, the colonel conveyed a life estate in the ranch to his dear friend, followed by a remainder to the unborn children of the colonel’s son. Because those children of the son are unborn (and therefore unascertained), their remainder interest is contingent; as each child (if any) is born, that child’s interest would vest, subject to open (i.e., subject to reduction by the birth of another child to the son).

How well did you know this?
1
Not at all
2
3
4
5
Perfectly
114
Q

A tenant rented an apartment from a landlord. The landlord did not have the tenant enter into a written lease.
Which of the following is not an implied duty of the tenant at the end of his tenancy?

(A) To find a new tenant.

(B) Not to commit waste.

(C) To leave the place in the condition it was rented, reasonable wear and tear excepted.

(D) To leave at the end of the tenancy.

A

(A) To find a new tenant.

How well did you know this?
1
Not at all
2
3
4
5
Perfectly
115
Q

An investor purchased Blackacre from the owner for $250,000, paying $50,000 as a down payment, and issuing a mortgage on Blackacre to secure payment of the remaining $200,000 plus interest over a period of 10 years. The mortgage arrangement called for the investor to pay $2,000 per month, with $800 allocated to interest and the rest to principal. Two years later the investor died, leaving a will devising Blackacre “to my wife for life, remainder to my daughter.”

Between the wife and the daughter, how should the burden for payment be allocated?

(A) The daughter is obligated to pay the interest, and the wife is obligated to pay the principal.

(D) The wife is obligated to pay both the principal and the interest.

(B) The daughter is obligated to pay the principal, and the wife is obligated pay the interest.

(C) The daughter is obligated to pay both the principal and the interest.

A

(B) The daughter is obligated to pay the principal, and the wife is obligated pay the interest.

How well did you know this?
1
Not at all
2
3
4
5
Perfectly
116
Q

An actress had owned Fernwood, a 100-acre tract, for most of her ninety years. Fernwood was located in the town of Boone, which has little land available for development other than the actress’ property. Developers had approached the actress for three decades with offers to purchase Fernwood, but she refused all overtures. When the actress determined that she was facing her final decline, she decided to resolve the issue of the future ownership of Fernwood and began to consider offers. Eventually, when friends asked her who the new owner was, the actress would only reply, “That’s all taken care of.”

The actress died leaving no heirs. Following her death, two parties emerged claiming ownership of Fernwood or a portion thereof. One was a developer planning a subdivision on the tract; the other was a director claiming to own the 10 central acres of the tract. The developer produced a properly executed land sale contract in which the actress agreed to convey Fernwood to the developer for $1.5 million. The director produced a handwritten document signed by the actress in which she agreed to sell to the director 10 acres of Fernwood “chosen and bounded by the buyer” for the price of $1,500 per acre to be paid in cash. The contract the director produced was dated one week prior to the contract produced by the developer. A site visit to Fernwood revealed stakes placed by the director bounding the central 10 acres of the 100-acre tract. The developer’s loss of the 10 acres would severely constrict the planned subdivision roadways and configuration.

If the developer sues to quiet title to Fernwood’s 100 acres, what is the most likely outcome?

The developer will prevail, if he has already started making improvements on the property.

The developer will prevail, because the actress-director land sale contract does not contain an essential term.

The director will prevail, because the actress-director agreement is signed by the party to be charged.

The director will prevail, because the boundary of the 10-acre tract within Fernwood is accurately marked by stakes.

A

The correct answer is: The developer will prevail, because the actress-director land sale contract does not contain an essential term.

Discussion of correct answer: The statute of frauds requires land sale contracts to be in writing, be signed by the party to be charged, and contain the essential terms of the contract, including the parties, a description of the property, and the terms and conditions of price and payment. Even if the actress-director agreement contains the signature of the party to be charged, the contract must comply with all of the other requirements of the statute of frauds. Here, the physical marking of the boundary of a tract purported to be conveyed by a land sale contract does not satisfy the statute of frauds.

How well did you know this?
1
Not at all
2
3
4
5
Perfectly
117
Q

At a family gathering to celebrate the Fourth of July, an uncle saw his favorite niece, who was just about to start college in his hometown. Knowing that she needed a place to stay, he proposed that she rent a room in a house that he owned, located just off campus, for $500 a month until she graduated. The niece was elated to find something so close and reasonably priced. They shook hands on the deal, and the niece moved in on August 1.

One month prior to the niece’s graduation, the uncle’s lawyer sent a letter to the niece demanding that she vacate the premises on the day after her graduation. On graduation day, the niece sent her uncle a check for the next month’s rent. The following day, the uncle brought an action to evict her.

What is the proper outcome?

(A) The niece is entitled to stay, because she and her uncle entered into a periodic tenancy, which was automatically renewed.

(B) The niece is entitled to stay until the uncle provides her with written notice of his intent to rent the apartment to someone else.

(C) The uncle may evict the niece, because she is a tenant at sufferance.

(D) The uncle may evict the niece, because he sent a letter demanding that she vacate the premises.

A

(C) The uncle may evict the niece, because she is a tenant at sufferance.

How well did you know this?
1
Not at all
2
3
4
5
Perfectly
118
Q

A man owned Debtacre. The man borrowed $50,000 from a bank and issued a mortgage on Debtacre to the bank to secure repayment of the debt. The bank immediately recorded the mortgage. Six months later the man borrowed $25,000 from his friend and gave his friend a mortgage on Debtacre to secure repayment of the debt. The friend immediately recorded her mortgage. Two months later the man then borrowed $20,000 from his father and gave his father a mortgage on Debtacre to secure repayment of the debt. The father immediately recorded his mortgage. One month later the man defaulted on the debt to bank and bank foreclosed. Both the friend and the father were notified of the foreclosure proceeding. At the foreclosure sale the father bought the property for $30,000. The $30,000 was paid to the bank. However, the bank was still owed an additional $10,000 on the original debt. The man had made no payments to either the friend or the father, so the full amounts of those debts are still owed.

Which of the following properly describes how title to Debtacre is held?

A. The father owns Debtacre free and clear of any mortgage interest.

B. The bank only has a mortgage on Debtacre for $10,000.

C. The friend only has a mortgage on Debtacre from $25,000.

D. The bank has a mortgage on Debtacre for $10,000 and the friend has a mortgage on Debtacre for $25,000.

A

A. The father owns Debtacre free and clear of any mortgage interest.

How well did you know this?
1
Not at all
2
3
4
5
Perfectly
119
Q

A farmer died owning 10 acres of farmland, part of a once-large tract of farmland that was now almost completely surrounded by housing developments. In the last few years before his death, the farmer used the acres fronting a local road for his home and to run a small farmers’ market, at which he sold produce grown on the “back” acres of his property. The farmer had no other access to these back acres other than via a dirt road that ran from the back acres past his house and out to the public road. The farmer left the property to his daughter.

The daughter sold the front four acres of the land, including the old homestead, to an entrepreneur and the remaining six acres to a construction company for the purpose of building townhouses. Neither deed contained any mention of an easement involving the dirt road. The entrepreneur planned to run a small home-based business on the property, supplying baked goods for her friend’s catering business. The construction company’s land had access to the public road only by means of the dirt road across the entrepreneur’s land. The construction company had no problem moving construction equipment over the dirt road, but now sought to pave the road in order to begin bringing in customers to look at their model homes. The entrepreneur refused to allow the construction company to pave the road because she feared that the noise and traffic would cause her cakes to fall.

If the construction company requests court permission to pave the road, in whose favor should the court rule?

For the construction company, because it has an implied easement and the right to repair and maintain it.

For the construction company, because its pavement of the road is within the reasonable contemplation of the parties.

For the entrepreneur, because the deed to the construction company did not expressly grant an easement across the entrepreneur’s property.

For the entrepreneur, because the owner of the servient estate has the ultimate right to control the maintenance of an easement.

A

The correct answer is:For the construction company, because its pavement of the road is within the reasonable contemplation of the parties.

Discussion of correct answer:The owner of an implied easement has a limited right to upgrade the easement, and generally may not develop or upgrade it beyond the reasonable contemplation of the parties at the time the property was divided. This answer is correct because the daughter sold the land to the construction company knowing that it would be used for townhouses and that the homeowners would need access to the public road. It was thus reasonably within the contemplation of the daughter and the construction company that the road would eventually need to be paved.

How well did you know this?
1
Not at all
2
3
4
5
Perfectly
120
Q

A pre-med student and a biology student lived together in a rental house off-campus. During the semester break, the campus dormitories were closed, and students were not permitted to stay on campus. An education student, who lived on campus, asked the pre-med student if he could spend the semester break at his house. The pre-med student, who was going to visit his family during the break, agreed. The pre-med student gave the education student a key to the house and told him, “My room is yours.” Then the pre-med student left town, leaving a note on the table for the biology student, informing him that the education student would be staying for the break.

That afternoon, the biology student came home after class but did not notice the pre-med student’s note. Late that night, the biology student was awakened by the sound of someone downstairs. He came down to investigate and discovered the education student in the hallway outside of the pre-med student’s room. Before the education student had a chance to explain, the biology student seized him, opened the front door, and pushed him out. The education student fell and fractured his leg.

Will the biology student face liability for forcibly removing the education student from the house?

(A) No, because the biology student can successfully assert self-defense.
(B) No, because the biology student used non-deadly force.
(C) Yes, because the biology student had a duty to retreat.
(D) Yes, because the education student was privileged to be in the house.

A

(A) No, because the biology student can successfully assert self-defense.

How well did you know this?
1
Not at all
2
3
4
5
Perfectly
121
Q

A landscaper entered into a commercial lease with a tailor who planned to operate a tailoring factory on the leased premises. During the term of the lease, the tailor firmly affixed $20,000 worth of state-of-the-art storage racks to the wall of the premises. At the end of the lease term, the tailor vacated the premises without removing the racks, and the landscaper leased the premises to the owner of a taxidermy business. The taxidermy business moved into the premises immediately after the expiration of the tailor’s lease. One day after the expiration of the tailor’s lease, the tailor’s shop foreman returned to the premises to remove the storage racks that the tailor had installed. The taxidermist refused to allow the foreman to remove the racks. The jurisdiction follows the majority rule. The tailor filed suit against the landscaper to recover the storage racks.

Who should prevail?

A The landscaper, because the racks now belong to him.
B The landscaper, because the tailor may not use self-help to remove trade fixtures left over from an expired tenancy.
C The tailor, because his ownership of storage racks was not affected by the termination of the lease.
D The tailor, because the racks are not part of the realty.

A

C The tailor, because his ownership of storage racks was not affected by the termination of the lease.

Discussion of correct answer: A vacating tenant’s ownership of trade fixtures is in fact affected by termination of the lease. If a vacating tenant does not remove trade fixtures from the leased premises within a reasonable time after the termination of the lease, ownership of the trade fixtures passes to the landlord. Here, given that the tailor attempted to remove the shelves one day after the termination date of the his lease, the tailor still owns the racks and may remove them. However, if the tailor had not attempted to remove the racks within a “reasonable time” after the termination of the lease, ownership of the shelves would have passed to the landscaper.

How well did you know this?
1
Not at all
2
3
4
5
Perfectly
122
Q

A businesswoman worked at her company’s headquarters. She was informed by her boss that for the next 6 months her presence would be needed at one of the company’s satellite offices. The businesswoman found an apartment near the satellite office and signed a six-month lease of the apartment to begin January 1 at a rent of $1,000 per month. The businesswoman moved into the apartment on January 1, and paid the rent each month for the six month period. Near the end of June the businesswoman’s boss informed her that her continued presence would be required at the satellite office for an additional month.

On July 1, the businesswoman gave the apartment owner a check for $1,000, telling the apartment owner that she would be staying a while longer. The apartment owner accepted the check. The businesswoman then learned that her presence at the satellite office would not be required after July. On July 31, the businesswoman vacated the apartment and moved back to her home near the company headquarters. The apartment owner brought suit against the businesswoman for rent due.

How much rent, if any, is the apartment owner likely to receive?

(A) Nothing, because the businesswoman effectively terminated the lease when she moved out on July 31.

(B) $1,000 for the month of August.

(C) $5,000 for the months of August through December.

(D) $6,000 for the months of August through January.

A

(B) $1,000 for the month of August.

How well did you know this?
1
Not at all
2
3
4
5
Perfectly
123
Q

A developer owns a large tract of land that she has subdivided into 80 smaller parcels and plans to develop as a residential community. She creates a development scheme for the property and begins to sell lots to buyers. As part of the development scheme, the developer constructs an extensive set of running and bicycling trails throughout the property. In the deed to each lot, the developer inserts a restriction that the lot will not be used for any other purpose other than the construction of a single-family residence. The developer has no land-use plans for the land other than the residential lots and the trail system. The deeds contain a provision guaranteeing lot owners access to the trails. Each deed also contains a clause obligating every lot owner to pay an annual fee to maintain the trails. Finally, the developer inserts a provision in each deed stating that both the benefit of using the system of trails and the burden of the annual maintenance fee shall transfer to any subsequent owner of a lot in the development.

Which of the following options offers the developer the best means of enforcing the requirement that each buyer of a lot pay an annual fee to maintain the trail system?

Express easement.

Easement by implication.

Covenant.

Personal contractual agreement between the developer and each purchaser.

A

The correct answer is: Covenant.

Discussion of correct answer: Covenants are restrictions on the right to use land that run with the land. Here, the requirement that each purchaser of a lot in the development pay an annual fee to maintain the trail system is a restriction that is intended to run with the land and constitutes a covenant. As such, this is the best response.

How well did you know this?
1
Not at all
2
3
4
5
Perfectly
124
Q

A father took out a loan from a lender and, as security for the loan, gave the lender a mortgage on his land. The father and the lender executed a promissory note. The father later conveyed the land to his son subject to the mortgage. When the loan payments to the lender went into default, the lender foreclosed on the land. The proceeds of the foreclosure sale did not cover the amount owed to the lender.

Who is personally liable to the lender for the deficiency?

(A) Both the father and the son.
(B) The father, but not the son.
(C) The son, but not the father.
(D) Neither the father not the son.

A

(B) The father, but not the son.

How well did you know this?
1
Not at all
2
3
4
5
Perfectly
125
Q

A doctor was the first person to purchase a parcel of land in Pine Bluff, a hilly, wooded tract of land in the north woods. The deed conveying the lot contained a reasonable description of the property, but when the doctor placed boundary markers on his property line he mistook one stone outcropping for another and bounded property not his own. The doctor built a house on the outcropping, affording him a spectacular view of the forest below. The doctor lived on the property for seven years before other people began to buy lots in the Pine Bluff tract. He noticed houses being constructed in the valley below, and one day a surveyor appeared at the doctor’s door informing him that his own house was located on a parcel recently purchased by a chiropractor. The doctor immediately brought an action to quiet title, and the court found that he held legal title by adverse possession.

With the increasing development on the Pine Bluff tract, the doctor decided to move to a more remote location and placed his property on the market. The doctor entered a properly executed purchase and sale agreement with a dentist. While preparing for the closing, the attorney for the dentist discovered that a roadway constructed by the doctor on the west side of his property encroached minimally on his neighbor’s land for several hundred yards, then encroached substantially at a point where the roadway swung around the stone outcropping on which the doctor’s house sat. When informed of the encroachment, the doctor was surprised. Because of the topography of the land, it was not possible to move the roadway completely onto the doctor’s land without blasting the stone outcropping. At the closing, the dentist refused to tender performance on grounds that the doctor could not tender marketable title because of the encroachment on his neighbor’s land and because the doctor’s claim of title was based on adverse possession.

If the doctor brings suit for specific performance, what is the most likely outcome?

The dentist will prevail, because the roadway encroachment renders title unmarketable.

The dentist will prevail, because the doctor’s claim of title by adverse possession renders title unmarketable.

The doctor will prevail, because he holds legal title to the property by adverse possession as declared by the court.

The doctor will prevail, because the roadway encumbrance does not destroy marketable title where it does not materially interfere with the use of his land.

A

The correct answer is: The dentist will prevail, because the roadway encroachment renders title unmarketable.

Discussion of correct answer: An encumbrance is a legal interest or liability on property that affects the marketability of title. Some encumbrances, such as easements that benefit the land, do not render title unmarketable; others, such as covenants and easements that restrict the buyer’s use and enjoyment of the land, do render title unmarketable. The roadway encroachment represents a potential lawsuit that the dentist could be forced to resolve if he purchases the doctor’s land. The dentist’s purchase of the land would include “purchasing a lawsuit.” Because the doctor cannot assure him otherwise, title is unmarketable and the dentist need not perform his obligations under the purchase and sale agreement. As a remedy, the dentist can rescind the contract and recover any down payment.

How well did you know this?
1
Not at all
2
3
4
5
Perfectly
126
Q

Brother 1 and Brother 2 own real estate together that is occupied by Brother 1. Brother 1 wanted to obtain a loan to purchase a motor home, but the bank insisted on a mortgage. Brother 2, receiving none of the benefits, refused to sign the note, but did sign the mortgage. Brother 1 defaulted on the mortgage payments.

What liability does Brother 2 after Brother 1 defaulted on the mortgage?

(C) He will be personally liable to the bank if it obtains a judgment in foreclosure.

(B) He will be personally liable to the bank for both a judgment in foreclosure and for a deficiency judgment.

(D) He will be liable to the bank if it obtains a judgment in foreclosure, but will not owe for a deficiency judgment the bank might obtain.

(A) He will owe the bank for any deficiency judgment it obtains.

A

(D) He will be liable to the bank if it obtains a judgment in foreclosure, but will not owe for a deficiency judgment the bank might obtain.

How well did you know this?
1
Not at all
2
3
4
5
Perfectly
127
Q

An aspiring actress owned a farm, but wanted to move to the big city and pursue her dream of acting in movies. The actress executed a written deed granting title of her property to its faithful caretaker, who properly recorded the deed. The state in which the farm is located has a recording act that reads, “A conveyance of an estate in land shall not be valid as against any subsequent purchaser for value, mortgagee, or judgment creditor, except such persons having notice of it, unless the conveyance is properly recorded.”

The aspiring actress did indeed quickly become a successful, though temperamental, star. A few years after arriving in the city, she quit the production of a film for “artistic” reasons. The producers sued her for breach of contract and the court awarded them a $200,000 judgment. The producers promptly recorded a judgment lien against the farm. The producers had no actual notice of the earlier conveyance of the property to the caretaker and did not make a search of the title records. The producers then sought enforcement of the judgment lien.

Will the court enforce the judgment lien against the farm?

Yes, because the recording of the judgment lien revoked the earlier deed of the property to the caretaker.

Yes, because as a donee, the caretaker was not a purchaser for value, mortgagee, or judgment creditor protected by the recording act.

No, because the producers are not purchasers for value.

No, because the caretaker properly recorded the deed to the property.

A

The correct answer is: No, because the caretaker properly recorded the deed to the property.

Discussion of correct answer: This state recording act is designed to protect only those who take property “without notice.” Each transaction must be examined to determine whether the purchaser had actual, inquiry, or constructive notice of any other claims to the same property. The caretaker promptly recorded the deed to the farm prior to the producer’s judgment lien. The law charges a subsequent claimant with constructive notice of all prior conveyances affecting the subject property that have been properly recorded. Therefore, the producers cannot be protected by the recording act; they have constructive notice of the deed to the caretaker.

How well did you know this?
1
Not at all
2
3
4
5
Perfectly
128
Q

A brother and sister inherited their childhood home from their father as tenants in common. The brother lived on the property with his father at the time the father died. The sister lived in a distant city. After their father’s funeral, the brother continued to live on the property. There was no discussion between the siblings concerning their common ownership, nor had there ever been any The brother paid all taxes, insurance, and other carrying charges on the property. He paid no rent or other compensation to his sister, nor did she request any such payment. Thirty years later, a series of disputes arose between the siblings for the first time concerning their respective rights. The jurisdiction recognizes the usual common law types of co-tenancies, and there is no applicable legislation on the subject. The jurisdiction has a 20-year adverse possession statute. The brother now claims fee simple title and brings an action against the sister to quiet title in himself.

For whom should the court rule?

(A) The brother, because he has exercised the type of occupancy ordinarily considered sufficient to satisfy the adverse possession requirements.

(B) The brother, because the acts of the parties indicate that the sister acquiesced to the brother’s right of ownership.

(C) The sister, because there is no evidence that the brother has performed sufficient acts to constitute her ouster.

(D) The sister, because one co-tenant cannot acquire title by adverse possession against another.

A

(C) The sister, because there is no evidence that the brother has performed sufficient acts to constitute her ouster.

How well did you know this?
1
Not at all
2
3
4
5
Perfectly
129
Q

A tenant entered into a two-year lease with a landlord for an apartment. At the start of the lease term, the tenant had several friends help unload her belongings from the moving truck. Later that day, wanting to see what the apartment’s view was like, one of the tenant’s friends attempted to open the sliding glass door that led to the tenant’s balcony. However, the handle appeared to be stuck. When she pulled slightly harder, the handle came off, hitting the friend in the face and causing her to suffer a deep laceration. The previous tenant had notified the landlord of the issue, but apparently, the maintenance crew never fixed the door.

Which of the following arguments would provide the landlord with his best defense?

(A) He warned the tenant of the glass door when she did a walkthrough the week before the tenant signed the lease.

(B) The injury was sustained by a guest of the tenant.

(C) The injury did not occur in a common area of the building.

(D) The maintenance crew, and not the landlord, was the proximate cause of the friend’s injury.

A

(A) He warned the tenant of the glass door when she did a walkthrough the week before the tenant signed the lease.

How well did you know this?
1
Not at all
2
3
4
5
Perfectly
130
Q

A woman leased an apartment on January 1 for a one-year term with rent payable on the first day of the month. Her lease specified that no assignments would be permitted without express written approval from the landlord. Eight months later the woman received an unexpected promotion which resulted in a transfer to a different city. Her younger brother jumped at the chance to take over his sister’s remaining lease. The woman paid rent on the first day of September, and her brother moved in the following day.

On September 29, the landlord came to the apartment. He said that he wanted to inform all of his tenants that the water was scheduled to be turned off the following day for routine repairs. The brother informed the landlord that his sister had moved, and that he had assumed her lease. He gave the landlord a check for October’s rent. The landlord accepted and deposited the check. The following month, the landlord sued the woman for unlawful assignment.

If the woman wins, it will most likely be for what reason?

A The brother paid his rent in a timely fashion.
B There was no mention of a prohibition against subletting in the woman’s lease.
C The landlord waived his right to object to the assignment by accepting and depositing the brother’s check.
D The brother, as a relative of the original tenant, had a legal right to occupy the premises.

A

C The landlord waived his right to object to the assignment by accepting and depositing the brother’s check.

How well did you know this?
1
Not at all
2
3
4
5
Perfectly
131
Q

An accountant leased a single-family home from a landlord for $700 per month and signed a three year lease. Six month later, the accountant received a one year work assignment overseas and leased the home to a baker for one year for $800 per month. Each month, the baker sent $800 to the accountant, $700 of which the accountant sent to the landlord. With three months remaining on the baker’s lease, the accountant stopped sending rent to the landlord.

Who may the landlord sue for unpaid rent?

A. Both the accountant and the baker.

B. The accountant only.

C. The baker only.

D. Neither the accountant nor the baker.

A

B. The accountant only.

How well did you know this?
1
Not at all
2
3
4
5
Perfectly
132
Q

A skier had skied for many years. The skier was proceeding down a relatively steep hill on an advanced course by a series of traverses. He was crossing the slope a number of times to diminish the angle of descent the skiing equivalent of switchbacks in hiking. At the same time a ski instructor, who was also the owner of her own ski school and a member of the ski patrol, came straight down the hill, saw the skier midway across the hill in one of his traverses, and attempted to ski behind the skier. The ski instructor miscalculated the speed at which the skier was traveling, and ran directly into him, fracturing the skier’s knee.

The ski instructor’s conduct will be judged against which of the following standards?

A. The conduct of a reasonably prudent person.

B. The conduct of a reasonably prudent person in an emergency.

C. The conduct of a reasonably prudent person with superior skiing knowledge and expertise.

D. The conduct of a reasonably prudent person with the same age, knowledge, and experience.

A

C. The conduct of a reasonably prudent person with superior skiing knowledge and expertise.

How well did you know this?
1
Not at all
2
3
4
5
Perfectly
133
Q

A football player owned a 100-acre ranch with a house, an Olympic-size swimming pool, a small pool house, and a six-car temperature controlled garage. The player was transferred to another city so he entered into a one-year written lease with a basketball player. The lease agreement provided that the basketball player would be entitled to possession of the land, house, and garage. The lease also contained an anti-assignment clause and a provision explicitly prohibiting subletting without the football player’s express written consent.

Which of the following arrangements would not violate the lease?

A The basketball player allowed his teammate to park an Italian luxury car in the garage for $100 per month.
B The basketball player allowed his cousin to stay at the pool house for two weeks in exchange for a $25 per week.
C The basketball player allowed his friend to come onto the property to remove small amounts of gravel, which he uses in his paving business.
D The basketball player allowed his friends to swim in the pool seven days a week and provided dry towels in exchange for a $10 per day towel fee.

A

D The basketball player allowed his friends to swim in the pool seven days a week and provided dry towels in exchange for a $10 per day towel fee.

How well did you know this?
1
Not at all
2
3
4
5
Perfectly
134
Q

A man owned a small ranch just outside of a city. During the early part of the century, his predecessor-in-interest had granted an express easement to a local mining company to construct, maintain, and operate a railroad line across his property, part of a spur line that ran from coal deposits in the nearby foothills to the city.

Just after the man had acquired the ranch, coal was largely replaced as a source of fuel by natural gas and electricity produced from oil-fired plants, and the mining company stopped mining coal in the foothills and stopped using the rail line. After a few years, the mining company removed the ties, which had become popular building materials, and the rails, which were sold for scrap metal.

About four years ago, the man planted fruit orchards on his property, a portion of which occupied all of the land formerly used by the mining company’s right-of-way. Recently, long and continuous increases in the cost of oil and the collapse of the nuclear power industry made coal mining a profitable venture again. Fifteen years after it last used the rail line, the mining company notified the man that it intended to reconstruct its rail line over the right-of-way granted by his predecessor.

If the man successfully obtains judicial intervention to prevent the mining company from using the right-of-way, what is the best reasoning?

Use of the right-of-way under these circumstances would be a substantial burden upon the servient tenement.

The mining company’s easement terminated when the purpose for which it was granted ceased to be practical.

The mining company’s conduct during the 15 years of non-use evidenced an intent to abandon the easement.

Non-use for a period greater than the statutory period for adverse possession terminates an easement.

A

The correct answer is: The mining company’s conduct during the 15 years of non-use evidenced an intent to abandon the easement.

Discussion of correct answer: Termination of an easement by abandonment requires non-use plus some act that demonstrates the easement holder’s intent to abandon. The removal of the railroad ties and rails is the sort of additional act that evidences an intent to abandon and is the best explanation for why the man would be able to prevent the mining company from using the easement again.

How well did you know this?
1
Not at all
2
3
4
5
Perfectly
135
Q

A buyer contracted with a seller to purchase Lisa’s Glitter Shack, a popular establishment in a high-visibility commercial area. The seller sent the buyer a document via overnight express mail, which stated: “I, Seller, agree to sell Lisa’s Glitter Shack to Buyer for $1,250,000. Check for 10% down received, in escrow. Signed, Seller. Date: April 3, 2004.” The document was also signed by the buyer directly below the seller’s signature. The buyer responded by letter, in which he outlined the time and place of the closing: May 15, 2004, at 2:30 p.m. at the Sword Law Offices located in an office complex around the corner from the Glitter Shack.

On the appointed day, both parties duly appeared to pass title to the property. The seller produced a deed conveying Lisa’s Glitter Shack to the seller from another owner dated May 10, 2004; the buyer learned that the other owner, in fact, owned Lisa’s Glitter Shack on April 3, 2004. The buyer refused to perform the contract, arguing that the seller cannot give good title to the property.

In a suit by the seller for specific performance, what is the likely outcome?

The buyer will prevail, because he did not contract with the other owner to buy the Glitter Shack.

The buyer will prevail, because the seller did not hold marketable title on April 3, 2004.

The seller will prevail, because he fulfilled the terms of the agreement by delivering marketable title on May 15.

The seller will prevail, because the buyer knew that another owner owned the Glitter Shack at the time the parties entered the contract.

A

The correct answer is: The seller will prevail, because he fulfilled the terms of the agreement by delivering marketable title on May 15.

Discussion of correct answer: Even if title is unmarketable at the time of the purchase and sale agreement, the seller does not breach the contract unless he is unable to tender marketable title at the time of the closing. Thus, the seller met the obligations of the seller on May 15 when he tendered marketable title.

136
Q

When twin sisters A and B had their 25th birthday, their mother gave them title to waterfront property. The deed gave them joint ownership with rights of survivorship. The twins immediately moved into the small bungalow on the property.

Five years later, the twins founded a religion that refused to recognize the legitimacy of the government. Their church met every week at the waterfront house. Furious, their mother demanded they return the property. Equally furious, the twins ripped up the deed and threw the shreds at her. The twins and the church vacated the bungalow and moved to Singapore. In order to protect what she felt was now her property, and to prevent the twins from changing their minds, the mother placed a fence around the property. However, the bungalow remained empty, and the land was not used.

Twenty years later, sister A died, leaving a son as her sole heir. The son returned to the United States to write a story about his mother’s church. He went to see the bungalow that featured prominently in all church documents. When he found the property in a state of disrepair, he resolved to purchase the land and house and restore them to their former glory. He went to the county clerk’s office, not because he accepted the authority of the government to keep indices, but to find the owner of the property. The clerk’s records contained only the deed from the mother to the sisters. With his aunt’s permission, he ripped out the fence and began to restore the property. The mother discovered what was happening and asked the sheriff to remove the son. When the sheriff refused, the mother brought suit for a writ of mandamus to force the sheriff to act.

What is the most likely result of the suit?

The writ will be denied, because the mother does not own the property.

The writ will be granted, because the twins abandoned the property, and title, thus, reverted to the mother.

The writ will be granted, because the twins reconveyed title to their mother.

The writ will be granted, because the mother had reacquired title to the land by adverse possession.

A

The correct answer is: The writ will be denied, because the mother does not own the property.

Discussion of correct answer: Once accepted, title to land can only be returned to the grantor by way of a deed delivered to and accepted by the original grantor. In this case, the twins merely destroyed the deed. Regardless of their intent, there is no effect to merely destroying the physical deed. While the physical instrument may be helpful in proving ownership, it is not necessary, under common law, to support a claim of title. Since the twins did not convey title to their mother by way of a signed deed, she has no interest in the property.

137
Q

A landlord entered into a residential lease agreement with a tenant. Five months after the commencement of the lease, the leased premises were destroyed by a fire caused by a lightning strike. The lease agreement did not contain any provision addressing natural disasters or acts of God.

Is the tenant required to rebuild the premises?

(A) No, because under the modern contract theory, neither the landlord nor the tenant is required to rebuild the premises in the event of an act of God.

(B) No, because under the common law estate theory, the tenant is required only to make minor repairs.

(C) Yes, because under the common law estate theory, the tenant has a duty to repair without exception.

(D) Yes, because the lease does not address acts of God.

A

(B) No, because under the common law estate theory, the tenant is required only to make minor repairs.

138
Q

At 10:00 a.m., a landowner sold his oil field to a developer for $800,000 and gave the developer a general warranty deed. The developer immediately sent the deed to his attorney with instructions to record it after their meeting the next morning. The attorney received the deed and the developer’s instructions at 11:00 a.m. At noon, the landowner gave a politician a general warranty deed to the same oil field in return for $1 million. The politician immediately signed the property over to his wife to make up for forgetting their anniversary. The politician’s wife immediately recorded the deed. The property was located in a race-notice jurisdiction.

In a quiet title action by the politician’s wife against the landowner and the developer, how will the court rule?

(A) The developer owns the property.

(B) The politician’s wife owns the property.

(C) The politician owns the property.

(D) The landowner owns the property.

A

(A) The developer owns the property.

139
Q

A telemarketer and a hairstylist owned their house as joint tenants. After a heated argument, the hairstylist decided to leave the telemarketer and moved out of town. On Monday, the hairstylist signed a contract to convey her interest in the house to a personal trainer. On Tuesday, the hairstylist died in a car accident before the transfer of title had been made. The hairstylist’s sister inherited the entirety of the hairstylist’s estate.

What is the state of the title to the house?

(A) The telemarketer owns the house as the surviving joint tenant.

(B) The telemarketer and the hairstylist’s sister own it as tenants in common.

(C) The telemarketer, the hairstylist’s sister, and the personal trainer own it as tenants in common.

(D) The telemarketer and the personal trainer own it as tenants in common.

A

(B) The telemarketer and the hairstylist’s sister own it as tenants in common.

140
Q

A company manufactured dandruff shampoo. The shampoo’s box cover advised consumers to make a “patch test” before applying because a small percentage of the population was allergic to the chemicals contained in the product. A non-English speaking woman purchased the shampoo. Unable to understand the label warning, she used the shampoo without making a “patch test,” and suffered an allergic reaction, causing her to lose her hair. The woman brought suit against the company for strict product liability.

Will the woman prevail?

(A) No, because she assumed the risk by not making the “patch test.”
(B) No, because she did not speak or understand English.
(C) Yes, because she suffered injury from her use of the product.
(D) Yes, because the manufacturer was aware that a small percentage of the population would suffer an allergic reaction to the shampoo.

A

(B) No, because she did not speak or understand English.

141
Q

A husband inherited a beach house on the ocean. The property taxes were so high that the husband could no longer afford to live at the beach house. The husband was divorced three times before he married his wife. The husband had three daughters with his wife and three sons from his prior marriages. The husband conveyed the beach house “to my wife and the female heirs of her body.”

Under the majority rule, which of the following is correct?

(A) The wife has a fee simple absolute.

(B) The wife has a life estate.

(C) The husband’s three daughters have a vested remainder.

(D) All the children have a remainder that vests upon death of the wife.

A

(A) The wife has a fee simple absolute.

142
Q

A builder owned Stoneland, a forested 200-acre parcel containing a rich deposit of marble, and Oceanside, a five-acre parcel with a private beach. The builder decided to divest himself of these properties and made the following conveyances: Stoneland “to my brother, until such time as the land is used for the mining of marble or other stone, then to my sister and her heirs;” and Oceanside “to my nephew, on condition that the property is not used for commercial purposes, but if it is so used, then to my niece and her heirs.” After the conveyance of Stoneland to his brother, the brother did not mine for marble but sold the right to do so to Monument Excavation, which exercised its right. After the conveyance of Oceanside to his nephew, the nephew used the property as a vacation destination for five years, then built and operated an oceanfront restaurant. The builder died leaving his real property to his stepbrother. The builder’s death occurred after his brother had sold Stoneland mining rights to Monument Excavation but before his nephew had constructed the restaurant on Oceanside.

In separate suits to quiet title to Stoneland and Oceanside, what is the likely outcome?

The stepbrother owns Stoneland, and the nephew owns Oceanside.

The stepbrother owns Stoneland and Oceanside.

The brother owns Stoneland, and the stepbrother owns Oceanside.

The brother owns Stoneland, and the nephew owns Oceanside.

A

The correct answer is: The stepbrother owns Stoneland, and the nephew owns Oceanside.

Discussion of correct answer: The language in the deed conveying Stoneland “to my brother, until such time as the land is used for the mining of marble or other stone” created a fee simple subject to an executory interest, where the durational language “until” provided that the present fee simple estate would terminate automatically upon the occurrence of a specified event (the mining of marble or other stone) and title would pass to a third party (“my sister and her heirs”). Executory interests are subject to the rule against perpetuities. In this case, it is possible that more than 21 years would pass before the interest vested in the sister, that is, mining, might not occur within 21 years after some life in being at the creation of the interest. Thus, the executory interest in favor of the sister and her heirs violates the Rule Against Perpetuities and is void. With the executory interest stricken from the conveyance, the remaining language creates a fee simple determinable, that is, a present fee simple estate limited by durational language (“until”) such that the estate terminates automatically if the property is used for mining purposes, at which time the property reverts back to the grantor or his successor. As the builder’s successor, the stepbrother takes Stoneland, because the property was used for mining purposes. As to Oceanside, the language in the deed conveying Oceanside “to my nephew, on condition that the property is not used for commercial purposes” also created a fee simple subject to an executory interest, where the conditional language (“on condition that”) provided that the present fee simple estate would terminate if the property was used for commercial purposes, at which point title would pass to a third party (“my niece and her heirs”). This executory interest, like the one in the Stoneland conveyance, violates the rule against perpetuities. It is possible that Oceanside could be used for more than 21 years after a life in being for purely noncommercial purposes. Voiding the executory interest in the conveyance of Oceanside, the remaining language creates a fee simple absolute in the nephew. The language does not create a fee simple subject to condition subsequent, because while the conveyance contains conditional language (“on condition that”), it does not contain an express statement that the grantor reserves the right of re-entry upon the occurrence of the designated event. A right of reentry cannot be implied. Thus, in a suit to quiet title, a court will find that the stepbrother owns Stoneland and the nephew owns Oceanside.

143
Q

An elderly man conveyed Blackacre to a student for value. The student did not record. Shortly thereafter, the elderly man died, devising all of his real property to his son. The son conveyed Blackacre to a teacher, who paid value and took without notice of the student’s interest in Blackacre. The teacher did not record.

The recording statute in the jurisdiction read: No conveyance of land shall be valid to pass any property interest against purchasers for a valuable consideration but from the time of registration thereof in the city where the land lies.

Who has a superior claim to Blackacre?

A The student, because the student took Blackacre first in time.
B The student, because the student paid value for Blackacre.
C The teacher, because the teacher is a subsequent purchaser for value and without notice of the student’s interest.
D The teacher, because the student did not record.

A

A The student, because the student took Blackacre first in time.

144
Q

Six unrelated college students decided to pool their funds to rent a house to live in while they were attending classes. The house that they rented was in a village that was completely zoned for single-family residences. The definition of a “family” was persons related by blood, adoption or marriage, or two unrelated persons who kept house as a single unit. The owner of the rental house was cited for violating the zoning ordinance. The owner and the students filed suit, alleging that the zoning ordinance violated the Equal Protection Clause of the Fourteenth Amendment.

Is the ordinance constitutional?

No, because the students had a right to live with one another.

No, because a zoning ordinance cannot regulate relationships.

Yes, because the ordinance is necessary to achieve a compelling state interest.

Yes, because the ordinance is rationally related to a legitimate government interest.

A

The correct answer is: Yes, because the ordinance is rationally related to a legitimate government interest.

Discussion of correct answer: Economic and social legislation, including zoning ordinances, will be upheld if the ordinance is reasonable, not arbitrary, and bears a rational relationship to a permissible state objective [Village of Belle Terre v. Boraas, 416 U.S. 1 (1974)]. In Village of Belle Terre, the U.S. Supreme Court held that, because the students were not a suspect classification, and because there was no fundamental right for unrelated people to live together, then the rational basis test applied. The zoning ordinance was rationally related to the village’s interest in providing a quiet atmosphere for families to live in, as opposed to allowing boarding houses and fraternity houses.

145
Q

A restaurant owner signed a mortgage to purchase a piece of property owned by the bank. The restaurant owner agreed to make monthly payments for a period of 15 years according to the mortgage agreement. The owner was in the process of obtaining the necessary permits to build the restaurant on the land, so he had not yet begun to construct the restaurant. Several months later, the bank conveyed the property to a developer who planned to put a hotel on the property. The developer paid cash to the bank for the property. One week later, the restaurant owner recorded his interest subject to the mortgage, and two days later the hotel developer recorded his conveyance. Assume this jurisdiction is a race jurisdiction. Who has priority?

(A) The developer has priority, because he had an outright conveyance of the property.

(B) The developer has priority, because the conveyance need not be recorded to be enforceable.
Correct!

(C) The restaurant owner has priority, because he recorded his interest first.

(D) The restaurant owner has priority, because an interest subject to a mortgage does not need to be recorded in a race jurisdiction.

A

(C) The restaurant owner has priority, because he recorded his interest first.

146
Q

A shopkeeper founded a local bakery that was a neighborhood fixture for over 50 years. The shopkeeper’s son, a baker, took over the management duties from his father in 1995. The baker had no children to take charge of the bakery when the baker died. However, the baker was very close to one of the bakery employees. The baker and the employee, who eventually became the baker’s assistant manager, had a virtual father-son relationship ever since the employee began working at the bakery in 2000. In 2010, the baker said to the employee, “I’m so grateful for the support and loyalty that you have given me these past 10 years. I have come to rely upon your business judgment and baking skills, and I plan to leave the bakery and the land to you when I die. I think it would be in good hands.” The employee continued to work for the baker for 12 more years, gradually taking over more and more of the business, until the baker died. However, the baker apparently died without a will. Under the state intestacy statute, the baker’s property passed to his sister as his only living relative. A dispute over ownership of the bakery arose between the employee and the sister.

Which of the following principles of law will govern the resolution of the dispute with respect to the land?

The parol evidence rule.

A resulting trust.

The Statute of Frauds.

Estoppel theory.

A

The correct answer is: The Statute of Frauds.

Discussion of correct answer: The Statute of Frauds applies to conveyances (transfers) of real property. A conveyance of land must be memorialized in writing signed by the party to be charged. The writing must contain at least: (1) an adequate description of the land; (2) the names of the parties to the contract; and (3) the price set by the parties, but it need not be a formal contract. Part performance is an exception to the Statute of Frauds. Part performance generally requires payment of all or part of the purchase price and significant acts consistent with the existence of a land-sale contract. The facts presented provide no evidence of part performance, and the two parties (the baker and the employee) did not comply with the Statute of Frauds.

147
Q

Two brothers purchased a farm as joint tenants in a state that follows the majority rule for mortgages. Several years later, the younger brother got married. His wife was offered a position in a law firm in New York City. Because the farm had never made a profit, the younger brother moved to New York with his wife. In order to purchase an apartment in the city, the younger brother mortgaged his interest in the farm.

Following the younger brother’s mortgage, what interest does the older brother hold?

(A) A one-half interest as joint tenant.

(B) A one-half interest as a tenant in common with his younger brother.

(C) A one-half interest as a tenant in common with the mortgagee.

(D) A one-half interest as a tenant in the entirety.

A

(A) A one-half interest as joint tenant.

148
Q

A grantor conveyed his property “to my brother for life, then to the heirs of my brother,” by a properly executed deed, for proper consideration. The brother accepted but did not record the deed. Four years later, the grantor purported to convey the same property in fee simple absolute to his two sons, by a warranty deed, as a gift. The two sons recorded their deed. Shortly thereafter, the brother ascertained that the grantor’s sons were about to take possession of the property. As a consequence, the brother promptly recorded his deed. The recording statute in this jurisdiction provided “unless recorded, all written instruments affecting title to land are void as to subsequent purchasers who paid value and without notice.”

If a court finds the brother to be the owner of the property, what is the most likely explanation?

(A) The brother paid valuable consideration for his deed.

(B) The brother recorded his deed before the grantor’s sons sought to oust him from the land.

(C) The grantor’s sons are not protected by the recording statute.

(D) The grantor’s knowledge is imputed to his sons.

A

(C) The grantor’s sons are not protected by the recording statute.

149
Q

A wealthy politician resided in a small town. Although he had supported many charitable causes during his career, the will he drafted in 1995 left all his property, real and personal, to his niece, who was also his only surviving heir. The politician died in 2005. To his niece’s surprise and disappointment, she found that the politician’s largest piece of real estate, an office and urban housing complex, had been deeded in 1979 to a local organization that raised funds to support its campaign to impose limits on the terms of members of Congress. The deed stated that if the local organization was unsuccessful in obtaining passage of a constitutional amendment limiting the terms of members of Congress to four years by the year 2009, then the property would go to a vision clinic and research organization “so long as some portion of the building is used for research into better methods of improving eyesight by surgical means.” If not so used while owned by the vision clinic, the property would go to a senior citizens auxiliary to be used as a retirement home for local indigent senior citizens. Real property conveyances in the town are governed by the common law. Assume that all of the grantee organizations qualify as charitable organizations under the law of the jurisdiction.

What is the best description of the local organization’s interest in the property?

A fee simple subject to a condition subsequent.

A fee simple subject to an executory interest.

A fee simple determinable.

A determinable fee subject to an executory interest.

A

The correct answer is: A fee simple subject to an executory interest.

Discussion of correct answer: A fee simple subject to an executory interest is a present fee simple that is limited in duration either by durational language or by conditional language such that it will terminate automatically upon the occurrence of a specified event (or non-occurrence of a specified event by a specified date), and title passes to a third party instead of back to the grantor. The interest of the local organization is a fee simple subject to an executory interest, because it would automatically terminate upon the non-occurrence of a designated event by a specified date, in this case the failure to pass a term-limit amendment by 2009. The interest was further subject to the ophthalmologist’s executory interest. Note that the Rule Against Perpetuities is not violated because of the charity-to-charity exception to the Rule.

150
Q

An aspiring actress owned a farm, but wanted to move to the big city and pursue her dream of acting in movies. The actress executed a written deed granting title of her property to its faithful caretaker, who properly recorded the deed. The state in which the farm is located has a recording act that reads, “A conveyance of an estate in land shall not be valid as against any subsequent purchaser for value, mortgagee, or judgment creditor, except such persons having notice of it, unless the conveyance is properly recorded.”
The aspiring actress did indeed quickly become a successful, though temperamental, star. A few years after arriving in the city, she quit the production of a film for “artistic” reasons. The producers sued her for breach of contract and the court awarded them a $200,000 judgment. The producers promptly recorded a judgment lien against the farm. The producers had no actual notice of the earlier conveyance of the property to the caretaker and did not make a search of the title records. The producers then sought enforcement of the judgment lien. Will the court enforce the judgment lien against the farm?

(A) Yes, because the recording of the judgment lien revoked the earlier deed of the property to the caretaker.

(B) Yes, because as a donee, the caretaker was not a purchaser for value, mortgagee, or judgment creditor
protected by the recording act.

(C) No, because the producers are not purchasers for value.

(D) No, because the caretaker properly recorded the deed to the property.

A

(D) No, because the caretaker properly recorded the deed to the property.

151
Q

On June 1, an owner contracted to sell his property known as Blackacre to a buyer for a valuable consideration. On June 30 the owner executed and delivered a deed to the buyer. On August 1, the owner executed and delivered to a neighbor a deed to Blackacre, also for valuable consideration. The neighbor, who was aware of the deed delivered to the buyer, promptly recorded the deed. On September 1, the neighbor delivered to his son a deed to Blackacre for valuable consideration. The son was unaware of the deed delivered to the buyer. On September 30, the buyer recorded his deed, and the son recorded his deed the next day.

The jurisdiction in which Blackacre is located has adopted the following statute:

“Any conveyance of an interest in land shall not be valid against any subsequent purchaser for value, without notice, whose conveyance is first recorded.”

Who owns Blackacre?

A. The buyer, because he was the first grantee.

B. The buyer, because the neighbor purchased Blackacre with notice of the buyer’s deed.

C. The son, because he was a subsequent bona fide purchaser.

D. The son, because the neighbor recorded his deed before the buyer.

A

B. The buyer, because the neighbor purchased Blackacre with notice of the buyer’s deed.

152
Q

A builder owned Stoneland, a forested 200-acre parcel containing a rich deposit of marble, and Oceanside, a five-acre parcel with a private beach. The builder decided to divest himself of these properties and made the following conveyances: Stoneland “to my brother, until such time as the land is used for the mining of marble or other stone, then to my sister and her heirs;” and Oceanside “to my nephew, on condition that the property is not used for commercial purposes, but if it is so used, then to my niece and her heirs.” After the conveyance of Stoneland to his brother, the brother did not mine for marble but sold the right to do so to Monument Excavation, which exercised its right. After the conveyance of Oceanside to his nephew, the nephew used the property as a vacation destination for five years, then built and operated an oceanfront restaurant. The builder died leaving his real property to his stepbrother. The builder’s death occurred after his brother had sold Stoneland mining rights to Monument Excavation but before his nephew had constructed the restaurant on Oceanside.

In separate suits to quiet title to Stoneland and Oceanside, what is the likely outcome?

(A) The stepbrother owns Stoneland, and the nephew owns Oceanside.

(B) The stepbrother owns Stoneland and Oceanside.

(C) The brother owns Stoneland, and the stepbrother owns Oceanside.

(D) The brother owns Stoneland, and the nephew owns Oceanside.

A

(A) The stepbrother owns Stoneland, and the nephew owns Oceanside.

153
Q

Ophelia, the record owner of Blackacre, conveyed Blackacre to Adam, who did not record. Ophelia then conveyed Blackacre to Benita, who paid value, recorded immediately, and did not have notice of Adam’s deed. Benita then conveyed Blackacre to Carlos, who took by gift. The jurisdiction has a race-notice recording statute. Who holds title to Blackacre?

(A) Adam, because Carlos took by gift.

(B) Benita, because Adam did not record.

(C) Carlos, because Benita paid value and recorded without notice of the conveyance to Adam.

(A) Adam, because the conveyance to Benita was fraudulent.

A

(C) Carlos, because Benita paid value and recorded without notice of the conveyance to Adam.

154
Q

The former owner of several parcels of land surrounding a small lake had wished to preserve the character of the lake, and therefore inserted into the deeds by which each purchaser took title language creating an equitable restriction prohibiting the construction of any permanent structure thereon or the use for any purpose other than camping, fishing, or related recreation. A hunter took possession of one such parcel, which was owned by a local mailman, occupying and using the property in a manner and for a period sufficient to acquire title by adverse possession. The mailman, unaware of the circumstances and effect of the hunter’s occupation of his property, purported to sell that property for market value to an engineer and executed a warranty deed purporting to convey the parcel to the engineer, who immediately recorded the deed. The recording act in the jurisdiction provides, “Every conveyance of real property (other than a lease for a term not to exceed one year) is void as against any subsequent purchaser or mortgagee of the same property (or any part thereof) in good faith and for a reasonable value consideration, whose conveyance is first duly recorded.” If the engineer brings an appropriate action to eject the hunter and quiet title to the disputed parcel, will she prevail?

(A) No, because the hunter’s possession of the parcel put the engineer on inquiry notice of his interest therein.

(B) No, because the recording act does not operate to void the hunter’s acquisition of title to the parcel.

(C) Yes, because the hunter failed to document his acquisition of title in a manner equivalent to recording a deed to the parcel.

(D) Yes, because intermittent use of land is not sufficient to put a bona fide purchaser on inquiry notice.

A

(B) No, because the recording act does not operate to void the hunter’s acquisition of title to the parcel.

155
Q

A mother enrolled her daughter in ballet lessons at a local studio. One Saturday the daughter was trying out for a part in the upcoming show. The mother drove the daughter to the ballet studio and parked her car in the local supermarket parking lot that adjoined the ballet studio. Because the studio did not have its own parking lot, parents often parked at the supermarket and walked about 100 feet to the ballet studio. The supermarket allowed the parents access to the parking lot with the expectation that they would shop at the supermarket. When the mother and daughter hopped out of their car in the supermarket parking lot, they were immediately accosted by a thief. He grabbed the mother’s pocketbook and pushed her to the pavement.

If the mother asserts a tort action against the supermarket, will she prevail?

A. Yes, because the mother was an invitee.

B. Yes, because the incident happened on the supermarket’s property.

C. No, because the supermarket is not liable for the intentional torts committed by third persons.

D. No, because the mother was a licensee.

A

C. No, because the supermarket is not liable for the intentional torts committed by third persons.

156
Q

A landowner owns 50 acres of open grassland that have over the years become surrounded by residential developments. Over the years, individuals and developers have offered her vast sums of money for the property. In hopes of preserving the grassland after her demise, the landowner conveys the property “to my sister, her heirs, and assigns, while the property is maintained as open space, then to Wildlands Trust.” Wildlands Trust is a nonprofit organization that preserves open space for public use, maintaining a trail system on its properties for hiking.

Which of the following is the most accurate statement regarding the effect of the conveyance of the grasslands from the landowner to her sister?

(A) The sister holds a fee simple determinable.

(B) The sister holds a fee simple subject to condition subsequent and right of re-entry.

(C) The sister holds a fee simple subject to an executory interest in favor of Wildlands Trust.

(D) The sister holds a fee simple absolute.

A

(A) The sister holds a fee simple determinable.

157
Q

A mother and her two daughters were enjoying the summer blockbuster season. As they waited on line at the theater for tickets to the latest film, the daughters began to fidget and complain. By the time they made it to their seats, the daughters had succeeded in greatly irritating their mother. The mother decided to put some space between them and her, and she took a seat in the row behind them. When the movie began, the daughters continued to whine and complain, and the mother could tell that they were beginning to annoy the rest of the audience, as well. She leaned forward and whispered to them to quiet down and enjoy the movie. The mother then left to the bathroom, and while the mother was in the bathroom, the daughters moved seats and two new girls sat in the seats of the daughters. The mother then came back to her seat from the bathroom and once again heard fussing, but this time from the two new girls. Thinking that these girls were her daughters, the mother reached over the seat and swatted each of them on the back. As a result of the summer heat, both girls were sunburned, and the mother’s swat left a visible mark on their bare backs. The girls now sue the mother for battery.

Will the mother face liability?

(A) Yes, because the mother swatted the two girls on the back.
(B) Yes, because the mother’s swat left a visible mark on the girls’ bare backs.
(C) No, because the mother thought she was initiating contact with her daughters.
(D) No, because the mother’s contact with her daughters is privileged.

A

(A) Yes, because the mother swatted the two girls on the back.

158
Q

A property owner owns a vacant warehouse on the edge of downtown that is being renovated. The building had construction equipment, damaged floors with large holes, exposed beams, exposed wiring, and construction debris littered throughout the structure. For over a month, a group of homeless men would routinely enter the building late at night to sleep and eat. The owner periodically checked on his property, and noted food scraps, sleeping blankets, and a small camp stove. Recently, one of the homeless men fell through a hole in the top floor and severely injured himself. The trial court entered summary judgment in favor of the property owner, ruling that he did not owe any duty to the homeless man. The homeless man has filed an appeal.

Should the appellate court affirm the trial court’s ruling?

A Yes, because the homeless man was a trespasser.
B Yes, because the homeless man was a licensee.
C No, because the property owner owed a duty of care to protect the homeless man.
D No, because the homeless man acted out of necessity.

A

C No, because the property owner owed a duty of care to protect the homeless man.

159
Q

A woman executed a mortgage with a bank for the purchase of a home. For the first few years of her 15-year mortgage, she had no trouble making timely payments on the mortgage note. One year, however, she got behind on her mortgage payments and, for five months, made no payments at all. The woman received notice from the bank indicating that she had defaulted on the loan and advising her of its intent to engage in foreclosure proceedings.

Realizing that the home was worth quite a bit more than it was when she purchased it, the woman tried to sell it herself rather than face foreclosure, but was unsuccessful. The bank subsequently foreclosed on the home and the sale price was more than sufficient to cover the outstanding debt. The woman contacted the bank to inquire about any remaining proceeds from the sale, and the bank told her that it had retained the remaining proceeds from the foreclosure sale as rent for the months that she lived in the house after she defaulted on the mortgage.

If the woman is successful in recovering the remaining proceeds, which of the following provides the best reason?

She lives in a jurisdiction that adheres to the intermediate theory of mortgage interests.

She lives in a jurisdiction that adheres to the title theory of mortgage interests.

She lives in a jurisdiction that adheres to the lien theory of mortgage interests.

She lives in a jurisdiction that adheres to the equitable theory of mortgage interests.

A

The correct answer is: She lives in a jurisdiction that adheres to the lien theory of mortgage interests.

Discussion of correct answer: In a lien theory jurisdiction, while the mortgage is being paid off, the mortgagee possesses only a lien on the property, while the mortgagor retains legal, equitable, and possessory interest in the property unless or until foreclosure occurs. If the home in question sits in a lien theory jurisdiction, the bank (as mortgagee) could not seek rent for the property for the period of time prior to the foreclosure, because technically, the bank’s interest in the property during this time was simply that of a lienholder. The lien theory is relied upon in a majority of U.S. jurisdictions.

160
Q

A borrower mortgaged a ranch to a mortgage company. The mortgage company did not record the mortgage for three years. Ten years later, the borrower promised, in a signed writing delivered to his daughter, to convey the tract of land to his daughter by a quitclaim deed and to pay the mortgage debt when it came due. A year later, the borrower delivered to the daughter a quitclaim deed that made no reference to the mortgage. The borrower later defaulted on the mortgage and the mortgage company brought an in personam action against the daughter to recover the amount due on the mortgage debt.

Is the mortgage company likely to succeed?

(A) No, because the daughter did not promise to pay the mortgage debt.

(B) No, because the law does not permit the mortgagor to delegate duties under this type of a property transfer.

(C) Yes, because the mortgage company is a third-party beneficiary of the agreement between the borrower and the daughter.

(D) Yes, because there was an implied delegation of duties to the daughter.

A

(B) No, because the law does not permit the mortgagor to delegate duties under this type of a property transfer.

161
Q

A tenant leased a condo from a landlord, and the parties signed a one year lease. Near the expiration of the lease term, the tenant told the landlord that the tenant might be moving to another state, and the tenant did not want to sign another one year lease. The tenant and the landlord orally agreed that the tenant could continue living in the condo, and the tenant had to give the landlord at least 30 days’ notice prior to moving out. The tenant remained in the condo and continued to send the landlord a rent check each month.

What type of tenancy does the tenant have?

A A tenancy for a term of years.
B A periodic tenancy.
C An at-will tenancy.
D A holdover tenancy.

A

B A periodic tenancy.

162
Q

The owner of a 50-acre parcel of unimproved land conveyed his property to a real estate developer. The deed by which the owner conveyed the property to the developer recited that a 25-foot wide easement for ingress and egress to the property immediately to the west, running along the southern boundary of the land, had previously been granted to his sister. The developer subsequently conveyed the land to a neighbor via a deed of gift that contained covenants of general warranty, quiet enjoyment, and right to convey, but which made no mention of the easement owned by the owner’s sister. No physical evidence on the property suggested the existence of that easement, and the sister had not made use of it since it was created. The neighbor learned of the existence of the right-of-way easement and brought an action against the developer to recover damages for breach of any of the covenants of title contained in the developer-neighbor deed.

What is the probable outcome of this action?

The neighbor will recover damages for breach of the covenant of quiet enjoyment.

The neighbor will recover damages for breach of the covenant of quiet enjoyment, but only if he paid consideration for his deed and if the sister is actually using the easement.

The neighbor will recover damages for breach of the covenant of right to convey.

The neighbor will recover damage for breach of the covenant of right to convey, but only if the sister is actually using the easement.

A

The correct answer is: The neighbor will recover damages for breach of the covenant of quiet enjoyment, but only if he paid consideration for his deed and if the sister is actually using the easement.

Discussion of correct answer: A breach of a deed covenant can only give rise to an action for damages, and recovery is almost always limited to the amount the grantor received for the property. The presence of an encumbrance that the grantee did not agree to take subject to is a breach of the “present” covenant against encumbrances (assertable only by the immediate grantee), and may be a breach of the “future” covenant of quiet enjoyment (also assertable by remote grantees). The covenant of quiet enjoyment, like the other “future” covenants of warranty and further assurances, is breached only when the grantee is actually disturbed by the holder of the encumbrance. Since the question gives a choice of only the covenant of right to convey, which is not breached by an encumbrance, and the covenant of quiet enjoyment, this choice must be the best answer, since it identifies the correct covenant and mentions both the fact that the sister must have begun using her easement, and that the neighbor must have paid consideration for the deed in order for the neighbor to recover damages.

163
Q

A man took out a 15-year mortgage to purchase a home, which the mortgagee promptly recorded. Unfortunately, the housing market declined rapidly after his purchase. After two years, the man sold his house, but the sales price did not cover his outstanding mortgage debt. The purchaser of the house paid in cash to buy it at a large discount. The purchase agreement was silent as to the man’s existing mortgage. One year after the sale of the house, the mortgagee had not received any payments on the outstanding mortgage debt.

What recourse does the mortgagee have?

(C) The mortgagee may sue the man for the balance of the mortgage debt but may not foreclose on the property.

(B) The mortgagee may foreclose on the property and sue the man for the balance of the mortgage debt.

(D) The mortgagee may sue the purchaser for the balance of the mortgage debt but may not foreclose on the property.

(A) The mortgagee may foreclose on the property and sue the purchaser for the balance of the mortgage debt.

A

(B) The mortgagee may foreclose on the property and sue the man for the balance of the mortgage debt.

164
Q

A chiropractor and a gardener are neighbors in a seaside resort town. The chiropractor’s house is on the beach, while the gardener’s house is across the street directly behind the chiropractor’s residence. Both houses are one-story, single-family dwellings. The gardener has a rose garden in front of her house that is highly regarded by townspeople and tourists alike. Several weekly bus tours stop briefly at her house to allow people to snap shots of the gardener’s award-winning roses. A decade after they became neighbors, the chiropractor decides to enlarge his house. The chiropractor hires an architect, who designs a two-story addition anchored by a three-story tower at the north end of the house. The town building code prohibits structures in excess of three stories. The gardener learns of the chiropractor’s planned addition and determines that the shadow cast by the second story and tower will effectively cut the amount of sun on her rose garden to a half-hour a day, not enough to maintain any roses. Assume that the statutory period for adverse possession is seven years.

If the gardener sues for injunctive relief, will she prevail?

Yes, because an easement is implied where light and air are reasonably necessary to the benefited estate.

Yes, because the gardener has obtained a right to light and air by prescription, since she has met the requirements for adverse possession.

No, because no negative appurtenant easement for light and air was conferred on the gardener’s property by the chiropractor or a previous owner of his property.

No, because the chiropractor’s grantor did not confer a negative easement in gross on the gardener’s property before the gardener acquired the property.

A

The correct answer is: No, because no negative appurtenant easement for light and air was conferred on the gardener’s property by the chiropractor or a previous owner of his property.

Discussion of correct answer: If the chiropractor or a previous owner of his property had conferred a negative appurtenant easement on the gardener’s property, it would pass to the new grantee of the dominant estate, and the gardener could enforce the easement against the chiropractor. Express negative easements for light and air are recognized by the courts and can run with the land. However, there is no evidence of such an easement in these facts. The facts also do not show that the chiropractor’s proposed addition violates the town’s building code. Therefore, the court will not grant the gardener’s requested injunctive relief.

165
Q

A man was concerned about his failing health. He called his attorney, who helped him draft and execute a will. The will contained a valid in terrorem clause, which stated that any gift to any person who challenged the will in court would be revoked. Under the terms of the will, he devised his home “to my only son for life.” He also devised “the rest and residue of my estate to my beloved daughter.” Three months later, the man died. His daughter brought suit challenging the validity of the will.

Should the court find for his daughter?

(A) Yes, because she has a reversion in his home.

(B) Yes, because she has a present possessory interest in his home.

(C) No, because the son has a present possessory life estate in the home.

(D) No, because the son has a fee simple absolute in the home.

A

(D) No, because the son has a fee simple absolute in the home.

166
Q

A factory worker purchases a home for $250,000. The factory worker obtains a loan from his local bank for the entire amount. The factory worker signs a loan agreement with the bank that contains provisions stating that he will pay a late fee of $500 if he is late on a mortgage payment and that he waives the right to redeem the property in case of foreclosure proceedings. One year later, the factory worker loses his job and can no longer pay his mortgage payments. After several months without receiving payment, the bank decides to initiate foreclosure proceedings. One month before auction, the factory worker finds another job. He immediately calls the bank to ask for extra time to pay off his missed mortgage payments and the associated late fees. The bank refuses to work with the factory worker, arguing that he previously waived his right to redeem the property.

Is the bank’s assertion correct?

A No, because the right to redeem may not be waived at the time a mortgage agreement is executed.
B No, because a bank can never ask a mortgagor to waive his right to redeem.
C Yes, because the factory worker waived his right to redeem at the time he obtained the mortgage.
D Yes, because the bank can at any time require the factory worker to waive his right to redeem.

A

A No, because the right to redeem may not be waived at the time a mortgage agreement is executed.

167
Q

On August 1, a student entered into an oral agreement with a landlord to lease an apartment for $500 per month, starting immediately and running through the end of the year. At the end of the year, the student failed to move out, instead paying the rent for the following month on time and directly to the landlord, who accepted the rent by depositing it into his bank account.

What is the relationship between the parties as of January 1?

(A) The student is a tenant at sufferance, and the landlord may immediately evict him.

(B) The student’s payment, and the landlord’s acceptance, of the rent formed an at-will tenancy, in which either party can immediately terminate the lease.

(C) Because the original lease agreement was not in writing, the Statute of Frauds will operate to invalidate the agreement, resulting in an at-will tenancy.

(D) A month-to-month periodic tenancy has been formed.

A

(D) A month-to-month periodic tenancy has been formed.

168
Q

A florist rented an apartment to a football player pursuant to a three-year lease. After the football player was traded to another team, he failed to make rent payments for five months. Without the football player’s permission, the florist employed a few weight lifters to move the football player’s property out of the premises and into a locked storage unit.

Under the law of most jurisdictions, did the florist act lawfully?

A. Yes, because the tenant committed a material breach of the lease by failing to pay rent.

B. Yes, because where the Rule in Dumpor’s Case applies, a landlord may employ non-violent self-help methods to evict a breaching tenant.

C. No, because under the modern rule, the proper remedy available for the football player’s breach of the lease is damages.

D. No, because the florist used self-help to evict the football player.

A

D. No, because the florist used self-help to evict the football player.

169
Q

A property owner was a charter member of the Tea-Totalism Society, an organization dedicated to complete abstinence from alcoholic beverages. The owner conveyed his restaurant, the Tea Room, “to Alice and her heirs for so long as liquor is not served on the premises.” The owner also conveyed all of his future interest to Tom, the founder of the Tea-Totalism Society.

Which of the following is correct?

(A) Alice has a fee simple subject to condition subsequent.

(B) The owner has a possibility of reverter.

(C) The owner has an automatic right of re-entry.

(D) Tom has a possibility of reverter, which becomes possessory automatically if liquor is served.

A

(D) Tom has a possibility of reverter, which becomes possessory automatically if liquor is served.

170
Q

A first-time home buyer signed a mortgage document with City Bank to finance $190,000 over a 30-year term. The buyer made timely monthly payments for the first five years but then defaulted due to the loss of her job. When the bank foreclosed the buyer still owed $150,000 on the mortgage. Due to a dramatic downturn in the housing market, the bank was unable to resell the home for a year, despite a good faith effort. Eventually, the bank accepted an offer of $125,000 for the property. City Bank sued the defaulting owner for $25,000, plus interest and fees.

If the bank loses, if would most likely be for what reason?

(A) The jurisdiction followed a title theory.

(C) The buyer did not sign a promissory note.

(B) The jurisdiction followed a lien theory.

(D) The buyer was still unemployed.

A

(C) The buyer did not sign a promissory note.

171
Q

A television station’s weekend news anchorwoman was scheduled to do her usual weekend work on November 5-7. Unfortunately, she came down with acute laryngitis and emailed the station manager that she would be unable to work that weekend. On November 1, the station manager mailed letters to three freelance anchors, offering to pay a $500 bonus if one of them could take over the anchor duties for that weekend on short notice. One of the freelance anchors received the letter the next day and immediately dropped a note in the mail: “I would love to work for you next weekend. As a matter of fact, I need to! I’m a month behind in my rent and worried about getting evicted. Hope you don’t mind that I’ve dyed my hair red since we last spoke, but I could get a quick blonde dye if you like.” After the freelance anchor sent the letter, but before the station manager received it, the regular anchor phoned the manager to tell her that, due to a new cough syrup she was trying, the anchor was recovering quickly and thought she would be able to do her usual weekend work. The station manager immediately phoned the freelance anchor to tell her she was no longer needed. The freelance anchor was very disappointed and sued the television station for breach of contract. The station asserted in its defense that there was no contract, because the freelance anchor had never effectively accepted the station manager’s offer.

Will the station win on the grounds of this defense?

A Yes, because the manager had the right to reject the freelance anchor’s attempted acceptance, since it contained an additional term.

B Yes, because there was no salary term contained in the freelance anchor’s letter.

C No, because the station manager’s attempted revocation by telephone was not communicated through the same medium as the station manager’s offer.

D No, because the freelance anchor dispatched the acceptance before the station manager attempted to revoke her offer.

A

D No, because the freelance anchor dispatched the acceptance before the station manager attempted to revoke her offer.

172
Q

On several occasions, a father orally stated his intention to leave the family home to his two children. The son quitclaimed his interest in the property to the daughter for a small amount of money, and the daughter recorded the deed. The daughter, with the father’s permission, made substantial improvements to the property. A few years later, the father sustained a financial loss and sold the house. When the daughter found out about the sale, she brought suit to set aside the conveyance. The jurisdiction’s recording statute provides that “an unrecorded conveyance or other instrument is invalid as against a subsequent bona fide purchase for value and without notice.”

Did the buyer have notice of the daughter’s claim?

(A) No, because the quitclaim deed could not convey title.

(B) No, because the quitclaim deed did not provide constructive notice of the daughter’s claim.

(C) Yes, because the quitclaim deed from the son was recorded.

(D) Yes, because the quitclaim deed provided constructive notice of the claim.

A

(D) Yes, because the quitclaim deed provided constructive notice of the claim.

173
Q

At 10 a.m., an owner sold his oil field to an oil investor for $800,000. The owner gave the investor a general warranty deed. The investor immediately sent the deed to her attorney with instructions to record it the following morning. On her way out of her meeting with the owner, the investor passed a famous financier in the hall. The financier was on his way in to meet with the owner as well. Noticing the oil investor, the financier was curious as to what business she could have with the owner, but he decided not to let it trouble him. At 11 a.m., the owner gave the financier a general warranty deed to the same oil field he had just sold to the investor, in exchange for $1 million. The financier immediately signed the property over to his wife, in order to make up for having forgotten their anniversary. The financier’s wife immediately recorded the deed. The property is located in a race-notice jurisdiction. In an action by the financier’s wife to quiet title to the property, how will the court rule, and why?

(A) The investor owns the property, and the financier’s wife is entitled to damages of $1 million from the original owner.

(B) The investor owns the property, and the financier’s wife is entitled to no damages.

(C) The financier’s wife owns the property and is entitled to damages of $1 million.

(D) The financier’s wife owns the property but is not entitled to money damages.

A

(A) The investor owns the property, and the financier’s wife is entitled to damages of $1 million from the original owner.

174
Q

A merchant owned a large ranch in the western United States. The merchant promised a ranch foreman that he would receive the ranch when the merchant died. However, the merchants will left everything to the merchant’s niece. The niece lived in New York and never visited the ranch. She refused to respond personally to a letter from the foreman. The niece had the executor of the merchant’s estate shut down the ranch, fire the ranch foreman, and inform the ranch foreman that the ranch would never be his. Two years later, the ranch foreman returned to the land and reopened the ranch. The ranch foreman paid the property taxes and otherwise held himself out as the owner of the land for the next 21 years. The ranch foreman finally decided that he was getting too old to work the ranch, and he contracted for the sale of the land to a conservation group. Their contract required that the ranch foreman convey “good and marketable title.” Before delivering the purchase price, an attorney for the conservation group discovered that the title to the land was in the niece’s name, not the foreman’s name, and the conservation group refused to go forward with the purchase. The ranch foreman sued the conservation group for breach of contract. The relevant statutory period is 20 years. Will the ranch foreman prevail against the conservation group?

(A) Yes, because the ranch foreman is entitled to specific performance of the purchase and sale contract.

(B) Yes, because the ranch foreman is entitled to damages from the conservation group for breach of contract.

(C) No, because the niece, as the owner of the ranch, is entitled to enforce the contract and receive payment from the conservation group.

(D) No, because although the ranch foreman is the owner of the ranch, the foreman’s title is not marketable.

A

(D) No, because although the ranch foreman is the owner of the ranch, the foreman’s title is not marketable.

175
Q

A student signed a three-year lease for a condo with the condo owner. After one year, the student decided to transfer schools and assigned the remaining two years of his lease to a classmate. Six months later, the classmate went abroad and sub-leased the condo to a friend for one year. The friend took possession of the condo but stopped making rental payments to the condo owner.

May the condo owner recover from the classmate the unpaid rent?

A No, because the classmate is not in possession of the condo.
B No, because the classmate is a subtenant.
C Yes, because the classmate is in privity of estate with the condo owner.
D Yes, because the classmate did not receive the landlord’s permission to sublease the condo.

A

C Yes, because the classmate is in privity of estate with the condo owner.

176
Q

The owner of a car restoration business decided to retire and convey his building to his son. The owner hoped that his son would take over the business and settle down. The owner conveyed the building by quit claim deed for the sum of one dollar “to my son and his heirs for so long as the premises are used for a car restoration business.” As soon as the owner retired and moved to Florida, his son converted the building to an after-hours jazz club and abandoned the car restoration business.
Did the estate automatically revert to the owner?

(A) Yes, because the estate was defeasible.

(B) Yes, because the estate was a fee simple determinable with the possibility of reverter.

(C) No, because the estate was a fee simple determinable.

(D) No, because the estate was a fee simple subject to condition subsequent.

A

(B) Yes, because the estate was a fee simple determinable with the possibility of reverter.

177
Q

A university leased a two-story house near the campus to a newly hired professor for a term of five years. According to the terms of the lease, assignments and subleases were prohibited, and the professor agreed to pay $500 per month in rent. The professor moved in with his family and lived there for two years. After two years, the professor had a new child, and decided that he needed more space. The professor moved out and assigned the lease to a student. The student agreed to pay the university the $500 monthly rent. The university did not object to the assignment.

One year later, the student subleased the top floor of the house to a friend for $250 per month, which the friend paid to the student. The friend paid rent for the first six months, but then stopped. The next month, the student paid the university $250 and notified them that they should collect the remainder of the rent from the friend, who refused to vacate the house. The university instead sued the student to collect the unpaid $250 rent.

What is the most probable outcome?

A The university will recover, because the student remains liable.

B The university will recover, because they did not object to the assignment or sublease.

C The university will not recover, because the friend is the party that is liable.

D The university will not recover, because the professor remains liable.

A

The correct answer is:The university will recover, because the student remains liable.

Discussion of correct answer:When a tenant subleases the premises, the sublessee does not come into privity of contract or privity of estate with the landlord. The tenant remains in privity with the landlord, and the sublessee is only in privity with the tenant/sublessor. Therefore, the student remains liable to the university for the unpaid rent, because she is in privity with the university.

Discussion of incorrect answers:

Incorrect. The university will recover, because they did not object to the assignment or sublease. Whether the university objected to the assignment or sublease or not, the university still can recover against the student because the student is in privity of estate with the university. However, the university could not recover from the friend, because the friend is not in privity with the university.

Incorrect. The university will not recover, because the friend is the party that is liable. When a tenant subleases the premises, the sublessee does not come into privity of contract or privity of estate with the landlord. The tenant remains in privity with the landlord, and the sublessee is only in privity with the tenant/sublessor. Therefore, the student remains liable to the university for the unpaid rent because she is in privity with the university. The student could then sue the friend for the rent (as well as to evict the friend), because she is the sublessor and in privity with the friend.

Incorrect. The university will not recover, because the professor remains liable. It is true that, absent a novation, the professor remains liable to the university, because they are in privity of contract with each other. However, that will not prevent the university from holding the student liable for the unpaid rent that accrued during the student’s possession, because she, as assignee, is in privity of estate with the un

178
Q

A brother, sister, and cousin owned Blackacre as joint tenants with right of survivorship. After an ownership dispute arose among the three parties, The brother brought an action for partition against his sister and cousin. While the suit was pending, but before a final judgment had been rendered, the brother died in an automobile accident. A dispute then arose among the sister, the cousin, and the brother’s heirs as to their ownership rights in Blackacre.

In a suit to quiet title to Blackacre with all parties properly represented, which parties will be found to hold title to the property, and how?

The sister and cousin as joint tenants, because no final judgment was rendered on the brother’s partition action before his death.

The sister and cousin as tenants in common, because no final judgment was rendered on the brother’s partition action before his death.

The sister, the cousin, and the brother’s heirs as tenants in common, because the brother’s partition action survives his death.

The sister and cousin as joint tenants and the brother’s heirs as tenants in common, because the partition action, though not finalized, severed the joint tenancy only with respect to the interest owned by the brother.

A

The correct answer is: The sister and cousin as joint tenants, because no final judgment was rendered on the brother’s partition action before his death.

Discussion of correct answer: It is important to point out that the filing of a complaint for partition does not result in the severance of a joint tenancy. The rationale for this rule is that the complaint may be withdrawn at any time before the equity court compels partition. Moreover, the right of survivorship will attach if a joint tenant dies prior to that time. Accordingly, the sister and cousin will inherit the brother’s interest in Blackacre through the right of survivorship, and the brother’s heirs will have no interest in the property.

Discussion of incorrect answers:

Incorrect. The sister and cousin as tenants in common, because no final judgment was rendered on the brother’s partition action before his death. While it is true that the brother’s partition action will have no effect on the property, because no final judgment was rendered on it before his death, this answer choice incorrectly describes the sister and cousin as owning Blackacre as tenants in common.

179
Q

A commercial landlord leased a large two-story piece of urban real estate to a group of investors who planned to open an upscale night spot. The club owners installed a state-of-the-art sound system and recessed tract lighting with a retractable disco ball on the ground floor. They also dismantled the existing elevator and replaced it with a larger glass-enclosed lift to transport customers to the rooftop bar and restaurant.

Business proved to be even better than the investors had hoped and when their lease came to an end they declined to renew it, choosing instead to move to a trendier location.

When the landlord came to assess the property after his tenants had vacated, he found the sound system and the lighting intact, but the new elevator dismantled and removed.

If the landlord sues the investors for removal of the elevator, which of the following reflects the most probable outcome?

A The landlord will be entitled to compensatory and punitive damages.
B The landlord will be entitled to the replacement value of the original elevator.
C The landlord will be entitled to the return of the glass elevator.
D The landlord will not be entitled to any relief.

A

B The landlord will be entitled to the replacement value of the original elevator.

180
Q

A real estate developer purchased a piece of real property in a lien theory jurisdiction on which he planned to construct low-income housing. The developer obtained a $5 million loan from a bank, which recorded a lien against the property. The bank loan documents specified that installment payments would start within six months of construction. A few days later, the developer obtained a second loan from the city for $1 million to construct the low-income housing. The city subsequently filed a lien against the property.

Five months later, the developer was still in the middle of construction and asked the bank to reschedule the installment payments for its loan. The bank agreed to reschedule the installment payments and modified its senior lien. The city found out about this modification and claimed that its junior mortgage now had priority over the bank’s senior mortgage. Is the city’s claim correct?

(C) Yes, because the rescheduling of loan payments is a modification.

(B) No, because the rescheduling of installment payments did not materially prejudice the city.

(D) Yes, because the rescheduling of loan payments materially prejudiced the city.

(A) No, because the bank may make any modifications to its loan without losing its senior mortgage status.

A

(B) No, because the rescheduling of installment payments did not materially prejudice the city.

181
Q

On July 15, a seller and a buyer entered into a written agreement which provided, among other things, as follows: “The seller hereby agrees to sell the unimproved property described as (a metes and bounds description appeared here), consisting of 50 acres, to buyer for the price of $500,000. The seller to convey good and marketable title. Closing of escrow is to occur no later than October 1.” On August 20, the seller discovered a 10-year-old easement running through the center of the property which a former owner had granted to the city for a possible rail system idea that was later abandoned. The seller immediately notified the purchaser of the easement and that she had begun negotiations with the city manager and the city-planning department to repurchase the easement and thereby extinguish it.

On September 20, the landowner reported that the planning commission was leaning toward revival of the rail line project. When the buyer asked what was the likelihood of successfully repurchasing the easement, the seller replied, “I’m not sure.” The next day, September 21, the buyer notified the landowner that he was rescinding the contract. The seller brought an appropriate action for damages for breach of contract, alleging that the market value of the parcel on October 1 was $495,000.

Which of the following is the buyer’s strongest argument that he did not breach the contract of sale?

The seller did not possess good and marketable title when the contract of sale was formed.

The seller was unlikely to be able to tender good and marketable title within any reasonable period.

The seller’s entry into the contract relationship without good and marketable title constituted a constructive fraud upon the purchaser.

Since time was of the essence of the contract, the buyer was entitled to rescind as soon as there was any indication that performance by the seller might be delayed in any way.

A

The correct answer is: The seller was unlikely to be able to tender good and marketable title within any reasonable period.

Discussion of correct answer: Generally, the seller in a land sale transaction need not actually possess marketable title until the time for closing, when he is called upon to convey the property to the buyer. Thus, a defect in title existing at the time of formation but which is discovered afterward and which the seller has a reasonable chance of remedying does not give the buyer any cause for rescission prior to the date set for closing. However, there is a modern trend to grant the purchaser a right of immediate rescission if it appears unlikely that the seller will be able to cure her title problems. In this problem, the circumstances demonstrate that the seller may never be able to repurchase the easement; an easement is an encumbrance that renders title unmarketable. Therefore, the buyer’s strongest argument is to rely on the modern notion that he need not wait for the date set for performance, or a reasonable time after, to discover that the seller will never be able to perform.

182
Q

A landlord leased an apartment in a high-rise building to a tenant. The jurisdiction in which the leased premises were located had enacted a statute requiring landlords to provide running water in leased apartments for no less than 25 days out of each month. The lease agreement itself did not contain any term involving running water. In early June, while making repairs on another apartment, the landlord accidently burst a water pipe leading into the tenant’s apartment. As a result, the tenant did not have running water for the last three weeks of the month, and was forced to move out of the apartment. The tenant then sued the landlord for violating the lease’s covenant of quiet enjoyment.

What remedies are available to the tenant?

A The only remedies available to the tenant are those provided for in the statute, because the landlord has not violated the lease.

B The only remedies available to the tenant are those provided for in the statute, because the tenant vacated the apartment.

C The tenant is entitled only to a rent abatement, because she has been constructively evicted.

D The tenant may treat the lease as terminated and withhold rent, because she has been constructively evicted.

A

D The tenant may treat the lease as terminated and withhold rent, because she has been constructively evicted.

Discussion of correct answer:A constructive eviction occurs when: (1) the landlord acts to substantially and permanently interfere with the tenant’s use and enjoyment of the premises; and (2) the tenant moves out. Withholding something required by statute will constitute constructive eviction. In this example, while a requirement to provide running water was not included within the terms of the lease itself, it was required by statute. Therefore, the landlord’s failure to properly supply running water for the specified number of days constitutes a constructive eviction. In most jurisdictions, a tenant who has been constructively evicted may treat the lease as terminated and withhold the entire rent.

Discussion of incorrect answers:

Incorrect. The only remedies available to the tenant are those provided for in the statute, because the landlord has not violated the lease. Every lease contains a covenant of quiet enjoyment (express or implied), the breach of which may constitute actual or constructive eviction of the tenant. Here, the landlord withheld something required by statute, causing the tenant to move out. Therefore, although the lease agreement itself did not contain a provision requiring the landlord to supply running water, the landlord’s failure to comply with the statutory requirement constitutes a breach of the covenant of quiet enjoyment contained in the lease. As such, under the facts presented, the landlord has in fact violated the lease agreement, and the tenant is entitled not only to whatever remedies are available under the statute, but also to the usual remedies for violation of the covenant of quiet enjoyment–namely, treating the lease as terminated and withholding rent.

Incorrect. The only remedies available to the tenant are those provided for in the statute, because the tenant vacated the apartment. A landlord violates the covenant of quiet enjoyment contained in every lease and constructively evicts a tenant when: (1) the landlord acts in a way that substantially and permanently interferes with the tenant’s use and enjoyment of the leased premises; and (2) the tenant moves out. Withholding something required by a statute, such as heat or running water, constitutes a violation of the covenant of quiet enjoyment. Here, given that the landlord violated the statute and the tenant vacated the apartment as a result, it is clear that the landlord has breached the covenant of quiet enjoyment contained in the lease agreement. Therefore, the tenant is entitled to the normal remedies available for breach of this covenant. In most jurisdictions, a tenant whose landlord has breached the covenant of quiet enjoyment is entitled to treat the lease as terminated and withhold rent.

Incorrect. The tenant is entitled only to a rent abatement, because she has been constructively evicted. It is true that the tenant has been constructively evicted, because the landlord withheld something required by statute and the tenant moved out as a result. However, under the majority rule, a tenant who has been constructively evicted from the entire property is not entitled to merely a rent abatement, but may treat the lease as terminated and withhold the entire rent. Therefore, this answer choice is incorrect.

183
Q

A tenant resided in an apartment for 15 years. Over the course of the term of the lease, the tenant noticed that the seals around the doors and windows had worn down, causing the outside air to leak in, which in turn caused water to come in when it rained. The tenant demanded that the landlord fix the problem, but the landlord refused.

Under common law estate theory, must the landlord repair the premises?

A Yes, because the landlord is under a duty to repair the premises while a tenant is living in the building.
B Yes, because the landlord is responsible for keeping the building wind and water tight.
C No, because the tenant is under a duty to repair the premises, including keeping the building wind and water tight.
D No, because the tenant has a duty to repair damage that is the result of ordinary wear and tear.

A

C No, because the tenant is under a duty to repair the premises, including keeping the building wind and water tight.

184
Q

An heiress owned a restaurant on a pier. The heiress entered a land sale contract with an investor for the sale of the restaurant for $1.5 million. The contract specified that the closing was to take place on the first day of September, when the heiress would tender the deed and the investor would tender the purchase price less the down payment of $150,000 made on the date the parties signed the contract. A week before the closing, a hurricane struck the town in which the restaurant was located. The roof of the restaurant was blown off, and the ensuing water damage destroyed 90 percent of the interior of the business. The hurricane also severely compromised the structural integrity of the pier on which the restaurant was located.

On the first of September, the heiress presented the investor with the deed to the restaurant. The investor refused to tender the purchase price because the restaurant could not be operated as a restaurant without being rebuilt. The land sale contract of the parties was silent as to the allocation of risk.

In a suit for specific performance, will the heiress prevail?

No, because absent a provision to the contrary, the seller assumes the risk of loss after execution of a binding land sale contract.

No, because the heiress was in possession when the hurricane struck.

Yes, because the parties had a binding purchase and sale agreement.

Yes, if the heiress’ insurance had lapsed.

A

The correct answer is: Yes, because the parties had a binding purchase and sale agreement.

Discussion of correct answer: Under the doctrine of equitable conversion, the buyer bears the risk of loss when the parties have a binding land sale contract. The parties entered a valid land sale contract (there being no evidence to the contrary), and at the time the contract was executed, the heiress retained legal title to the property while the investor acquired equitable title to the property and the risk of loss. Because the hurricane struck and destroyed the restaurant after a binding land sale contract had been executed, the investor must bear the risk of loss under the doctrine of equitable conversion.

185
Q

A butcher represented to a banker that he owned Brownacre. On March 1, the butcher and banker entered into a written land sale contract. The terms of the contract provided that on August 1, the banker would pay the butcher $100,000, and the butcher would deliver to the banker a general warranty deed to Brownacre. Unknown to the banker, the butcher was only leasing Brownacre from an architect. On April 1, the banker and a singer entered into a written land sale contract. According to the terms of the contract, on September 1, the singer would pay to the banker $120,000 and the banker would deliver to the singer a general warranty deed to Brownacre. On August 1, the banker tendered $100,000 to the butcher. In exchange, the butcher delivered to the banker a purported general warranty deed to Brownacre. Then, on September 1, the singer paid the banker $120,000 and the banker delivered to her a purported general warranty deed to Brownacre. On September 15, the banker discovered that the architect, not the butcher, was and is the owner of Brownacre. The banker immediately approached the architect and convinced her to sell Brownacre to him. On September 30, the architect gave the banker a general warranty deed to Brownacre in exchange for $110,000. When the singer discovered that the banker had not had title to Brownacre on April 1 or September 1, she brought an action to recover damages from the banker.

Should the singer recover?

Yes, because the banker conveyed Brownacre to her by general warranty deed.

Yes, because the banker breached the warranty of marketable title implied in his land sale contract with the singer.

No, because, under the doctrine of estoppel by deed, the singer received title to Brownacre on September 30.

No, because the banker is protected from liability by the doctrine of equitable conversion.

A

The correct answer is: Yes, because the banker conveyed Brownacre to her by general warranty deed.

Discussion of correct answer: The singer should recover, because the banker conveyed Brownacre to her by general warranty deed. A warranty deed is a deed in which a grantor explicitly makes one or more assurances or covenants to the grantee regarding the quality of the title of the property conveyed. A general warranty deed contains six different covenants of title, including the covenant of seisin, the covenant of right to convey, the covenant against encumbrances, the covenant of quiet enjoyment, the covenant of warranty (which is virtually identical to the covenant of quiet enjoyment), and the covenant of further assurances. The first three of these covenants are considered “present” covenants in that they are violated, if at all, at the moment the grantor delivers the deed to the grantee. The remedy for the breach of covenants of title is recovery of damages. The banker used a general warranty deed in his attempt to convey Brownacre to the singer. He breached at least two covenants of title, and the singer is entitled to reject the after-acquired title and sue the banker for damages.

186
Q

A librarian and a teacher were best friends. The teacher often visited at the librarian’s lovely home and exclaimed how she wished she had such a beautiful home. The librarian asked the teacher if she would like to buy her home, so that the librarian’s estate would be more liquid. The teacher had no funds to purchase the home, so the librarian came up with the idea of a 30-year lease of the property at a reasonable monthly rental payment that was less than mortgage payments would have been for the teacher. The lease also granted the teacher the option to purchase the property at any time during the term of the lease for a specified price somewhat below its current market value.

Five years later, the librarian died, leaving a will that specifically devised the property to her niece. The teacher then notified the administrator of the librarian’s estate that she wished to purchase the property pursuant to her option under the lease, but the executor refused to sell. The teacher subsequently brought an action to compel the executor to execute the option pursuant to the lease agreement.

Who will prevail?

The teacher, because her option was valid and enforceable against the librarian’s executor.

The teacher, because the property was not subject to the specific devise.

The executor, because the purchase price was for below the current market value.

The executor, because the lease and option terminated upon the librarian’s death.

A

The correct answer is: The teacher, because her option was valid and enforceable against the librarian’s executor.

Discussion of correct answer: The option is valid and enforceable against the librarian’s executor. A lease does not automatically terminate upon the death of either the landlord or the tenant. The librarian’s death has no effect on the teacher’s lease or the purchase option, which is valid for the term of the lease. The librarian’s rights and obligations under the lease passed to her executor.

187
Q

A scientist encouraged all three of his children (two boys and one girl) to pursue scientific endeavors for the good of humankind. His two sons earned doctorates and engaged in medical research and went on to prestigious and noble professions. However, to the scientist’s chagrin, his daughter became an actress. The scientist found acting a frivolous and self-indulgent profession. Eventually, to the scientist’s infinite embarrassment, the daughter became a soft-porn star, who was well known in the industry. The scientist
threatened the daughter with disinheritance if she persisted in her “career.” She insisted that the films were art and that she would not give up her true calling. The scientist died at the age of 90. By will, he created a trust, with income to be distributed for life to the two sons. The remainder was to be distributed among the scientist’s grandchildren, including the daughter’s progeny. The daughter was specifically disinherited. At the time of the scientist’s death, one son is married but has no children; the other has two sons; and the
daughter has no children.

Which of the following best describes the interests that the scientist’s grandchildren hold in the testamentary trust?

(A) Vested remainders subject to open.

(B) Shifting executory interests.

(C) Contingent remainders subject to express condition.

(D) Contingent remainders.

A

(A) Vested remainders subject to open.

188
Q

A buyer and seller executed a contract that called for the buyer to purchase the seller’s house and the small business he operated out of the basement. The buyer expressly stated to the seller that she intended to live in the house and continue to operate the business. At the closing, the buyer learned that the business was in violation of local zoning ordinances. The buyer informed the seller that she wanted her down payment returned and that she would not complete the closing. The seller, who had already used the buyer’s down payment to make a down payment of his own on a condominium, sued for specific performance. The buyer defended on the grounds that the seller could not convey marketable title.

Who will prevail and why?

The buyer, because the seller misrepresented the legality of the business.

The buyer, because the zoning violation renders the title unmarketable.

The seller, because the buyer could have easily learned of the zoning restrictions before she signed the purchase- and-sale agreement.

The seller, because the zoning ordinance does not render the title unmarketable.

A

The correct answer is:The buyer, because the zoning violation renders the title unmarketable.

Discussion of correct answer:Title to property currently in violation of zoning regulations is unmarketable. It is true that it is the business that is in violation of zoning regulations here, and not the property. However, the seller knows that the buyer intends to continue the business after purchasing the property, and that the business is currently in violation. Because the seller sought to sell property currently in violation, which he knew the buyer intended to continue to use in the same manner, he did not have marketable title to convey. The seller is essentially selling what he knows will be a lawsuit for the buyer, because he knows the business violates zoning regulations and knows that the buyer intends to continue the business. Therefore, the buyer will be entitled to rescission of the contract and a return of her down payment.

189
Q

A landowner leased certain farmland to a soybean farmer for a 10-year term. The soybean farmer expressly covenanted for herself, heirs and assigns, to make semi-annual rental payments on or before February 1 and August 1 of each year. During the third year of the lease term, the soybean farmer assigned her lease to an herbalist by written instrument. The herbalist made timely rental payments to the landowner for four years. At that point, the herbalist reassigned her interest in the property to a banker. She did not know that the banker was insolvent and would not be able to make the semi-annual rental payments. The banker defaulted on making the rental payments to the landowner. The landowner sues the herbalist to recover the unpaid rent.

Is the court likely to find in favor of the landowner?

Yes, because the herbalist could not assign her rights to the banker because it would materially alter the risks and obligations for the landowner.

Yes, because the herbalist remained in privity of estate with the landowner despite reassigning the lease to the banker.

No, because, as an assignee, the herbalist never had an obligation to make rental payments to the landowner.

No, because the herbalist’s obligation to make rental payments to the landowner ceased when she reassigned her interest to the banker.

A

The correct answer is: No, because the herbalist’s obligation to make rental payments to the landowner ceased when she reassigned her interest to the banker.

Discussion of correct answer: After the herbalist reassigned her interest to the banker, the herbalist was no longer in privity of estate with the landowner. She was not in privity of contract with the landowner because there was no novation, substituting the herbalist for the farmer. Because the herbalist was no longer in privity of estate with the landowner, and was never in privity of contract with him, she was no longer liable to the landowner for the rent.

190
Q

A restaurant owner signed a mortgage to purchase a piece of property owned by the bank. The restaurant owner agreed to make monthly payments for a period of 15 years according to the mortgage agreement. The owner was in the process of obtaining the necessary permits to build the restaurant on the land, so he had not yet begun to construct the restaurant. Several months later, the bank conveyed the property to a developer who planned to put a hotel on the property. The developer paid cash to the bank for the property. One week later, the restaurant owner recorded his interest subject to the mortgage, and two days later the hotel developer recorded his conveyance. Assume this jurisdiction is a race jurisdiction.

Who has priority?

(A) The developer has priority, because he had an outright conveyance of the property.

(B) The developer has priority, because the conveyance need not be recorded to be enforceable.

(C) The restaurant owner has priority, because he recorded his interest first.

(D) The restaurant owner has priority, because an interest subject to a mortgage does not need to be recorded in a race jurisdiction.

A

(C) The restaurant owner has priority, because he recorded his interest first.

191
Q

A landlord and a tenant signed a lease agreement for a one-year term. One month into the lease term, the tenant changed jobs and abandoned the apartment because his new job was an inconvenient commute from the apartment. The landlord did not find a replacement tenant. At the end of the lease term, the landlord sent the tenant a bill for the unpaid rent.

In determining how much tenant owes, to which of the following would most courts give the greatest weight?

(A) How burdensome the tenant’s new commute is.

(B) Whether the lease contained a covenant of quiet enjoyment.

(C) Whether the landlord has met his burden of showing that he attempted to find a replacement tenant.

(D) Whether the tenant has met his burden of showing that the landlord failed to attempt to find a replacement tenant.

A

(C) Whether the landlord has met his burden of showing that he attempted to find a replacement tenant.

192
Q

An ornithologist’s will left his land to his wife for life, remainder to his children. The land consisted of 500 acres of wilderness, including two lakes. A small house was built on one corner acre of the land. The ornithologist and his wife moved into the house on the day they were married and lived there together until he died. During the ornithologist’s lifetime, the land was left in its natural state, except for the corner acre. His wife made a good living as a welder, and the couple never needed to use the land as a source of income. After the ornithologist’s death, the children left home to pursue their own careers. After many years alone, the wife began to lose her mental faculties. The ornithologist’s old college friend approached her with an offer
to rent the lakes on her property for use as catfish farms. The wife had been having difficulty paying bills and believed that the rental income from the fish farms might ease her financial burdens. She agreed to the offer and signed the lease to that effect. The friend’s firm established farms on each of the lakes. The farms themselves had little effect on the lakes. However, to reach the lakes and monitor the fish, the friend’s firm had to cut large swaths through the trees to construct roads, which he paved. The firm built buildings next to the lakes and outhouses to accommodate the workers. Water was piped from the lakes to the buildings, and wastewater was released to the streams draining the lakes. All the paving and construction created a serious runoff problem, which killed all the natural vegetation. All told, the catfish enterprise created a mess. The ornithologist’s children filed suit against their mother to stop the farming and to return the land to its previous condition.

Will the children prevail?

(A) No, because, as a life tenant, the wife has the right to make reasonable use of the land.

(B) No, because the friend created the damage, not the wife.

(C) Yes, because a life tenant is not entitled to use the land differently from the way the grantor used it.

(D) Yes, because as holders of a remainder interest, they are entitled to receive the land in a state no worse than that in which the grantor left it.

A

(D) Yes, because as holders of a remainder interest, they are entitled to receive the land in a state no worse than that in which the grantor left it.

193
Q

A teacher and a yoga instructor bought adjoining lots in a new residential subdivision. Each had custom homes constructed on their respective lots. The teacher also built a swimming pool with an extensive deck surrounding it. Two years later, the yoga instructor was doing some landscaping and struck concrete where she didn’t expect it. She hired a surveyor and learned that the concrete footings of the swimming pool extended across the property line into her lot, covering a triangular area 75’ long and varying in depth from a maximum of six inches at one end down to zero at the other. The yoga instructor brought an action for trespass against the teacher, seeking damages caused by the intruding concrete footings.

How is the court likely to rule?

(A) Against the yoga instructor, because the minimal intrusion does not touch or concern any present or contemplated use of the yoga instructor’s land.

(B) Against the yoga instructor, if the teacher acted in good faith and was unaware of the intrusion until after the swimming pool was in place.

(C) In favor of the yoga instructor, despite the fact that the yoga instructor has suffered no actual damages to her land.

(D) In favor of the yoga instructor, because the teacher did not meet the time requirement for adverse possession.

A

(D) In favor of the yoga instructor, because the teacher did not meet the time requirement for adverse possession.

194
Q

A woman leased a small storefront in a rundown area of the city so that she could open her own manicure and pedicure salon. The area was zoned commercial with a sub-classification of retail. The storefront had a large, plain glass window, and the building dated from 1925. The woman did not feel safe from possible break-ins at the location, so she wanted to replace the large window with glass block that was four inches thick. The zoning code stated that retail windows had to be plain glass and make up more than 50% of the side of the building facing the sidewalk.

What is the best path for the woman to pursue to achieve her goal?

File an application for an area zoning variance, showing practical difficulties in meeting the window requirement.

File an application for a use zoning variance, showing undue hardship in meeting the window requirement.

File for a special use permit.

File for a conditional permit.

A

The correct answer is: File an application for an area zoning variance, showing practical difficulties in meeting the window requirement.

Discussion of correct answer: A variance is an exception to the zoning code for a single piece of property. When the variance is sought from an area restriction, the petitioner must show that there are practical difficulties in meeting the requirements of the zoning code or that the requirements are unreasonable or create an undue hardship. Here, the woman must show that it is not practical to secure the premises against criminal break-ins and vandalism without installing glass block in the window space.

195
Q

A landowner owned two adjacent vacant lots in a hilly area. Lot 1 was downhill of Lot 2, and directly between Lot 2 and the nearby ocean. In 2030, the landowner sold Lot 2 to an accountant. To preserve the ocean view for Lot 2, the deed by which she conveyed contained a covenant that she would never build any structure on Lot 1 with a height greater than 20 feet. The accountant immediately recorded his deed and constructed a custom two-story home with the entire second level facing Lot 1 and the ocean and consisting primarily of windows. In 2035, the landowner sold Lot 1 to a sociologist, who had no knowledge of the restriction imposed upon Lot 1 by the deed to Lot 2. The landowner-sociologist deed also made no mention of the restriction. In 2037, the sociologist commenced construction of a three-story home on Lot 1, the plans to which indicated that the roofline of this home would be 30’ high. The accountant sued to enforce the height restriction described in his deed by enjoining construction of the sociologist’s home. The jurisdiction maintains both an official grantor-grantee index and an official tract index. The applicable recording act provides, “Every conveyance of real property (other than lease for a term not to exceed one year) is void as against any subsequent purchaser or mortgagee of the same property in good faith and for a valuable consideration, whose conveyance is first duly recorded.”

What is the strongest argument in favor of the accountant?

An equitable restriction binds successive owners of the burdened land independently of any notice they may receive about it.

The covenant regarding height is in the record chain of title as to Lot 1.

The sociologist was put on inquiry notice of the restriction by the circumstances–the presence of a potential view always requires potential purchasers to investigate height restrictions.

The landowner could not convey to the sociologist a greater interest in Lot 1 than she herself owned.

A

The correct answer is: The covenant regarding height is in the record chain of title as to Lot 1.

Discussion of correct answer: So long as a deed is considered within the chain of title of a particular piece of real property, any prospective buyer is charged with constructive notice of the contents of that deed. The facts of this question present the difficulties created where an interest in land affecting one parcel is mentioned only in a deed concerning a different parcel of land. A grantee is charged with constructive notice of the contents of deeds of adjacent properties when all of the properties are derived from a common grantor. Thus, the sociologist must examine the title record at least forward from the point in time where the common grantor sold one of the adjacent parcels to determine whether there are any interests–such as the equitable restriction at issue here–that affect her property but are contained only in the deed to adjacent properties. An additional reason for considering the equitable restriction in the landowner-accountant deed within the chain of title to Lot 1 is the fact that the jurisdiction maintains a tract index. This means that all recorded instruments affecting a particular parcel are listed in one place as pertaining to that parcel, greatly facilitating a search for interests such as the height restriction, which should have been noted on the Lot 1 index when the landowner-accountant deed was recorded as to Lot 2. For all of these reasons, the sociologist will be charged with notice of the equitable restriction mentioned in the accountant’s deed, and the recording act will not protect her later-acquired interest (because she cannot purchase in good faith when she has notice, constructive or otherwise, of the height restriction).

196
Q

A woman leased an apartment from a landlord. When the woman arrived to take move in, she found that the former tenant was still in the apartment, and refused to leave. As a result, the woman refused to pay any rent to the landlord. The landlord then sued the woman to enforce the lease.

How should the court rule?

A For the landlord, because the American rule requires him to provide only the right to possess the apartment.

B For the landlord, because the English rule requires him to provide only the right to possess the apartment.

C For the woman, because the American rule requires the landlord to provide her with both the right to possess and actual possession of the apartment.

D For the woman, because the English rule requires the landlord to provide her with both the right to possess and actual possession of the apartment.

A

D For the woman, because the English rule requires the landlord to provide her with both the right to possess and actual possession of the apartment.

Discussion of correct answer:The majority rule is the English rule, which requires that a landlord provide not just the right to possession, but also actual possession of the leased property. Therefore, it is the landlord’s responsibility to evict the former tenant and deliver actual possession of the ranch to the woman.

Discussion of incorrect answers:

Incorrect. For the landlord, because the American rule requires him to provide only the right to possess the apartment. The American rule is not the majority rule, but rather, the minority rule. Had the question indicated that the American rule was to be followed, then the landlord would prevail. The American rule provides that the landlord’s duty is merely to transfer the right to possess the land. This is the minority rule, and if applicable, then the landlord has fulfilled that obligation, and the woman would have to force the former tenant out of the apartment through an eviction action in order to take her rightful possession. However, the majority rule, which is the English rule, requires that the landlord must also deliver actual possession of the property. Therefore, it is the landlord’s responsibility to evict the former tenant and thereby deliver possession of the apartment to the woman.

Incorrect. For the landlord, because the English rule requires him to provide only the right to possess the apartment. This is only a partial application of the English rule. The English rule requires the landlord to deliver both the right to possess and actual possession of the property, not merely the right of possession. As such, this answer choice is incomplete.

Incorrect. For the woman, because the American rule requires the landlord to provide her with both the right to possess and actual possession of the apartment. The American rule is the minority rule, not the majority rule. Because the facts do not state that the minority rule would apply, the majority English rule would apply instead. Additionally, this answer choice incorrectly applies the American rule. The American rule requires only that the landlord deliver the right to possess the property; it does not require the landlord to deliver actual possession.

197
Q

A landlord owned a commercial property which he originally leased to a fast food chain. When that lease term ended, the landlord entered into a new lease with a bakery. The bakery was scheduled to take possession of the premises on May 1, but when the bakers arrived they were denied entrance by the fast food restaurant, which was still in possession. Fifteen days later the bakery still had no access to the property. The jurisdiction where the property is located follows the English rule.

If the bakery sues the landlord, which of the following most accurately reflects the probable outcome?

A. The bakery will win because the landlord is required to provide legal possession on the first day of the lease term.

B. The bakery will win because the landlord is required to provide actual possession on the first day of the lease term.

C. The landlord will win because it is the bakery’s responsibility to eject the restaurant.

D. The landlord will win because the bakery should have filed their lawsuit against the restaurant.

A

B. The bakery will win because the landlord is required to provide actual possession on the first day of the lease term.

Discussion of correct answer: All jurisdictions require that legal possession of leased premises be given to a tenant on the first day of the lease term. However, jurisdictions differ regarding actual possession. In jurisdictions following the English rule, the landlord is responsible for providing actual as well as legal possession to his tenant, which means that it is his job to eject any holdover tenants. In some American jurisdictions, the landlord is not responsible for providing actual possession, so the new tenant would have to deal with the holdover. Since the question clearly identifies this as an English jurisdiction, the bakery will win because landlord failed to provide them with actual possession on May 1.

198
Q

A student challenged a professor to a backgammon match; the loser to pay $10,000 to the winner. They played the match and the student won. The professor said that she did not have $10,000 in cash at that time, but that she would sell her beachfront property to the student for $200,000, which is $10,000 less than its current market value. “So, I’ll pay you $200,000, and the beachfront property is mine, right?” said the student. “Agreed,” said the professor, “we can close next week.” The next day, the student received a note from the professor stating, “I regret that I cannot go through with our agreement to sell my beachfront property to you. Although the $200,000 purchase price we agreed to is a fair price, I just can’t part with the property after all.” The note was signed by the professor. The next week, when the student attempted to give $200,000 in cash to the professor, the professor refused to accept the money or to execute a deed conveying the beachfront property to the student. The student immediately commenced an appropriate action to compel the professor to accept the money and convey the property.

The court should award judgment to which party?

The student, because he won the bet.

The student, because he had an enforceable purchase contract with the professor.

The professor, because of the Statute of Frauds.

The professor, because of a public policy defense.

A

The correct answer is: The student, because he had an enforceable purchase contract with the professor.

Discussion of correct answer: After the backgammon match, the student and the professor entered into an oral agreement for the purchase of the professor’s beachfront property. Key to the student’s success in this case is the written documentation of the agreement in the note from the professor. The written documentation contained a signature by the party to be charged, a description of the parties, a description of the property, and the price. This writing is sufficient to satisfy the Statute of Frauds. By trying to get out of the agreement with this writing, the professor proved the existence of the agreement.

199
Q

In a written agreement, a landlord leased his home to a tenant for a term of three years, ending on December 31 of the third year, at the rate of $1,000 per month. The lease provided that the tenant could sublet and assign the lease. The tenant lived in the home for one year and paid the rent promptly each month. After one year, the tenant leased the home to a student for one year at a rent of $1,000 per month. The student took possession of the home and lived there for six months, but because of her unemployment, she never made any rent payments. After six months, the student abandoned the home, which then remained vacant for the balance of that year.

The tenant retook possession of the home at the beginning of the third and final year of the lease term, but made no further rent payments to the landlord. At the end of the lease term, the landlord brought an action against both the tenant and the student to recover $24,000, the unpaid rent for the second and third years of the lease term.

Assuming the landlord is successful, how will the court rule regarding damages?

(A) Against the tenant individually for $24,000, and no judgment against the student.

(B) Against the tenant individually for $18,000, and against the student individually for $6,000.

(C) Against the tenant individually for $12,000, and against the tenant and student jointly and severally for $12,000.

(D) Against the tenant individually for $18,000, and against the tenant and student jointly and severally for $6,000.

A

(A) Against the tenant individually for $24,000, and no judgment against the student.

200
Q

A bartender and a chef owned a restaurant as joint tenants with right of survivorship. The bartender executed a mortgage on the restaurant to secure a debt to a judgment creditor. Before the indebtedness to the judgment creditor had been satisfied, the bartender died intestate, leaving his son as his sole heir. In a jurisdiction that follows the lien theory of mortgages, if an appropriate action is brought to determine ownership of the restaurant, which of the following is most likely to reflect the court’s judgment?

(C) The chef owns the restaurant in fee simple, subject to the judgment creditor’s mortgage.

(B) The bartender’s son and the chef own the restaurant in undivided half interests as tenants in common, with the entire interest subject to the judgment creditor’s mortgage.

(A) The bartender’s son and the chef own the restaurant in undivided half interests as tenants in common, with the bartender’s interest subject to the judgment creditor’s mortgage.

(D) The chef owns the restaurant in fee simple.

A

(D) The chef owns the restaurant in fee simple.

201
Q

A son inherited a 20-acre tract of land in fee simple from his father. The son never resided on the property. A man went into possession of the property and lived there continuously for eight years. During that period, he held himself to be the rightful owner of the property. His possession was open, notorious, and exclusive. At the end of the eight-year period, the son visited the property and encountered the man. The son agreed to allow the man to continue in possession under a leasehold arrangement.

The son proceeded to lease the property to the man for a two-year term at a rental fee of $2,000 per month. The period of adverse possession in this jurisdiction is 10 years. The man continued to reside on the property under the terms of the lease agreement. When the two-year lease expired, the man asserted ownership in fee simple by adverse possession. He claimed that he was in continuous possession for a period of 10 years, once the initial eight years was coupled with the two years of occupancy under the lease. The son sought to oust the man from the property, but the man refused to vacate.

In an appropriate action to quiet title, for whom should the court rule?

The man, because he has been in continuous possession for the statutory period.

The man, because his adverse possession for the first eight years can be “tacked” to the two years of the lease.

The man, because he paid taxes for all 10 years.

The son, because the lease agreement presupposes that the man’s possession was no longer adverse.

A

The correct answer is: The son, because the lease agreement presupposes that the man’s possession was no longer adverse.

Discussion of correct answer: In order to acquire title to property by adverse possession, the possession must be: (1) exclusive; (2) open and notorious (meaning not secret or clandestine); (3) continuous for the statutory period; and (4) hostile and adverse (meaning without the dispossessed owner’s permission). The period of adverse possession of one possessor can be “tacked” to the period of adverse possession of another possessor when there is privity between the two. However, this question does not involve tacking. Rather, during the period of adverse possession, the man entered into a lease agreement with the son. The two- year lease was with the owner’s permission, and so the possession during the leasehold was neither adverse nor hostile.

202
Q

The owner of a sprawling orchard executed a general warranty deed in favor of a buyer, conveying his “24-acre parcel of land known as Blackacre.” After the owner died, his widow challenged the legal sufficiency of the transfer, arguing that Blackacre in fact consisted of 20 acres, and the deed was therefore invalid for having failed to describe the land conveyed with sufficient specificity.

If ownership of Blackacre turns solely on the sufficiency of the property description contained in the deed, how should the court determine title?

For the buyer, because the land conveyed has been adequately described despite the discrepancy in area.

For the buyer, because the deed failed to describe the land conveyed in metes and bounds.

For the widow, because the area of land set forth in the deed did not comport with the actual area of the property purportedly conveyed thereby.

For the widow, because the owner could not convey a greater amount of land than he actually owned.

A

The correct answer is: For the buyer, because the land conveyed has been adequately described despite the discrepancy in area.

Discussion of correct answer: There is no precise standard for assessing the sufficiency of a description of land that must be contained in a deed or a contract for the sale of land. Many different systems are used and are acceptable. Extrinsic evidence is generally admissible to clear up ambiguities. The test is essentially whether a court can effectively identify the land so as to enforce the contract or effectuate the conveyance. Since Blackacre was described by name and location, the description was sufficient to effectuate the conveyance despite the relatively minor discrepancy in the area of the land involved.

203
Q

A merchant owned a large ranch in the western United States. The merchant promised a ranch foreman that he would receive the ranch when the merchant died. However, the merchant’s will left everything to the merchant’s niece. The niece lived in New York and never visited the ranch. She refused to respond personally to a letter from the foreman. The niece had the executor of the merchant’s estate shut down the ranch, fire the ranch foreman, and inform the ranch foreman that the ranch would never be his. Two years later, the ranch foreman returned to the land and reopened the ranch. The ranch foreman paid the property taxes and otherwise held himself out as the owner of the land for the next 21 years.

The ranch foreman finally decided that he was getting too old to work the ranch, and he contracted for the sale of the land to a conservation group. Their contract required that the ranch foreman convey “good and marketable title.” Before delivering the purchase price, an attorney for the conservation group discovered that the title to the land was in the niece’s name, not the foreman’s name, and the conservation group refused to go forward with the purchase. The ranch foreman sued the conservation group for breach of contract. The relevant statutory period is 20 years.

Will the ranch foreman prevail against the conservation group?

Yes, because the ranch foreman is entitled to specific performance of the purchase and sale contract.

Yes, because the ranch foreman is entitled to damages from the conservation group for breach of contract.

No, because the niece, as the owner of the ranch, is entitled to enforce the contract and receive payment from the conservation group.

No, because although the ranch foreman is the owner of the ranch, the foreman’s title is not marketable.

A

The correct answer is: No, because although the ranch foreman is the owner of the ranch, the foreman’s title is not marketable.

Discussion of correct answer: The ranch foreman is the owner of the ranch by adverse possession because his possession was open, visible, and notorious; actual; exclusive; hostile; and continuous for the statutory period. Title acquired by adverse possession may become marketable if the adverse possessor quiets title before attempting to convey it. However, because there is no evidence that the ranch foreman had brought an action to prove his adverse possession claim and quiet title, the ranch foreman’s title was not marketable. Modern courts might still find that title acquired by adverse possession is marketable if the adverse possession has been for a very long time; the record owner is unlikely to sue; and the record owner is unlikely to prevail if she did. The facts do not suggest that the niece, as title holder of record, would be unlikely to sue. Therefore, the ranch foreman would not prevail.

204
Q

An engaged couple bought a house, and the seller drafted the deed from the seller to “[the man] and [the woman].” A few months after they moved in to the house, the couple split up. The man moved out of the house and executed a quitclaim deed transferring his interest in the property to a friend whom the woman despised.

Can the man convey his interest in the property to the friend without the woman’s consent?

(A) No, because an attempted conveyance by one tenant by the entirety is void.

(B) No, because a joint tenant may not convey a partial interest without the permission of the other joint tenant.

(C) Yes, because the conveyance to the friend amounted to a voluntary partition of the property.

(D) Yes, because a tenant in common may convey his interest freely.

A

(D) Yes, because a tenant in common may convey his interest freely.

205
Q

A man conveyed his farm to his sister, but she did not record her deed. A few days later, the man conveyed the same farm to a friend. The friend recorded the deed, even though he had actual knowledge of the prior conveyance to the sister. The friend then sold the farm to a farmer, who had no notice of the conveyance to the sister and who duly recorded his deed. The day after the farmer recorded his deed, the sister recorded her deed. The jurisdiction has a notice recording statute.

Who owns the farm?

(A) The farmer.
(B) The sister.
(C) The friend.
(D) The man.

A

(A) The farmer.

206
Q

The owner of an office supplies store rented the building in which his store was located as a periodic tenant for the past six years. Several years ago, the owner installed a number of display shelves on the walls. Recently, the landlord of the building visited the office supplies store to speak with the owner about his lease. Both parties agreed that the lease would end at the end of the next month. The owner, who planned to rent a larger space for his growing business, intended to remove the shelves from the walls and take them with him. However, the landlord told the owner that he expected him to leave the shelves behind, since they were affixed to the walls.

Who is correct?

A. The landlord, because a chattel which is annexed to the land by a tenant becomes a fixture.

B. The landlord, because a fixture annexed to a building by a periodic tenant must remain in the building upon termination of the lease.

C. The tenant, because a trade fixture may be removed by a tenant during the tenancy as long as the tenant repairs any damage caused by its removal.

D. The tenant, because a trade fixture may be removed by a tenant at least one month prior to the end of the tenancy as long as the removal of the fixture does not cause any damage.

A

C. The tenant, because a trade fixture may be removed by a tenant during the tenancy as long as the tenant repairs any damage caused by its removal.

Discussion of correct answer: When a tenant annexes a chattel to the land during his tenancy to advance his trade or business, it becomes a trade fixture. In general, a tenant is free to remove a trade fixture during the tenancy (and in many states, for a brief period after the termination of the tenancy) but must repair any damages caused by the removal. Therefore, the owner of the office supplies store may indeed remove the shelves as long as he repairs any damages that causes.

207
Q

A railroad owned a railway station in the heart of a large city. The railway station was beautifully designed in 1913 and attracted many tourists. The railroad wished to lease out the top of the railway station so that the lessee could build a 55-story office building on top of it. The city, however, designated the railway station as a historic landmark and disapproved the plans. The city said that the railroad and the lessee could build the office building on other property belonging to the railroad. The railroad and the lessee filed suit, claiming that the city had taken private property without just compensation.

Do the plaintiffs have a valid claim for compensation?

No, because the plaintiffs can build elsewhere.

No, because not all economically beneficial uses have been prohibited.

Yes, because the city cannot “spot” zone, even if it is for the public welfare.

Yes, because the plaintiffs have been deprived of the highest use of the property.

A

The correct answer is: No, because not all economically beneficial uses have been prohibited.

Discussion of correct answer: The Fifth Amendment provides that private property shall not be taken for public use without just compensation. This prohibition applies to the states through the Due Process Clause of the Fourteenth Amendment. The U.S. Supreme Court held, in Penn Central Transportation Co. v. New York City [438 U.S. 104 (1978)], that there were at least three factors to consider when deciding whether a zoning ordinance or other land-use regulation created a “taking” under the Fifth Amendment: (1) the economic impact of the regulation on the claimant; (2) the extent to which the regulation interferes with investment-backed expectations; and (3) the character of the governmental action. Applying these factors, the Court found that the economic impact was minimal and the investment-backed expectations remained the same because the railway station would go on being used as a railway station, as it had for the past 65 years. The character of the governmental action was a proper designation of historical landmarks for the benefit of the public. Thus, because the plaintiffs could still make economically beneficial use of the railway station, there was no compensable taking.

208
Q

A teacher borrowed $50,000 from a bank and issued to the bank a mortgage on Goldacre, a parcel owned by the teacher. The mortgage, which was properly recorded, provided that the debt was to be paid in full by September 1, 2025. Six months later the teacher borrowed $100,000 from a credit union and gave the credit union a mortgage on Goldacre. The credit union recorded the mortgage. One year later the teacher and the bank entered into a modification agreement in which the bank agreed to extend the due date on mortgage held by bank to September 1, 2030.

Which mortgage has top priority?

(B) The credit union’s mortgage, because the modification on the bank’s mortgage changed its priority.

(C) The bank’s mortgage, because it was recorded prior to the credit union’s mortgage.

(A) The credit union’s mortgage, because it was issued for a higher loan amount than the bank’s mortgage.

(D) The bank’s mortgage, because priority is unaffected by the modification agreement.

A

(D) The bank’s mortgage, because priority is unaffected by the modification agreement.

209
Q

A landlord leased an apartment to a college senior for $1,000 a month. After six months, the college senior assigned his interest to a college junior. The junior paid the rental on the apartment for two months and then assigned his interest to a sophomore. The sophomore paid the rental for one month but did not pay the rental for the next three months. Then, the sophomore assigned the premises to a freshman who took possession but did not pay any rental to the landlord for five months. The landlord brought an action against the freshman to recover the outstanding rent.

How much rent will the landlord be able to recover from the freshman?

(A) $8,000.
(B) $7,000.
(C) $5,000.
(D) $4,000.

A

(C) $5,000.

210
Q

A consumer purchased a new car from a car dealer. The car dealer performed a thorough inspection of the car before the sale was completed. One evening, the consumer was driving along a city street when he saw the traffic light turn from green to yellow. He sped up, hoping to cross the intersection before the light turned red. However, he quickly realized that he could not do so and applied the brakes, which failed because of a defect in the brake mechanism. The consumer swerved to avoid hitting a bus that was crossing the intersection at a right angle to him. As a result of the swerve, the consumer’s car rode up on the sidewalk and overturned. The consumer was severely injured.

In an action by the consumer against the car dealer based on strict liability, is the consumer likely to recover?

(A) No, because the car dealer carefully inspected the car before selling it.
(B) No, because the consumer contributed to his own injury by speeding up.
(C) Yes, because the brakes failed because of a defect present when the consumer purchased the car.
(D) Yes, because the brakes failed while the consumer was driving his car.

A

(C) Yes, because the brakes failed because of a defect present when the consumer purchased the car.

211
Q

A nine-year-old boy’s parents entered him in several off-road motorcycle races. At one such event, the boy was so overwhelmed by the presence of the number of spectators that he drove off the course into the crowd. An injured spectator brought a lawsuit against the boy for negligence. At trial, a psychiatric expert testifying for the defense said that the boy’s reactions were normal for a nine-year-old. The boy’s attorney argued that this shows that the boy acted reasonably given his age.

Will the attorney’s argument regarding the standard of care be successful?

A. No, because common care is not necessarily reasonable care.

B. No, because the boy’s actions must be judged according to the standard of care for adults.

C. Yes, because the boy acted as a reasonably prudent person of his age would have in the circumstances.

D. Yes, because the negligence that caused the accident was the negligence of the boy’s parents in entering him in the race.

A

B. No, because the boy’s actions must be judged according to the standard of care for adults.

212
Q

A landlord and a tenant lived together in an apartment as roommates. The lease agreement between the two parties stated that the tenant would have access to all of the apartment’s common areas, including the living room, bathroom, and kitchen. Believing that the tenant had damaged the television in the living room, the landlord removed all of the tenant’s personal property from the living room, locked the door to the living room, and told the tenant that the tenant would no longer have access to that room. Although the tenant objected, the tenant continued to reside in the other rooms of the apartment.

To what relief is the tenant entitled under the law of most jurisdictions?

(A) The tenant is relieved of all liability for rent, because he has been actually evicted.

(B) The tenant is relieved of all liability for rent, because he has been constructively evicted.

(C) The tenant is only entitled to a rent abatement, because he has been partially evicted.

(D) The tenant is only entitled to a rent abatement, because he has been constructively evicted.

A

(A) The tenant is relieved of all liability for rent, because he has been actually evicted.

213
Q

A rancher granted Greenacre to his son-in-law, the son-in-law’s heirs and assigns, provided that the son-in-law is survived by
issue of himself and the rancher’s daughter, but if the son-in-law dies without issue of himself and the rancher’s daughter, then to the
rancher’s nephew. The son-in-law has discovered that the mountains on the eastern edge of Greenacre are rich in copper.

May he properly mine copper from Greenacre?

(A) No, because, under the open mines doctrine, the son-in-law is not permitted to open a new mine after he takes possession of Greenacre.

(B) No, because mining at Greenacre would invade the nephew’s future interest in the property.

(C) Yes, because the son-in-law holds a fee simple determinable, so he may use Greenacre as he pleases.

(D) Yes, because the son-in-law holds a fee simple subject to an executory interest, so he may use Greenacre as he pleases.

A

(D) Yes, because the son-in-law holds a fee simple subject to an executory interest, so he may use Greenacre as he pleases.

214
Q

A highly successful land developer grants a chain of hotels to his spoiled daughter. The deed reads “to my daughter for life, then to my son and his heirs.” Shortly after the grant is made, the developer passes away. The daughter, the executor of the developer’s estate, and the son are fighting over a number of things having to do with the land that was granted to the daughter for life.

Which of the following is true?

(A) The estate must make all mortgage payments.

(B) The son must pay all taxes on the property.

(C) The daughter has no common law duty to pay taxes.

(D) The daughter has a duty to pay the interest on the mortgage.

A

(D) The daughter has a duty to pay the interest on the mortgage.

215
Q

A landlord and tenant entered into a residential lease agreement. According to the terms of the agreement, the lease would continue until the landlord, in his sole discretion, terminates it by providing 30 days’ notice. Two years later, the tenant received a new job and moved out of state, assigning the lease to a new tenant and notifying the landlord. The landlord then filed an action to evict the new tenant.

What is the most likely outcome?

A The landlord will prevail, because the landlord had the discretion to terminate the lease at any time.

B The landlord will prevail, because the lease was terminated when the tenant assigned it.

C The new tenant will prevail, because the lease did not prohibit assignments.

D The new tenant will prevail, because the landlord did not provide the required notice.

A

The correct answer is: B. The landlord will prevail, because the lease was terminated when the tenant assigned it.

Discussion of correct answer:A tenancy that has no fixed duration and lasts only as long as the landlord or tenant desire is an at-will tenancy. In this case, the lease agreement had no fixed duration and provided that only the landlord would have the power to terminate the lease. However, most courts will interpret this language as creating an at-will tenancy that can be terminated by either party. An at-will tenancy is terminated automatically if: (1) either party dies; (2) the tenant commits waste; (3) the tenant attempts to assign his interest; (4) the landlord transfers his interest; or (5) the landlord transfers the premises to a third party for a term of years. Because the tenant transferred his interest to a new tenant, the lease was terminated at that time.

Discussion of incorrect answers:

Incorrect. The landlord will prevail, because the landlord had the discretion to terminate the lease at any time. It is true that this was an at-will tenancy, and as such, the landlord had the discretion to terminate the lease at any time, by providing the notice as required by the lease agreement. However, because this was an at-will tenancy, it was terminated when the tenant assigned his interest to a new tenant. As such, the landlord did not need to provide notice, because the lease had already ended.

Incorrect. The new tenant will prevail, because the lease did not prohibit assignments. As a general rule, leases are transferable unless there is a lease term prohibiting assignments or subleases. However, this is not the case for at-will tenancies, such as the one in this question. An at-will tenancy will terminate if either party attempts to transfer their interest. Therefore, when the tenant tried to assign his interest in the property, the lease was terminated.

Incorrect. The new tenant will prevail, because the landlord did not provide the required notice. The lease agreement in this question was an at-will tenancy. At modern law, where a lease provides only the landlord with the discretion to terminate the agreement, most courts will find that an at-will tenancy was created pursuant to which either party could terminate the lease. According to the terms of this lease, therefore, either party could terminate the lease by providing 30 days’ notice. However, an at-will tenancy is terminated when either party attempts to transfer their interest. Therefore, when the tenant assigned the lease, the tenancy was terminated, and the landlord did not need to provide any notice.

216
Q

A man wanted to buy a house but did not have enough cash saved, so he signed a mortgage with a bank in order to obtain a loan. The mortgage deed identified the man as the mortgagor and the bank as the mortgagee. It also specifically stated that the parties intended to create a security interest in the mortgage. Both parties then signed the mortgage deed. What is missing from the mortgage deed?

(A) A description of the property with sufficient detail to put a subsequent bona fide purchaser on notice of the mortgage.

(B) A description of the property which satisfies the Statute of Frauds.

(C) The amount of the loan, interest rate of the loan, whether the interest rate is fixed or variable, and the term of the loan.

(D) A “due on sale” clause, which requires the entire balance due on the note to be paid before the property may be transferred by the mortgagor/seller to a buyer.

A

(A) A description of the property with sufficient detail to put a subsequent bona fide purchaser on notice of the mortgage.

217
Q

A soccer player bought a home and some land from his uncle under an installment land sale contract. The contract called for payment of the installments on the first of every month. The soccer player was frequently traveling and, more often than not, paid his installment very late in the month. Sometimes, the soccer player had to pay for three months all at once because he was so delinquent. The uncle never complained because he knew that payment would arrive eventually. Once, after three months had gone by and the uncle had not received a payment, the uncle learned by reading the sports pages that the soccer player had been cut from the team. The uncle called the soccer player and demanded that the soccer player bring his payments up to date or face foreclosure. The soccer player did not respond and the uncle began foreclosure proceedings in the appropriate venue.

Will the uncle be permitted to foreclose on the property?

Yes, because the soccer player was in default on his installment land sale contract.

Yes, because the soccer player’s grace period of three months had expired.

No, because the soccer player’s grace period did not begin until the uncle made the phone call.

No, because the uncle did not ask for timely payment in writing.

A

The correct answer is: No, because the uncle did not ask for timely payment in writing.

Discussion of correct answer: In an installment contract, if the seller has permitted the buyer to make late payments, the seller may be deemed to have waived the right to timely payment. To regain the right of timely payment, the seller must inform the buyer, in writing, of the timeliness requirement and give the buyer reasonable time to make back payments owed. So in this case, the uncle accepted very late payments on a regular basis. To establish a timely payment schedule again, the uncle’s phone call was not enough. The uncle needed to provide written notice to the soccer player and give the soccer player a reasonable amount of time to catch up on his payments. Therefore, this is the correct answer choice.

218
Q

A conservationist conveyed an undeveloped tract of land “to my nephew, provided that the property shall never be used for any commercial purpose. If any portion of said tract is used for a commercial purpose, the grantor may re-enter as of the grantor’s former estate.” This deed was properly recorded. The nephew died intestate 18 years later, survived by his wife. The conservationist died soon after, survived by his two daughters. The nephew’s wife conveyed the tract to a developer, who recorded the deed and constructed a hotel complex on the tract. The deed did not mention the restriction noted above. One of the daughters brought suit to enjoin the developer from constructing the hotel.

Is she likely to prevail?

(A) No, because a common development scheme had been established for the entire tract.

(B) No, because the daughters did not receive a valid possibility of reverter from their father.

(C) Yes, because either daughter has the power of termination for condition broken.

(D) Yes, because either daughter’s right to the tract vested immediately upon the developer’s construction of the hotel complex.

A

(C) Yes, because either daughter has the power of termination for condition broken.

219
Q

Upon a doctor’s death, the doctor’s will devised all his real property, wherever situated, to a nurse and all his personal property to a resident. During probate of the will, it was discovered that prior to his death, the doctor had entered into a valid written agreement to sell a large parcel of land on which were located several residential properties to an investor for $10 million. The date set for closing in the agreement between the doctor and the investor had not yet arrived. The jurisdiction follows the common law doctrine of equitable conversion and there are no applicable statutes on the subject. The nurse petitioned the probate court for an order directing the doctor’s executor to transfer title to the large parcel of land to him. The resident opposed this petition, seeking an order directing the executor to proceed with the sale of the large parcel of land and to transfer the $10 million in proceeds to him.

How should the court decide?

The resident will take the purchase money as personalty because the nurse will have to honor the sale.

The court will sell the land pursuant to the contract and split the proceeds with the resident and the nurse.

The nurse will take the land because the contract became void upon the death of the seller.

The nurse will take the land because title to the land has become unmarketable.

A

The correct answer is: The resident will take the purchase money as personalty because the nurse will have to honor the sale.

Discussion of correct answer: If a seller dies after executing a valid contract for the sale of land, but before the closing date, legal title passes to the seller’s devisee but the devisee will have to honor the sale and the purchase money passes as personalty. Therefore, in this case, the resident will take the money from the sale of the land because he was devised the doctor’s personal property.

220
Q

A wealthy businessman had supported many charitable causes. His 2035 will left all his property, real and personal, to his niece, who was also his only surviving heir. The businessman died in 2045. To his niece’s disappointment, she found that the businessman’s largest piece of real estate, an office and urban housing complex, had been deeded in 2029 to a political group that raised funds to support term limits on members of Congress.

The deed stated that if the group was unsuccessful in obtaining passage of a constitutional amendment limiting the terms of members of Congress by 2049, then the property would go to a medical group that specialized in cataract surgery, “as long as some portion of the building is used for research into better methods of improving eyesight by surgical means.” If not so used while owned by the medical group, the property would go to the local senior citizens’ auxiliary to be used as a retirement home for indigent senior citizens. Real property conveyances in the state are governed by the common law. Assume that none of the grantee organizations qualify as charitable organizations under the law of the jurisdiction.

Which of the following most accurately describes the property interest of the medical group?

A springing executory interest, void under the Rule Against Perpetuities.

A shifting executory interest, void under the Rule Against Perpetuities.

A springing executory interest, valid under the Rule Against Perpetuities.

A shifting executory interest, valid under the Rule Against Perpetuities.

A

The correct answer is: A shifting executory interest, valid under the Rule Against Perpetuities.

Discussion of correct answer: Both shifting and springing uses pass title to a third party upon the occurrence of a designated event or condition, but a shifting use cuts off an interest conveyed to a grantee, whereas a springing use cuts off an interest retained by the grantor. Here, the medical group’s executory interest cuts off the political group’s estate, not the businessman’s. The medical group’s interest is not void under the Rule Against Perpetuities. It must vest, if at all, by 2049, which is less than 21 years from the date of the creation of the interest (2029). Note that it could be longer if a measuring life were involved (because the Rule Against Perpetuities allows an interest to vest or fail within 21 years of a measuring life), but that does not seem to be the case here.

221
Q

A noted paleontologist was working on an important dig in a remote area in the western United States. In the middle of their planned stay on location, her group was fortunate to find a nearly intact skeleton of an infant velociraptor. Anxious to determine if there were adult specimens nearby, the group redoubled their efforts to expand the dig. The paleontologist hired a local excavator to do the initial soil removal at the new location. The excavator had no experience at dinosaur digs and was not accustomed to working with many people near his backhoe. One afternoon, the excavator moved a pile of rocks to a ledge above the original dig. A large boulder later rolled off and hit a volunteer with the paleontologist’s group who joined through his hometown science museum. The volunteer was severely injured and will never walk normally again. The volunteer sued the paleontologist and the excavator in a jurisdiction which has adopted a contribution rule based on degrees of fault, but has retained joint and several liability. The jury found that the volunteer’s damages were $100,000, the volunteer was not negligent, the paleontologist was 20% at fault, and the excavator was 80% at fault in causing the volunteer’s injuries. After the excavator paid the volunteer $100,000, he sued the paleontologist for reimbursement.

What is the likely result?

A. The paleontologist will prevail and not have to reimburse the excavator, because the excavator’s fault was greater than the paleontologist’s was.

B. The excavator will recover 20% of the total damages, because he and the paleontologist are jointly liable for the volunteer’s injury.

C. The excavator will recover 50% of the total damages, because the paleontologist and the excavator are jointly and severally liable.

D. The excavator will recover 100% of the amount he paid to the volunteer, because excavation is an inherently dangerous activity.

A

B. The excavator will recover 20% of the total damages, because he and the paleontologist are jointly liable for the volunteer’s injury.

222
Q

A farmer dies owning 10 acres of farmland. The farmer’s 10 acres are part of what used to be a large tract of farmland, which is now almost completely surrounded by housing developments. In the last few years before his death, the farmer used the acres fronting a local road for his home and a small farmers’ market, where he sold produce grown on the “back” acres of his property. The farmer’s only access to the “back” acres of his property was a dirt road that ran from the “back” acres past the house and out to the public road.

The farmer leaves the property to his daughter, who has no interest in maintaining the farm. The daughter sells four acres of the land, including the old homestead, to a cook. The daughter sells the remaining six acres to a developer for the purpose of building townhomes. The cook plans to run a small home-based business on the property, supplying baked goods for her friend’s catering business.

The developer’s land has access to the public road only by means of the dirt road across the cook’s land. The developer had no problem moving construction equipment over the dirt road, but now seeks to pave the road in order to begin bringing in customers to look at their model homes. The cook refuses to allow the developer to pave the road, because she fears the noise and traffic will cause her cakes to fall.

If the developer requests court permission to pave the road, in whose favor should the court rule?

For the developer, because it has an implied easement and the right to repair and maintain it.

For the developer, because its pavement of the road is within the reasonable contemplation of the parties.

For the cook, unless the deed to the developer expressly granted an easement across the cook’s property.

For the cook, because the owner of the servient estate has the ultimate right to control the maintenance of an easement.

A

The correct answer is: For the developer, because its pavement of the road is within the reasonable contemplation of the parties.

Discussion of correct answer: The developer has an implied easement by necessity, since there is no other access to the property. (In this case, there was some use of the road before the properties were divided, so “strict” necessity is not required.) The owner of an implied easement has a limited right to upgrade the easement and generally may not develop or upgrade it beyond the reasonable contemplation of the parties at the time the property was divided. This choice is correct, because the daughter sold the land to the developer knowing that it would be used for townhomes and that the homeowners would need access to the public road. It was thus reasonably within the contemplation of the daughter and the developer that the road would eventually need to be paved.

223
Q

A landlord owns an upscale apartment building. An optician rented Apartment A for five years. The lease stated the rent would be paid in installments of $2,000 a month and prohibited subleases. The optician paid his rent on time for the first year of his lease, then got married and decided to move out. The optician signed an agreement with an editor for the editor to move into Apartment A and to pay the rent for the rest of the lease. The editor moved into Apartment A and paid the rent on time for the next three years. At that time, the editor lost his job, and vacated the apartment without paying the last two months’ rent on the lease.

Who is liable for the last two months’ rent?

The optician, because subleases were prohibited.

The optician is liable for $2,000, and the editor is liable for $2,000.

The optician and the editor are jointly and severally liable for $4,000.

The editor, because he agreed to pay the rent.

A

The correct answer is: The optician and the editor are jointly and severally liable for $4,000.

Discussion of correct answer: Absent an express provision in the lease prohibiting or restricting transfers, a tenant may freely transfer his leasehold interest in whole or part. If he makes a complete transfer of his entire remaining estate, he has made an assignment. If he retains any part of his leasehold interest, the transfer is a sublease. The original tenant is still liable for rent even after making an assignment, and assignees are bound to perform original covenants that run with the land. Here, the lease prohibited subleases. When such a restriction is present, it will be strictly construed. As such, while the optician will be prohibited from subleasing, he will be free to assign his interest. The facts state that the optician signed a five-year lease and that after one year he signed an agreement with the editor for the editor to move into the apartment and pay the rent for the remaining four years. As the optician transferred his entire remaining estate, he has created an assignment. Because the lease did not prohibit assignments, the optician’s assignment to the editor is valid. However, the duty of a tenant to perform those obligations for which he covenanted in his lease is not ended when he assigns away his leasehold. Thus, if the assignee fails to make the rental payments, the landlord can recover against the original tenant. Assignees are bound to perform the original covenants in the lease if either they assume them or the covenants run with the land. Covenants to pay rent always run with the land. As such, the optician and the editor are jointly and severally liable for the $4,000.

224
Q

A man obtained a loan from a bank to purchase a home. The man signed a loan agreement with the bank which stated that he waived the right to redeem the property in case of foreclosure proceedings. One year later, the man lost his job and could no longer pay his mortgage payments. After several months of nonpayment, the bank, initiated foreclosure proceedings. However, one month before the auction, the man found another job, and he immediately asked the bank for extra time to pay off his missed mortgage payments. The bank refused, arguing that he waived his right to redeem the property.

Is the bank’s assertion correct?

(A) No, because the right to redeem may not be waived at the time a mortgage agreement is executed.

(B) No, because a bank can never ask a mortgagor to waive his right to redeem.

(C) Yes, because the bank can at any time require the man to waive his right to redeem.

(D) Yes, because the man waived his right to redeem at the time he obtained the mortgage.

A

(A) No, because the right to redeem may not be waived at the time a mortgage agreement is executed.

225
Q

A pet owner had a cat that would viciously attack anyone who came to the owner’s home, so the owner kept the cat inside and away from people. The pet owner’s neighbors knew about the cat’s viciousness and avoided any contact with it. Late one night, some neighborhood children snuck into the pet owner’s yard without her permission. As a prank, one of the children opened the window and let the cat outside. The cat scratched and bit the child, causing injuries.

If the child’s parents sue the pet owner for their child’s injuries, will the parents prevail?

(A) No, because the pet owner exercised due care in trying to keep the cat away from other people.
(B) No, because the pet owner had no duty of care towards unknown trespassers.
(C) Yes, because the pet owner had a duty to warn of the presence of her dangerous cat.
(D) Yes, because the pet owner is strictly liable for keeping a cat with known vicious propensities.

A

(A) No, because the pet owner exercised due care in trying to keep the cat away from other people.

226
Q

A landlord entered into a written residential lease agreement with a tenant for a mountain cabin. According to the terms of the lease, the landlord agreed to make any necessary repairs to the cabin within a reasonable time except for ordinary wear and tear, which the tenant was responsible for. Several years later, part of the cabin’s roof collapsed during one of the mountain’s annual windstorms. The tenant called the landlord and informed him of the damage, but the landlord refused to make the repairs. A couple weeks later, a loosened piece of wood fell from the damaged area and landed on the tenant’s shoulder, causing him injury. The tenant sued the landlord.

For what claim(s), if any, may the tenant be awarded damages?

A For breach of the lease, but not for the tenant’s injuries.

B For the tenant’s injuries, but not for breach of the lease.

C For breach of the lease as well as for the tenant’s injuries.

D For neither breach of the lease nor the tenant’s injuries.

A

D For neither breach of the lease nor the tenant’s injuries.

Discussion of correct answer:The tenant is entitled to damages for neither breach of the lease nor the tenant’s injuries. The landlord here agreed only to make repairs for damages beyond ordinary wear and tear. There is nothing in the fact pattern to indicate that this windstorm was anything out of the ordinary; the area experienced them every year. As such, this is most likely ordinary wear and tear, for which the tenant, and not the landlord, was responsible. Because the tenant was responsible for making the repairs, the landlord will not be liable for the tenant’s injuries that resulted from the damage not being repaired, either.

Discussion of incorrect answers:

Incorrect. For breach of the lease, but not for the tenant’s injuries. In general, a tenant is entitled to money damages for a landlord’s breach of the lease. However, this was ordinary wear and tear, for which the tenant was responsible for making repairs. Thus, the landlord will not be liable for breach of the lease or the tenant’s injuries that resulted from the damage not being repaired.

Incorrect. For the tenant’s injuries, but not for breach of the lease. Generally, a landlord is not liable for the injuries of tenants or their guests caused by the conditions of the premises. However, a landlord is liable for such injuries if the landlord has contracted to repair the premises. In this case, though, the damage was caused by ordinary wear and tear, and so the tenant was responsible for repairing it, not the landlord. As such, the landlord will not be responsible for the tenant’s injuries.

Incorrect. For breach of the lease as well as for the tenant’s injuries. A tenant may seek money damages for a landlord’s breach of the lease. In addition, although the general rule is that a landlord is not responsible for the injuries of a tenant or the tenant’s guests caused by conditions of the premises, there exists an exception to this general rule where the landlord has covenanted to repair. However, in this case, the tenant is responsible for making repairs because this was ordinary wear and tear. As a result, the landlord will be liable for neither breach of the lease nor the tenant’s injuries.

227
Q

Owner 1 owed a mortgage obligation to the bank for a loan against her real estate. Owner 1 sold her real estate to Owner 2. Owner 2 and Owner 1 agreed that Owner 2 would assume the mortgage. Owner 2 did not enter into an agreement with the bank to assume the mortgage and defaulted on the mortgage.

Is Owner 1 responsible to the bank for the balance of the mortgage?

(D) No, because Owner 2 assumed the mortgage.

(C) No, because there was a novation.

(B) Yes, because there was a novation.

(A) Yes, because Owner 2 did not make the mortgage payments.

A

(A) Yes, because Owner 2 did not make the mortgage payments.

228
Q

A grantor conveyed land to an architect, who did not record. The grantor entered a lease with the architect and continued to live on the land. The grantor then conveyed the land to a baker, who paid value and recorded. The baker then conveyed the land to a carpenter for value, and the carpenter recorded. The carpenter and the architect were friends, and the carpenter was aware of the architect’s interest in the land.

The recording statute in the jurisdiction states: Every conveyance of real property is void as against a subsequent purchaser in good faith and for value whose conveyance is first duly recorded, unless the conveyance shall have been duly recorded prior to the record notice of action.

Who has a superior claim of title in the land?

A The architect, because his conveyance was first in time.
B The architect, because the carpenter had actual notice of the architect’s deed.
C The carpenter, because the baker was a bona fide purchaser without notice who recorded first.
D The carpenter, because the carpenter paid value for the land and recorded before the architect.

A

C The carpenter, because the baker was a bona fide purchaser without notice who recorded first.

229
Q

A teacher buys a new townhouse. A few months later, she notices that the doors in her house are not closing properly. When she calls the developer from which she bought the house, the developer’s staff sends someone over to sand the door edges. A few months later, when the same problem occurs, she calls in a house inspector, who determines that the foundation to the house was improperly laid and that her house is sinking. The foundation can be shored up at a cost of $10,000.

If the sale documents contained no express warranties, does the teacher have a cause of action?

Yes, because the house was not merchantable.

Yes, because she bought a new house from the builder.

No, because the implied warranty of quality only applies to commercial premises.

No, because the house was not custom-made.

A

The correct answer is: Yes, because she bought a new house from the builder.

Discussion of correct answer: The townhouse here is new, so the implied warranty of quality applies in any jurisdiction. The implied warranty of quality requires houses not to have significant latent defects caused by the builder or remodeler’s poor workmanship. It requires that the homeowner discover the defects within a reasonable time, which seems to be the case here. Note that some jurisdictions apply this warranty to resold homes as well.

230
Q

Harvey and Wanda were engaged. Not wanting to wait for the actual marriage, Harvey’s uncle gave the couple a lovely vacation cabin on an acre of land in the mountains, conveying to them as “joint tenants with right of survivorship.” Harvey and Wanda were subsequently married. Unknown to Harvey, Wanda was a compulsive gambler, and she soon was in debt on various credit cards and loan accounts. Not wanting to reveal her obsession to Harvey, Wanda secretly quitclaimed her interest in the mountain vacation property to Larry in exchange for half of the property’s market value. Shortly thereafter, Wanda was killed by a drunk driver as she
drove at night to an illegal Bingo game. Wanda’s valid will left all her property, real and personal, to her sister, Naomi. No applicable statute has altered the common law in any relevant way.

Who owns the mountain property?

(A) Harvey.

(B) Harvey and Naomi.

(C) Harvey and Larry, as joint tenants.

(D) Harvey and Larry, as tenants in common.

A

(D) Harvey and Larry, as tenants in common.

231
Q

A landlord leased an office building to a tenant for 10 years. The tenant always paid his rent on time. After four years, the tenant assigned his interest in the building to a medical practice. The medical practice paid the rent on time for the first six months, but then stopped paying the rent. After being in the office building for a year, the medical practice decided to downsize and assigned the lease to a publisher. The publisher never paid any rent. One year later, the landlord sued the tenant, medical practice, and publisher for the accrued unpaid rent.

From whom should the landlord recover damages?

A From the tenant or the medical practice for six months’ rent, and from the tenant or the publisher for one year’s rent.

B From the tenant or the medical practice for six months’ rent, and from the tenant, medical practice, or publisher for one year’s rent.

C Only from the tenant for six months’ rent, and only from the publisher for one year’s rent.

D Only from the tenant, for all of the unpaid rent.

A

The correct answer is:From the tenant or the medical practice for six months’ rent, and from the tenant or the publisher for one year’s rent.

Discussion of correct answer:Where a tenant assigns all of his leasehold interest to an assignee, such assignment does not release the original tenant from his contract obligations to the landlord under the terms of the lease absent a novation, which is a separate agreement discharging a contractual party from liability. This is true even when the assignee subsequently reassigns the lease to a new assignee. As such, the tenant will be liable for any unpaid rent. However, the medical practice will only be liable for the unpaid rent that accrued during its one year of possession. When a lease is assigned to a new tenant, the assignee comes into privity of estate with the landlord, but not privity of contract. If the assignee then reassigns the property to someone else, the middle tenant is no longer in privity with the landlord at all, and is therefore not liable for any unpaid rent of the subsequent assignee unless the assignor expressly assumed the duty to pay rent. Therefore, the medical practice will not be liable for the publisher’s unpaid rent; only the original tenant and the publisher will be liable for this amount.

Discussion of incorrect answers:

Incorrect. From the tenant or the medical practice for six months’ rent, and from the tenant, medical practice, or publisher for one year’s rent. When an assignee subsequently reassigns the leasehold interest to a new tenant, the assignee will no longer be in privity with the landlord, because the original tenant will be in privity of contract and the current tenant will be the party in privity of estate. As such, unless there was an express assumption of the duty to pay rent, an assignee who reassigns the property will not be liable for any unpaid rent owed by the subsequent assignee. This answer choice is therefore incorrect, because it states that the medical practice will be liable for the publisher’s unpaid rent.

Incorrect. Only from the tenant for six months’ rent, and only from the publisher for one year’s rent. This answer choice is incorrect for several reasons. First, the tenant, as the original tenant to the lease, will be liable for all of the unpaid rent absent a novation discharging him from liability. This means that the tenant will be liable for both the six months’ rent and the one year’s rent. Second, even though the medical practice reassigned the lease to the publisher, they will still be liable for any unpaid rent that accrued during their tenancy. Thus, the tenant and medical practice will both be liable for the six months’ rent that the medical practice failed to pay.

Incorrect. Only from the tenant, for all of the unpaid rent. It is true that, absent a novation, the tenant will be liable for all of the unpaid rent. However, this answer choice fails to take into account the fact that the medical practice will also be liable for the six months’ rent that accrued during their tenancy, and the publisher will be liable for the one year’s unpaid rent that they failed to pay.

232
Q

An investor purchased Blackacre from the owner for $250,000, paying $50,000 as a down payment, and issuing a mortgage on Blackacre to secure payment of the remaining $200,000 plus interest over a period of 10 years. The mortgage arrangement called for the investor to pay $2,000 per month, with $800 allocated to interest and the rest to principal. Two years later the investor died, leaving a will devising Blackacre “to my wife for life, remainder to my daughter.”

Between the wife and the daughter, how should the burden for payment be allocated?

The daughter is obligated to pay the interest, and the wife is obligated to pay the principal.

The daughter is obligated to pay the principal, and the wife is obligated pay the interest.

The daughter is obligated to pay both the principal and the interest.

The wife is obligated to pay both the principal and the interest.

A

The correct answer is: The daughter is obligated to pay the principal, and the wife is obligated pay the interest.

Discussion of correct answer: A life tenant is generally obligated to pay current expenses relating to the property, such as mortgage interest. The remainderman is responsible for repaying mortgage principal. Thus, the wife as life tenant must pay the mortgage interest and the daughter, as the remainderman, must repay the mortgage principal.

233
Q

The Smith family had a lakeside cottage. Mr. and Mrs. Smith held the cottage as tenants by the entirety. Three months before Mr. Smith died, he conveyed the property “to my son Alvin for life, then to my daughters Betty, Charlotte and Diane.” Six months after Mr. Smith died, Mrs. Smith died. Mrs. Smith’s valid will held a provision that read “I leave the cottage to my children Alvin, Betty, Charlotte, and Diane as joint tenants with the full right of survivorship.” Betty died leaving two daughters, Emily and Fay. After Betty died, Alvin mortgaged the cottage for $100,000. Diane then died leaving three sons, Gary, Herb, and Irvin.

Which of the following is correct?

(A) Alvin holds a life estate in the cottage.

(B) Emily and Fay are tenants in common.

(C) Gary, Herb, and Irvin are tenants in common.

(D) Charlotte owns 50% and Alvin owns 50% as joint tenants.

A

(D) Charlotte owns 50% and Alvin owns 50% as joint tenants.

234
Q

A landowner owned a piece of property, which he intentionally left vacant. The owner then conveyed the property to his friend “for so long as no buildings are built on the property, then to [an environmental group].” Two years later, the friend built a small shed on the property. The environmental group informed the friend this action violated the condition and the property belonged to the group.

Is the environmental group entitled to the property?

(A) No, the shifting executory interest violates the Rule Against Perpetuities, so the property reverts to the original land owner.

(B) No, the shifting executory interest violates the Rule Against Perpetuities, so the friend has a fee interest in the property.

(C) Yes, because the charity-to-charity rule makes the shifting interest valid despite the Rule Against Perpetuities.

(D) Yes, because a charitable trust is not subject to the Rule Against Perpetuities.

A

(B) No, the shifting executory interest violates the Rule Against Perpetuities, so the friend has a fee interest in the property.

235
Q

A man bought a house from a woman. The man bought the house subject to the mortgage and took over the mortgage payments. The man subsequently sold the house to an investor, who bought the house by an installment land-sale contract that made no mention of the mortgage. After receiving just two of the 20 agreed upon payments under the installment land-sale contract, the man stopped making payments on the mortgage. The mortgagee initiated foreclosure proceedings.

What will happen to the house and the debt?

The mortgagee is estopped from foreclosing on the property, because the last purchaser is a bona fide purchaser.

The mortgagee may foreclose on the property and hold the investor personally liable for any debt remaining after the foreclosure sale.

The mortgagee may foreclose on the property and hold the man personally liable for any debt remaining after the foreclosure sale.

The mortgagee may foreclose on the property and hold the woman personally liable for any debt remaining after the foreclosure sale.

A

The correct answer is: The mortgagee may foreclose on the property and hold the woman personally liable for any debt remaining after the foreclosure sale.

Discussion of correct answer: When property is sold “subject to the mortgage,” as opposed to “assuming the mortgage,” the mortgagee may foreclose on the property, but liability for any remaining debt after the foreclosure sale remains with the original mortgagor. Here, the woman sold the house to the man subject to the mortgage. The man subsequently sold the house to the investor by an installment land-sale contract that was silent as to the mortgage. Given these terms, the mortgagee will be permitted to foreclose and the original mortgagor, the woman, will be personally liable for any debt remaining after the foreclosure sale. The installment land-sale contract entered into by the last purchaser does not have any effect on the existing mortgage on the property.

236
Q

An architect and a developer were long-time companions who had summered in the developer’s cottage on the seacoast for 30 years. When the developer was confined to an assisted living facility because of poor health, she conveyed the deed to the cottage to the architect, who continued to summer there without her friend. The developer’s health continued to decline, and she soon died, leaving a will in which she bequeathed her personal property to the architect and devised her summer cottage to a favorite nephew who had spent time in the cottage with the developer and architect in the past. When the architect learned of the devise to the nephew, she immediately recorded the deed conveying the cottage to her.

Assume that the jurisdiction in which the summer cottage is located has the following statute: “A conveyance of an estate in land, other than a lease for less than one year, shall not be valid as against any subsequent purchaser for value, except such persons having notice of it, unless the conveyance is properly recorded.”

If the architect files an action to quiet title to the cottage, what is the most likely outcome of the suit?

The architect will prevail, because the nephew had knowledge of her residence in the cottage.

The architect will prevail, because she acquired title to the cottage before the nephew.

The nephew will prevail, because the architect failed to record before the conveyance by will to him.

The nephew will prevail, because the architect acquired title to the cottage by gift.

A

The correct answer is: The architect will prevail, because she acquired title to the cottage before the nephew.

Discussion of correct answer: A grantee who acquires title to real property by devise is not protected by a notice recording act against prior claimants who failed to record. As devisee of the cottage (and not a bona fide purchaser), the nephew cannot invoke the recording act to defeat the architect’s claim. With the recording act inapplicable, the court will apply the common law rule of “first in time, first in right” to find the architect the owner of the summer cottage.

237
Q

A man owned Blackacre, a 10-acre parcel improved with a one-story residence. The man contracted with a woman to perform gardening services on Blackacre for four years. Two years after forming the contract with the woman, the man decided he did not need a gardener anymore and informed the woman that her services would no longer be needed. The woman then sued the man for breach of contract. While this suit was pending, the man borrowed $100,000 from a bank and granted a mortgage on Blackacre to the bank to secure the loan. The bank did not record the mortgage. The woman subsequently prevailed in her suit against the man, and the judgment became a lien against Blackacre.

The jurisdiction has the following statute:

“A conveyance of an estate in land shall not be valid as against any subsequent purchaser for value, except such persons having notice of it, unless the conveyance is properly recorded.”

Can the woman enforce her lien against Blackacre?

A Yes, because a judgment creditor is treated as a “subsequent purchaser for value” under a recording statute of this type.
B Yes, because the bank did not record the mortgage.
C No, because the bank’s mortgage lien on Blackacre arose before the woman’s judgment lien attached.
D No, because as a judgment creditor, the woman is not considered a “subsequent purchaser for value” under a recording statute of this type.

A

D No, because as a judgment creditor, the woman is not considered a “subsequent purchaser for value” under a recording statute of this type.

238
Q

Oliver conveyed his log cabin “to Anne for life, remainder to Anne’s widower for life, remainder to Anne’s children living at the death of Anne’s widower.”

Which of the following is most accurate?

Anne’s life estate is presently possessory, and therefore, the grant is not subject to the Rule Against Perpetuities.

The remainder for life in Anne’s widower is contingent, because there cannot be a widower of a living person, and therefore, the grant is void.

The remainder in Anne’s children living at the death of Anne’s widower is contingent, because there is a condition precedent to vesting, and therefore, the grant is not subject to the Rule Against Perpetuities.

Anne retains a life estate, Anne’s widower will take for life after Anne dies if he survives Anne, and Oliver retains a reversion, because the grant is partially subject to the Rule Against Perpetuities.

A

The correct answer is: Anne retains a life estate, Anne’s widower will take for life after Anne dies if he survives Anne, and Oliver retains a reversion, because the grant is partially subject to the Rule Against Perpetuities.

Discussion of correct answer: Anne’s life estate is presently possessory, and therefore is not subject to the Rule Against Perpetuities. The remainder for life in Anne’s widower is contingent because there cannot be a widower of a living person, and therefore is subject to the Rule Against Perpetuities. The remainder in Anne’s children living at the death of A’s widow is contingent because there is a condition precedent to vesting, and therefore is subject to the Rule Against Perpetuities. Oliver and Anne are express lives in being. Anne’s widower is not a life in being because he has not yet been identified. Anne’s children, even if some are already born, may not be lives in being because their class is open. Therefore, Oliver and Anne are the only possible measuring lives. Anne’s widower will take (or not take) at Anne’s death; hence, her interest is valid. However, Anne’s children may well take more than 21 years after Oliver and Anne die; hence, their interest violates the Rule Against Perpetuities. Anne retains a life estate. Anne’s widower will take for life after Anne dies if he survives Anne. Because the interest in Anne’s children is void, Oliver retains a reversion, which will vest when Anne’s widower dies. Thus, this answer is correct.

239
Q

A man knew that the brakes on his car were in poor shape. The brakes had been squeaking for months and, over the past few weeks, it had become harder and harder to stop the car. Nevertheless, the man continued driving. One day, as he was driving with proper attention and within the speed limit, a six-year-old boy suddenly jumped out from behind a parked car right in front of the man’s car. The man jammed on his brakes but was unable to stop in time. As a consequence, the man ended up running into (and over) the boy, causing him personal injuries. It will be stipulated at trial that the boy jumped out from behind the parked car so suddenly and so close to the man’s car that he would have suffered the same injuries even if the man’s brakes been properly adjusted.

If the boy’s parents sue the man for personal injuries, will they likely recover?

A. No, because the man’s breach of duty in not keeping his brakes properly serviced was not the cause-in-fact of the boy’s injuries.

B. No, because the man’s breach of duty in not keeping his brakes properly serviced was not the proximate cause of the boy’s injuries.

C. Yes, because the man should reasonably have foreseen that he might cause an accident by not being able to stop in time.

D. Yes, because the man’s breach of duty in not keeping his brakes properly serviced was the proximate cause of the boy’s injuries.

A

A. No, because the man’s breach of duty in not keeping his brakes properly serviced was not the cause-in-fact of the boy’s injuries.

240
Q

A property developer and a real estate broker entered into a contract of sale, with the developer agreeing to sell his cabin to the broker for $100,000. At closing, the developer gave the broker a general warranty deed, and the broker handed the developer a cashier’s check for the full purchase price. The broker promptly recorded the deed, then sold the property to his dentist, also by general warranty deed. Many years later, the dentist was mending the fence that encircles the land when he found an eviction notice from the bank, which was commencing foreclosure proceedings against the developer’s father for defaulting on a mortgage to the property. The dentist then spoke to the bank and discovered that the developer had indeed sold the property to his own father one week before completing the transaction with the real estate broker.

Can the dentist recover from the developer?

Yes, because the dentist is a bona fide purchaser for value.

Yes, because the covenant for quiet enjoyment runs with the land.

No, because only the real estate broker could recover from the developer.

No, because the covenant against encumbrances is a present covenant.

A

The correct answer is: Yes, because the covenant for quiet enjoyment runs with the land.

Discussion of correct answer: A general warranty deed includes the covenant for quiet enjoyment. A covenant for quiet enjoyment is a covenant by the grantor that the grantee will not be disturbed in his possession or enjoyment of the property by a third party’s lawful claim of title. The covenant for quiet enjoyment is continuous in nature and may be enforced by remote parties on the basis of privity of estate. In this case, the bank is making a lawful claim of title. The benefit of the covenant for quiet enjoyment “runs with the land,” giving the dentist a basis for recovery against the developer.

241
Q

A state has the following recording statute in effect: “No conveyance is good against a subsequent purchaser for a valuable consideration and without notice, unless the same be recorded prior to subsequent purchase.” A landowner owned a 20-acre tract of unimproved land in fee simple. On May 1, the landowner sold the property to a buyer for $40,000 under a quitclaim deed. The buyer did not record the deed. On May 5, the landowner died, leaving a valid will by which he devised the property to his son. On August 1, the son conveyed the property to an investor by warranty deed for $75,000. When the investor acquired title, he had no actual knowledge of the buyer’s deed (which was still unrecorded). On August 10, the buyer recorded his deed. The investor did not record his deed until August 15. In an appropriate action to quiet title, in which all interested parties have been joined, who owns title to the property?

(A)The buyer, because his deed preceded the investor’s deed.

(B) The buyer, because his deed was recorded prior to the investor’s deed.

(C) The investor, because he is protected by the recording statute.

(D) The investor, because he took title by warranty deed and the buyer took title by quitclaim deed.

A

(C) The investor, because he is protected by the recording statute.

242
Q

A developer purchased a subdivision, which he divided into 10 comparably-sized lots. Each lot was listed for sale between $1,000 and $10,000. An investor wished to purchase a lot, but couldn’t decide between Lots A and B, which were each priced at $2,000. The investor and the developer therefore struck a deal in which the investor paid the developer $2,000 and the parties executed a deed that was complete except that the box labeled “Lot Number” was left blank. The developer told the investor to inspect the lots one more time, then fill in the blank with either Lot A or Lot B and record the deed. After revisiting the subdivision, the investor decided he preferred a third lot, Lot C, which was selling for $5,000. Without a second thought, the investor wrote “Lot C” in the blank on the deed, which he then properly recorded.

The investor quickly resold Lot C for $4,000 to a young woman who properly recorded the deed. The woman, who had long been interested in the subdivision, knew that the lot had previously been listed for $5,000, but she did not know the specifics of the transaction between the developer and the investor. She had made a thorough search of the land title records and had found the investor’s title to Lot C to be good, marketable, and free of encumbrances. When the developer discovered what had happened, he brought an action against the woman to recover title to Lot C.

If the woman prevails, it will likely be for which of the following reasons?

The woman is a bona fide purchaser and the developer is estopped from recovering title from her.

The developer is barred by the “clean hands” doctrine from relief from the court.

Once the woman properly recorded the deed, a court is precluded from finding the deed invalid.

The Statute of Frauds prevents the introduction of evidence of the oral agreement between the developer and the investor in the dispute between developer and the woman.

A

The correct answer is: The woman is a bona fide purchaser and the developer is estopped from recovering title from her.

Discussion of correct answer: Once the developer actually gave the deed to the investor, there was a presumption that a valid delivery had occurred, and an argument can be made that the developer could not restrict the agency that he had given the investor to record the deed. Moreover, even if the developer could prevail against the investor, the woman reasonably relied on the deed and the land records and appears to be a bona fide purchaser and, therefore, is protected by the law. The developer would likely be estopped from pursuing a claim against her.

243
Q

Several farmers in the drought-ridden state of Arizona jointly decided to sell their land to a developer, negotiating a good price, and extracting a promise from her to build only upscale ranch-style homes on lots no smaller than 10 acres each. After the sale was consummated, the developer proceeded to develop and sell large homes on the lots, including in each deed a restrictive covenant by which the grantee promised not to subdivide the purchased parcel. Before the developer had completed constructing homes on all of the lots, a buyer offered to purchase the last five 10-acre lots, unimproved.

The developer sold the last lots to the buyer subject to the same restrictive covenant contained in the other deeds. The buyer immediately resold the lots to his father’s construction company. However, the deed did not contain the restrictive covenant. The construction company then sought and obtained a zoning change and construction permit for the development of 200 condominiums on small lots to be subdivided from the original five lots. The first purchaser of a home from the developer brought an action against the construction company to prevent the subdivision as a violation of the restrictive covenant.

What is the likely outcome?

Judgment for the first purchaser, because county’s alternation of the zoning classification violated the vested property rights of the previous purchasers of the developer’s lots.

Judgment for the first purchaser, because the restrictive covenant is enforceable against the construction company.

Judgment for the construction company, because enforcement of the restrictive covenant would constitute an unreasonable restraint on alienation.

Judgment for the construction company, because the county may rezone regardless of vested property rights.

A

The correct answer is: Judgment for the first purchaser, because the restrictive covenant is enforceable against the construction company.

Discussion of correct answer: A covenant that runs with the land is a promise that runs with the land. In that covenant, the promisor promises to do or refrain from doing something on his land. Unlike easements, most running covenants are negative in nature, as they require the parties not to do something. The restrictive covenant here meets the requirements to be enforceable and to run with the land: it touches and concerns the land in that it restricts style of structures and size of lots, there was intent and notice that the restrictions would run with the land as shown by the deeds, and there is privity. Horizontal privity is the relationship that exists between the original covenantor and covenantee; here, the developer and the buyer. Vertical privity is the relationship that exists between the original party to the running covenant and the successor in interest to the original party. A plaintiff must show the successor “stepped into the shoes” of the original party to show vertical privity. If for some reason the covenant is unenforceable, the original restriction would be enforceable as an implied reciprocal servitude, since the lots purchased by the construction company were part of a common development plan that put the construction company on notice. As such, this is the best response.

244
Q

A dentist owned an undeveloped tract of land, which he sold for fair market value and conveyed by general warranty deed to a professor, who forgot to record his deed. Soon thereafter, the dentist purported to convey the land to his lawyer as a wedding gift, which the lawyer promptly recorded. Soon after that, the professor finally recorded his deed.

The lawyer then purported to sell his interest in the property to a sailor by warranty deed, which the sailor immediately recorded. The sailor had no actual knowledge of the professor’s interest in the property. When the sailor attempted to take possession of the land, the professor filed an action to quiet title.

The applicable recording act reads as follows: “A conveyance of land shall not be valid as against any subsequent purchaser for value, except such person having notice of it, unless the conveyance is properly recorded.”

Which of the following best supports a claim of title for the sailor?

Because the lawyer recorded earlier than did the professor, the sailor’s title is paramount.

The professor’s prior recorded deed is deemed to be outside the chain of title to the property so far as the sailor is concerned.

The lawyer’s deed, being subsequent to the professor’s, has seniority.

Because the lawyer had no actual or constructive notice of the professor’s interest, the sailor’s title is paramount under the “shelter rule.”

A

The correct answer is: The professor’s prior recorded deed is deemed to be outside the chain of title to the property so far as the sailor is concerned.

Discussion of correct answer: In the majority of jurisdictions (including the one in this fact pattern), the professor’s deed is considered a “deed recorded late” and is outside the chain of title. When the sailor searched the title records, the lawyer would appear as record holder of title, having obtained ownership from the dentist and having not conveyed to another person. The dentist-professor conveyance would not have appeared in the records until after the dentist-lawyer conveyance, meaning the sailor would have no constructive notice of the conveyance. The sailor had no actual notice of the professor’s interest, and because the land was vacant, no basis for inquiry notice. Having no actual, inquiry, or constructive notice of the professor’s claims, the sailor is a bona fide purchaser and should prevail in the litigation over title to the land.

245
Q

A landlord owned several apartment buildings surrounding a college campus, which he rented to students. After several bad experiences, the landlord now includes in all leases a prohibition against assignments. A sophomore rented one of the apartments for a year, beginning July 1, for $1,200 per month. Unless proper notice was given, the lease would renew automatically each year. During the fall semester, the sophomore was accepted into a program allowing her to study abroad in the spring. Not wanting to give up her apartment due to a housing shortage, the sophomore agreed to let her friend take over the lease for the months of January through May. The friend moved into the apartment, but never paid any rent to the landlord. When the sophomore returned in June, she was greeted by a very angry landlord, who demanded the previous five months’ unpaid rent.

From whom can the landlord recover the rent owed?

A The sophomore, because assignments were prohibited under the terms of the lease.

B The sophomore, because the friend was a subtenant.

C The sophomore and the friend, because they are jointly and severally liable.

D The friend, because he agreed to take over the lease.

A

B The sophomore, because the friend was a subtenant.

The correct answer is:The sophomore, because the friend was a subtenant.

Discussion of correct answer:Absent an express provision in the lease prohibiting or restricting transfers, a tenant may freely transfer his leasehold interest in whole or part. If he makes a complete transfer of his entire remaining estate, he has made an assignment. Where the tenant retains any part of the leasehold interest, the transfer is a sublease. The tenant-sublessor remains both in privity of estate and in privity of contract with the landlord, and thus continues to be obligated to pay the rent. A sublease creates no legal relationship between the landlord and the subtenant, so the sublessee is not liable to the landlord. As such, the sublessee is not liable to the landlord to pay rent. Here, the sophomore only conveyed her lease interest to her friend for five months. As such, this was a sublease, not an assignment. Though the lease prohibited assignments, it did not prohibit subleases, making the sublease valid. Accordingly, the sophomore is liable to the landlord for the unpaid rent; the landlord could not collect directly from the friend.

Discussion of incorrect answers:

Incorrect. The sophomore, because assignments were prohibited under the terms of the lease. If a lease does not expressly prohibit transfers, a tenant may freely transfer his leasehold interest in whole or part. A complete transfer of one’s entire remaining estate is an assignment; where the tenant retains any part of the leasehold interest, the transfer is a sublease. Restrictions on assignment are strictly construed. As such, given the restriction, the sophomore was prohibited from assigning her interest, but was free to sublease the apartment. Because she only conveyed her interest for five months to her friend, this was a sublease, not an assignment, and therefore a valid transfer. However, a sublease creates no legal relationship between the landlord and the subtenant. Because the friend was not in privity with the landlord, he does not share the liability.

Incorrect. The sophomore and the friend, because they are jointly and severally liable. It is not true that the sophomore and her friend are jointly and severally liable. The sophomore here subleased the apartment to her friend for five months. However, a sublease creates no legal relationship between the landlord and the subtenant. While the friend will owe the rent to the sophomore, the sophomore is nevertheless the only one liable to the landlord for the rent.

Incorrect. The friend, because he agreed to take over the lease. A sublease creates no legal relationship between the landlord and the subtenant. The friend will be liable to the sophomore for the unpaid rent, but only the sophomore will be directly liable to the landlord.

246
Q

On December 15, a landlord and tenant entered into a lease for a period of one year, beginning at the start of the calendar year, with rent to be paid on the first of each month. One month before the end of the lease term, the tenant called the landlord and the two agreed to renew the lease for another year at the same terms.

As of January 1, what type of tenancy do the landlord and tenant have?

(A) A term of years.

(B) A periodic tenancy.

(C) A tenancy at-will.

(D) A tenancy at sufferance.

A

(A) A term of years.

The correct answer is:A term of years.

Discussion of correct answer:A tenancy for a term of years has a definite beginning and end, and no notice is required to terminate the lease. The initial lease was such a tenancy, terminating at the end of the calendar year with no notice required. When the landlord and tenant agreed to another one-year lease on the same terms, the parties either extended the duration of the lease or created another term of years for the following calendar year. Either way, the lease remained one for a term of years, which will terminate at the end of the lease with no notice required.

Discussion of incorrect answers:

Incorrect. A periodic tenancy. This answer choice is tempting, because it may appear as if the parties here may have created a periodic tenancy for a period of one year, which simply renewed at the end of the lease term when neither party gave notice to terminate the lease. However, the parties here actually have a lease for a term of years, which expires at the end of the lease term without any notice required. When the landlord and tenant agreed to renew the lease for another year, they did not convert the tenancy to a periodic tenancy, but instead, either extended to lease term for another year or entered into a new lease for a term of years on the same lease terms. In either case, the current arrangement is still a lease for a term of years, and not a periodic tenancy.

Incorrect. A tenancy at-will. An at-will tenancy has no fixed duration and lasts only as long as the landlord and tenant desire. This is not the case here, as the lease has a specific duration and will terminate at the end of the time period.

Incorrect. A tenancy at sufferance. When a tenant remains in possession of the property (“holds over”) after the expiration of the lease term, the tenant is considered a tenant at sufferance, and the landlord will be entitled to recover the reasonable rental value for the holdover period. However, this is not the case here, because the tenant is not remaining in possession after the duration of the lease term. Instead, the landlord and tenant have agreed to a lease for another year.

247
Q

A mother gave title to a waterfront bungalow to her twin daughters for their 30th birthday. The deed gave the twins joint ownership with rights of survival. The twins immediately moved into the small bungalow and founded a religious cult, which met every week at the beachfront house. Furious, the mother demanded that they return the property. Equally furious, the twins ripped up the deed and threw the shreds at their mother. They then vacated the bungalow and moved to Mexico.

In order to protect what she felt was now her property and to prevent the twins from changing their minds, the mother placed a fence around the property. However, the bungalow remained empty, and the land was not used. After not hearing from the girls for many years, the mother decided to sell the property. She sold it for a great deal of money to an investor, who promptly recorded her deed. Soon thereafter, a son of one of the twins, who was the sole heir to their estates, decided to move into the bungalow. When he found the investor comfortably ensconced in what he thought was his house, he brought an ejection action. To his surprise, he lost, and the investor was declared the owner.

Which of the following is the best explanation for the court’s ruling?

A The court found that the twins had abandoned the property, causing title to revert to their mother.

B By destroying the deed, the twin’s rejected the gift from their mother, thereby reconveying title to their mother.

C The investor’s claim is superior to the son’s, because the investor paid value for the property.

D The twins failed to record their deed, and the investor had no other notice of their interest.

A

D The twins failed to record their deed, and the investor had no other notice of their interest.

248
Q

On August 1, a student rented an apartment on a month-to-month basis in a residential area near a local college. The lease required 30 days’ notice of intent not to renew, with rent payable on the first day of each month. The student made timely rental payments through June 1. On June 10 the landlord informed the student in writing that his lease would not be renewed, and he had 30 days to vacate the premises. The student, who was still in possession, paid his rent on July 1.

According to the terms of the lease, the student will need to be out of the apartment by which of the following dates?

A. July 1.

B. July 10.

C. July 31.

D. August 1.

A

C. July 31.

249
Q

In 1985, a seller conveyed a ranch to a buyer for $750,000. The parties included in the deed a provision reserving to the seller an option to buy back the property in the future. The provision granted the seller, his heirs or assigns, a right of first refusal to repurchase the property in the event that the buyer or his successors decided to sell the ranch for 25 years (until 2010).

According to the terms of the buyback arrangement, the seller paid the buyer $100,000 consideration for the option. The deed included a complicated purchase price formula in the event that the option was exercised. The buyback purchase price was to be based on a national real estate measurement scheme.

In February 2009, the buyer wished to sell the ranch. The seller indicated that he wished to exercise his right of first refusal. Based on the national measurement scheme recited in the deed, the seller could purchase the property for $850,000. However, the fair market value for the ranch at the time was $2,000,000. As a result, the buyer refused to sell the property to the seller based on the option price. The seller filed suit for specific performance seeking to exercise his option under the terms set forth in the original deed.

Will the seller prevail?

No, because the option is unenforceable as violative of the Rule Against Perpetuities.

No, because the option is unenforceable as an illegal restraint against alienation.

Yes, because the seller has an equitable right to redeem and reclaim title based on enforcement of the option provision.

Yes, because the seller paid valuable consideration for the option arrangement.

A

The correct answer is: No, because the option is unenforceable as violative of the Rule Against Perpetuities.

Discussion of correct answer: The Rule Against Perpetuities states that no interest in property is valid unless it must vest, if at all, not later than 21 years after some life in being (the measuring life) at the creation of the interest. If there is any possibility that the interest might vest more than 21 years after a life in being, it is void. The application of the Rule Against Perpetuities has raised persistent problems with options and rights of first refusal. Cases that apply the rule generally hold that an option is similar to a springing executory interest, and thus is void unless it may be exercised no later than 21 years after some life in being at its creation. A human life (i.e., a measuring life) is usually not involved in determining an option’s duration. Thus, in most cases, an option that can be exercised more than 21 years after the date of its creation will be void. In this question, the seller’s option is for 25 years, and will therefore be void because it can be exercised more than 21 years after the date of its creation.

250
Q

An investor purchased a 100-acre piece of land 30 years ago. Ten years later, the investor built a house on a different residential property and moved in. The investor instructed an accountant to pay all property taxes on both the 100-acre piece of land and the residence, and to inspect the 100-acre piece of land once every five years. Shortly thereafter, a trespasser cleared a five-acre section in the northwest corner of the 100-acre piece of land. The trespasser built a house, a barn, and a store and posted a sign, “Welcome to my town.” The trespasser never entered the other 95 acres of the land. Twenty years after the trespasser entered the property, the investor sold the 100 acres to a developer. The developer announced plans to build a mall and proceeded to clear the remainder of the property. The trespasser sought an injunction to stop the developer. The relevant statute of limitations is 10 years. Will the trespasser succeed in proving that the trespasser has acquired title to the whole of the 100-acre property?

(A) No, because the property was inspected every five years.

(B) No, because the trespasser acquired title to only the five acres of land that he had cleared and used.

(C) Yes, because the trespasser obtained title to all of the 100-acre property by adverse possession.

(D) Yes, because the developer should have known the trespasser was in possession of part of the property.

A

(B) No, because the trespasser acquired title to only the five acres of land that he had cleared and used.

251
Q

A scientist was transporting a large quantity of highly flammable petroleum in his car. While driving along a city street with the glass jugs of petroleum, he lost control of the vehicle. The car flipped over, spilling the petroleum onto the sidewalk. Moments later, a doctor ran over to render medical assistance. As he approached the car, the doctor slipped on the petroleum and fell, fracturing his ankle. The doctor brought a claim against the scientist based on strict liability.

Will the doctor prevail?

(A) No, because it was unforeseeable that the doctor’s injury would result from the type of harm threatened.
(B) No, because the doctor assumed the risk by voluntarily acting as a Good Samaritan.
(C) Yes, because the scientist was engaged in an abnormally dangerous activity by transporting highly flammable petroleum in his car.
(D) Yes, because the transportation of flammable petroleum in glass jugs necessarily involves a high degree of risk of serious harm.

A

(A) No, because it was unforeseeable that the doctor’s injury would result from the type of harm threatened.

252
Q

The owner of a limousine business decided to retire and convey his building to his daughter. The owner hoped that his daughter would take over the business and settle down. The owner conveyed the building by quit claim deed for the sum of one dollar “to my daughter and her heirs for so long as the premises are used for a limousine business.” As soon as the owner retired and moved to California, his daughter converted the building to a dance club and abandoned the limousine business.

Did the estate automatically revert to the owner?

(A) Yes, because the estate was defeasible.

(B) Yes, because the estate was a fee simple determinable with the possibility of reverter.

(C) No, because the estate was a fee simple determinable.

(D) No, because the estate was a fee simple subject to condition subsequent.

A

(B) Yes, because the estate was a fee simple determinable with the possibility of reverter.

253
Q

In order to finance construction of a new, 52,000-seat stadium, the owner of a baseball team borrowed $25,000,000 from a trust company, which secured the debt with a first mortgage on the facility. The trust company recorded the mortgage as required by statute. However, despite the new, state-of-the-art stadium, the team played miserably and finished in last place. On account of their last place finish, the team had the lowest attendance in the league, averaging only 6,500 fans per game.

In an attempt to recoup his losses due to the low attendance, the owner negotiated a contract with a professional football team to allow them to play their league games at the stadium. However, adapting the stadium for football games required the owner to enlarge the playing field and remove 1,000 seats, which had been affixed into the concrete infrastructure of the stadium, from the centerfield bleacher area. When the trust company learned about the owner’s plans, it objected to the removal of the 1,000 seats, claiming that it would impair the value of its security interest in the stadium.

If the trust company brings suit against the owner to enjoin the proposed removal of the 1,000 seats at the stadium, judgment should be for whom?

The owner, because the seats are trade fixtures, removable by the affixer. The owner, because his actions were reasonable under the circumstances.

The trust company, because the seats are fixtures, making them part of the realty and subject to the trust company’s security interest.

The trust company, because the removal of the seats would reduce the value of its security interest.

A

The correct answer is: The owner, because his actions were reasonable under the circumstances.

Discussion of correct answer: This difficult question deals with an interrelationship of fixtures, mortgages, and remedies. Clearly, the trust company has a construction mortgage interest in the stadium complex. The seats are fixtures that have been affixed or annexed to the structure. The mortgagor, the owner, wishes to remove part of the seating in order to enhance the value of the stadium to make it adaptable for football (as well as baseball) games. A mortgagor is permitted to do such acts on the mortgaged property that are considered usual and proper, including remodeling structures and removing fixtures in a reasonable manner. An injunction will not be granted for the removal of fixtures as long as there remains adequate security for the mortgagee.

254
Q

A man owned a mansion on 100 acres where he raised ostriches and ran a record company. He purchased the property for $5,000,000. He paid the seller a down payment of $1,000,000 and secured a mortgage from a bank for $4,000,000. The interest rate on the mortgage loan was fixed at 7% for a term of 30 years. In order to secure the loan in a timely manner, he signed documents at closing in which he waived his right to redeem the property in the event of foreclosure. He had an attorney represent him at the closing. For four years, income from the record company and selling ostrich eggs to local eateries was sufficient to pay the $27,000 per month mortgage payment. Due to a downturn in the demand for the man’s type of music, and the loss of numerous ostriches from disease, he defaulted on the mortgage loan. The bank commenced foreclosure proceedings. The bank had the property appraised at $8,000,000. After hearing about the appraisal, the man borrowed money from his music industry friends. The judge granted the bank’s motion for summary judgment and set a date for a sheriff’s sale. The man tendered a check to the bank in the amount sufficient pay off the mortgage and all debt otherwise due on the loan. The bank refused to accept his payment. His attorney filed a motion to dismiss the foreclosure suit.

Should the motion be granted?

Yes, because he was entitled to redeem the property even after the foreclosure sale.

Yes, because the waiver of the right to redeem was invalid.

No, because he may only redeem the property after the foreclosure sale.

No, because he waived the equitable right of redemption.

A

The correct answer is: Yes, because the waiver of the right to redeem was invalid.

Discussion of correct answer: At any time after default but before foreclosure, the mortgagor has the right to redeem the property by paying the debt due. The right to redeem may not be waived (“clogged”) at the time the mortgage is created. Even though he signed the document, and had an attorney present, a person may not waive his right to redeem. Since he tendered the full amount due, the bank must accept payment. Thus, the motion to dismiss should be granted.

255
Q

A buyer purchased a two-acre residential lot fronting a recreational lake owned by the local county. He intended to construct his retirement home on the property. His friend was envious of the prime location and expressed an interest in purchasing the lot from the buyer. The buyer refused, but said that he might be willing to share the land with his friend.

Eventually, the friend wore the buyer down, and he agreed that he would deed half of the land to her in exchange for a payment of $100,000, roughly half the market value of the total property. They discussed the matter over dinner, and the buyer made some notes on the back of a paper dinner napkin. He noted the price, wrote that he agreed to sell “half of my lot fronting County Lake,” and signed the napkin. At that dinner, the buyer stated that he intended to keep the north half of the lot, as it had the best view, and that he would deed the south half to his friend. The friend gave the buyer the $100,000 the next day.

Immediately thereafter, the friend hired an architect to design a home for her on the south half of the land. When the architect asked to see the description of the property in the deed, she replied that she did not have one yet. The architect advised her that it would be unwise to begin his design until he had seen a deed of the land and had the land surveyed. When the friend went to the buyer to ask for a deed, he said that he had changed his mind, and that he had decided that he needed the middle of the land to build the home that he had in mind.

If the friend brings an action for specific performance of her agreement with the buyer, to what, if anything, will she be entitled?

Nothing.

The south half of the land, as originally agreed.

Whichever (contiguous) half of the land the buyer chooses; he cannot grant her two half-acre lots on either side of the middle lot.

The south half of the land, since that is where she has started to make improvements.

A

The correct answer is: Nothing.

Discussion of correct answer: To be valid, a contract for the sale of land must be in writing, signed by the party to be charged, and contain essential terms, including an adequate description of the property to be sold. Here, the description written on the napkin (“half of my lot fronting County Lake”) is an insufficient description of the land to satisfy the statute of frauds. Likewise, the buyer’s oral agreement to sell his friend the south half of the land is unenforceable. As such, the buyer will be able to bar specific performance of this agreement by raising the statute of frauds as a defense.

256
Q

A day trader made several jokes about a lawyer’s physical appearance at a bar. In an attempt to put an end to the insults, the lawyer walked over to the day trader, picked up a beer bottle and stared down the day trader. Unaffected by the lawyer’s display, the day trader laughed and sat back down.

Is the lawyer liable for assault?

(A) Yes, because the lawyer intended to put the day trader in fear of harmful touching.
(B) Yes, because the lawyer picked up a beer bottle and approached the day trader.
(C) No, because the day trader was not in fear of harmful contact.
(D) No, because the lawyer did not threaten an immediate contact.

A

(C) No, because the day trader was not in fear of harmful contact.

257
Q

A building owner leased commercial property to business tenants. A spa owner rented space from the building owner. During a period of multiple heavy snowfalls in the winter, the building owner failed to keep the parking lot clear of snow and ice. The spa owner remained in the leased spaced and continued to open her business to customers, but the spa owner complained to the building owner several times that the condition of the parking lot was causing her to lose business. The spa owner stopped paying rent to the building owner and claimed that no rent was owed because the building owner breached the covenant of quiet enjoyment contained in the lease.

Is the spa owner correct?

A. Yes, because the building owner has a duty to maintain the parking lot.

B. Yes, because the building owner’s actions substantially interfered with the spa owner’s use of the space.

C. No, because the spa owner remained in the leased space.

D. No, because the building owner’s actions did not substantially interfere with the spa owner’s use of the space.

A

C. No, because the spa owner remained in the leased space.

Discussion of correct answer: Every lease contains a covenant of quiet enjoyment whereby the landlord promises not to disturb the tenant’s possession of the property. If the landlord breaches the covenant of quiet enjoyment by constructive eviction of the tenant, the tenant can treat the lease as terminated and withhold rent. Constructive eviction requires that the landlord substantially and permanently interfere with the tenant’s use and enjoyment of the premises, and the tenant must move out. Here, it is arguable that the building owner’s act in not keeping the parking lot clear of ice and snow substantially interfered with the spa owner’s use and enjoyment of the premises. However, the spa owner did not move out of the space, so the spa owner cannot claim breach of the covenant of quiet enjoyment by constructive eviction. (Further, the non-removal of ice and snow would not be a permanent interference with the spa owner’s use of the space because the ice and snow would eventually melt, so the constructive eviction theory would fail for that reason as well.) Test tip: When the question asks “Did the landlord breach the covenant of quiet enjoyment?” it is usually asking you to decide if the landlord constructively evicted the tenant, which requires the tenant to move out before bringing an action. In this case, the tenant never moved out and continued to keep her business open, so her motion would fail.

258
Q

A man sold property to a friend to settle his gambling debts. His friend did not record the deed. Instead, she told the man that she would keep the property for him until he took care of his gambling problem. The man planned to enter a treatment program for compulsive gamblers, but before he did, he played in one more poker game. To cover his losses in the game, he sold the same property to one of the other players. The man did not tell the other player about the deed to his friend. The player recorded the deed immediately.

A few weeks later, the player donated the property to a charity. The friend brought a suit for a declaratory judgment, claiming superior title to the property. The jurisdiction has a race-notice recording statute.

Does the friend have a claim to the property that is superior to that of the charity?

(A) No, because she never recorded her deed.

(B) No, because the shelter rule allows the charity to have superior title even though it does not qualify under the race-notice statute.

(C) Yes, because the charity was not a purchaser for value.

(D) Yes, because the sale from the man to the player was fraudulent.

A

(B) No, because the shelter rule allows the charity to have superior title even though it does not qualify under the race-notice statute.

259
Q

A lawyer and teacher entered into a one year residential lease agreement for a ski chalet. The lease provided that the lawyer was required to provide heat to the chalet during the winter months. During the entire month of January, the temperature was below 50 degrees F in the chalet. The teacher complained to the lawyer on numerous occasions and put him on formal notice by sending him a letter that the heat was inadequate and the chalet was “uninhabitable”. The teacher used a portable wood stove to stay warm and remained in the chalet. The jurisdiction follows the majority rule.

Which is the most accurate statement?

A. The teacher may not withhold the entire rent, because she remained in the chalet.

B. The teacher is entitled to a rent abatement because the lawyer has breached the covenant of quiet enjoyment but is not relieved of all liability for rent.

C. The teacher may withhold the entire rent because she has been constructively evicted.

D. The teacher may not withhold the entire rent, because she has merely been constructively evicted.

A

A. The teacher may not withhold the entire rent, because she remained in the chalet.

Discussion of correct answer: Under the majority rule, a tenant may treat a lease as terminated if the landlord breaches the covenant of quiet enjoyment (express or implied in every lease) by constructive eviction of the tenant. In order for a constructive eviction to exist: (i) the landlord must substantially and permanently interfere with the tenant’s use and enjoyment of the premises; and (ii) the tenant must move out. In this case, given the teacher did not move out, she has not been constructively evicted and may not withhold the rent.

260
Q

A strict mother routinely sent her 6-year-old son to his bedroom whenever she believed he needed a “time-out.” As a result, the rambunctious and often disobedient son frequently spent up to four hours per day in his room.

One day, the mother had a business appointment in the city and hired a babysitter to watch her son. After only an hour, the boy began to misbehave to the point where the babysitter decided that he needed a time-out. The babysitter locked the boy in his bedroom, where he remained until the mother returned several hours later. Playing happily in his room, the boy was unaware that his bedroom door was locked.

Will the babysitter face liability for confining the boy?

(A) Yes, because such confinement is considered false imprisonment.
(B) Yes, because the babysitter was not the boy’s parent.
(C) No, because the mother did not give the babysitter permission to confine the boy to his bedroom.
(D) No, because the babysitter believed that the boy needed to be confined.

A

(D) No, because the babysitter believed that the boy needed to be confined.

261
Q

A landlord entered into a lease agreement with a tenant. The jurisdiction in which the leased premises were located had enacted a statute requiring landlords to maintain a temperature of at least 65 degrees Fahrenheit in leased apartments during the winter months. The lease agreement itself contained no such requirement. In January of the first year of the lease, the temperature in the apartment was no higher than 58 degrees. The tenant vacated the apartment and sued the landlord for violating the lease’s covenant of quiet enjoyment.

What remedies are available to the tenant in most jurisdictions?

(A) The tenant may treat the lease as terminated and withhold rent, because he has been constructively evicted.

(B) The tenant is entitled only to a rent abatement, because he has been constructively evicted.

(C) The only remedies available to the tenant are those provided for in the statute, because the landlord has not violated the lease.

(D) The only remedies available to the tenant are those provided for in the statute, because the tenant vacated the apartment.

A

(A) The tenant may treat the lease as terminated and withhold rent, because he has been constructively evicted.

262
Q

A teacher borrowed $50,000 from a bank and issued to the bank a mortgage on Goldacre, a parcel owned by the teacher. The mortgage, which was properly recorded, provided that the debt was to be paid in full by September 1, 2025. Six months later the teacher borrowed $100,000 from a credit union and gave the credit union a mortgage on Goldacre. The credit union recorded the mortgage. One year later the teacher and the bank entered into a modification agreement in which the bank agreed to extend the due date on mortgage held by bank to September 1, 2030.

Which mortgage has top priority?

(B) The credit union’s mortgage, because the modification on the bank’s mortgage changed its priority.

(A) The credit union’s mortgage, because it was issued for a higher loan amount than the bank’s mortgage.

(C) The bank’s mortgage, because it was recorded prior to the credit union’s mortgage.

(D) The bank’s mortgage, because priority is unaffected by the modification agreement.

A

(D) The bank’s mortgage, because priority is unaffected by the modification agreement.

263
Q

A therapist rented an apartment from a landlord pursuant to a lease for a term of one year. Rent was payable monthly, and the lease was renewable by the therapist by sending written notice of intent to renew at least 30 days before the end of the lease term. Six months into the lease term, the therapist was hospitalized with a terminal illness. Her sister was appointed conservator of her estate because the therapist was adjudicated unable to manage her own affairs. The landlord tried to find a tenant to fill the remainder of the lease term but could not.

If the landlord brings an action against the therapist’s sister as her conservator to recover the rent due for the remainder of the lease term, what will be the likely outcome?

The estate will prevail, because the therapist’s incompetency terminated the lease agreement.

The estate will prevail, because privity of estate between the therapist and the landlord was broken when the therapist abandoned the premises.

The landlord will prevail under the jurisdiction’s dead man’s statute.

The landlord will prevail, because the therapist’s possessory interest in the apartment was not terminated by the adjudication of her incompetency, but passed to her estate.

A

The correct answer is: The landlord will prevail, because the therapist’s possessory interest in the apartment was not terminated by the adjudication of her incompetency, but passed to her estate.

Discussion of correct answer: In most jurisdictions, a lease does not terminate upon the death or incapacity of either party to the lease. Rather, the deceased or incompetent party’s interest passes to his or her estate. The therapist’s estate will be liable for the lease. Thus, this is the correct answer.

264
Q

In an area where many homes are heated with natural gas, a gas company has an easement on Landowner’s land for the pipe that brings the natural gas into Landowner’s home. When Landowner switches to a combination of solar and geothermal energy, he discontinues the use of natural gas.

Has the gas company’s easement been terminated?

Yes, because the easement is no longer strictly necessary.

Yes, because Landowner is no longer receiving natural gas through it.

No, because easements are permanent unless they are created by a writing stating otherwise.

No, because the gas company has shown no intent to abandon the easement.

A

The correct answer is: No, because the gas company has shown no intent to abandon the easement.

Discussion of correct answer: An easement is not abandoned just because it is not being used. Instead, abandonment of an easement requires intent to abandon it and an act in furtherance of that intent. Neither is present here, so the easement is still effective.

265
Q

A major electronics store hired a new employee. The employee went through all of the required training and did very well. He was soon elevated from trainee and permitted to assist customers without supervision. One day, as he came down the ladder from retrieving some merchandise for a customer, another customer commented on the employee’s actions using a derogatory racial epithet. Momentarily stunned, the employee then followed the customer outside and across the street to where the customer’s car was parked. The employee caught up to the customer and asked for an apology. The customer repeated what she already said, and the employee punched her in the face. The customer filed suit against the employee and the electronics store.

Will the customer prevail in her action against the electronics store?

A. No, because the customer was not injured on store property.

B. No, if the employee was acting outside the scope of his employment.

C. Yes, because of the doctrine of respondeat superior.

D. Yes, because the employee was at work when the incident occurred.

A

B. No, if the employee was acting outside the scope of his employment.

266
Q

A pear grower owned a large tract of land with a farmhouse, a tool shed, and two small pear orchards. In one orchard, the grower was raising a common pear for sale in local grocery stores. In the other, the grower was experimenting with a foreign pear type that had required a lot of capital to establish but if successful, would net a small fortune from internet-based fruit basket retailers. The grower had a bank-owned mortgage on her property and because of unexpected setbacks in the new fruit’s production, she had fallen behind and was in default. The new pears were nearly ripe when the mortgage bank began judicial foreclosure proceedings.

The grower begged the bank to allow her time to harvest the fruit and catch up on her payments, but the bank denied her pleas. The foreclosure sale occurred, at a small loss, and the property was purchased by a developer who planned to turn the property into a high-end subdivision. The day after the sale, the grower went back onto the property with a crew of temporary laborers and harvested the pears, storing them at a facility she had rented. The developer noticed that the pears were missing when he went out to survey the
property a couple of days later. The developer had planned on benefiting from the income from the pears and, suspecting the grower, contacted the police. What is the likely outcome?

(A) The grower will be arrested for theft.

(B) The grower will keep the pears but owes the proceeds from their sale to the mortgage bank to cover the loss.

(C) The developer will get the pears back but will owe the proceeds from their sale to the mortgage bank to cover the loss.

(D) The bank will take ownership of the pears and sell them to cover the loss.

A

(A) The grower will be arrested for theft.

267
Q

An owner held a parcel of land in a race-notice jurisdiction in fee simple. She sold the property to a buyer for a considerable amount of money. The buyer received a general warranty deed. She did not immediately record the deed. The owner then gave a general warranty deed to the property to her sister as a gift. The sister immediately recorded the deed. She had no notice of the sale to the buyer. The buyer then remembered the deed and recorded it. The sister subsequently mortgaged the property to her boss, who immediately recorded the mortgage. Some time after that, the buyer mortgaged the property to her best friend, who immediately recorded the mortgage. The buyer defaulted on her mortgage, and the buyer’s best friend now seeks to foreclose. The sister’s boss learned of the proceedings from the newspaper and contested the foreclosure.

If the buyer’s best friend prevails in her action, which of the following best accounts for the court’s ruling in her favor?

The sister was not a bona fide purchaser.

The sister’s deed is outside the buyer’s chain of title.

The boss is charged with notice of the buyer’s mortgage to her best friend.

The owner’s conveyance to the sister is void because she had no interest to convey.

A

The correct answer is: The sister was not a bona fide purchaser.

Discussion of correct answer: Recording statutes are designed to protect only subsequent bona fide purchasers. Because the sister received the property as a gift, she would not qualify as a subsequent bona fide purchaser who could claim protection under the recording statute. The sister’s boss, as mortgagee, would qualify as a giving value, though. However, the buyer recorded before the sister took out the mortgage with her boss. As a result, the buyer’s best friend would be able to shelter under the buyer’s deed and claim higher priority over the sister’s boss.

268
Q

When an astronomer went overseas to study a solar eclipse, she left her assistant in charge of her financial affairs. Before leaving, she signed a document stating: “My assistant has my permission to deal with my real and personal property as if it were his own, as long as any profits realized remain my property.” The assistant did not record the document. The assistant sold the astronomer’s Fifth Avenue penthouse to a florist. The selling price was well above the amount remaining on the mortgage. Title was conveyed by warranty deed signed by the assistant. The astronomer returned from her expedition to find a large sum of money in her mutual fund and the florist living in her home. The astronomer brought suit to eject the florist. The florist then brought suit against the assistant for breach of the covenants included in a warranty deed.

Will the astronomer succeed in her action to have the florist ejected?

Yes, because a warranty deed must be signed by the person listed in the county clerk’s records as the owner.

Yes, because the florist is not a bona fide purchaser.

No, because the florist is a bona fide purchaser.

No, because the assistant had the authority to convey title by warranty deed.

A

The correct answer is: No, because the assistant had the authority to convey title by warranty deed.

Discussion of correct answer: A landowner may give written permission to another person to act as an agent in the sale of land. Here, the astronomer clearly gave her assistant permission to sell her property. Thus, the authority she granted to her agent, the assistant, binds her under the sale agreement.

269
Q

An elderly woman who needed to move into assisted housing agreed to sell her home to her neighbor for a below market price. Because the neighbor was notorious for taking poor care of his own property, a coalition of local residents came together in opposition to the deal. The coalition members offered the woman an extra $10,000 for the home, and the woman sold it to them on the spot. They paid her immediately, but they did not record the deed right away. With the elderly woman’s permission, they started planting flowers in the yard and they painted the outside of the house. When the time came for the elderly woman to move, the neighbor visited, told the woman he was disturbed by rumors he had heard that she had sold the property to the neighborhood coalition, and reminded her of their deal. Feeling guilty, the elderly woman lied and said that the deal with the coalition had fallen through, though the elderly woman wasn’t a very convincing liar. The neighbor knew that the coalition had planted flowers and painted the house. However, because he wanted the house, the neighbor gave the woman a cashier’s check in exchange for the deed, which the neighbor recorded immediately. The following day the coalition recorded its deed. Assume this transaction took place in a race-notice jurisdiction. Who owns the property and why?

(A) The coalition, because it purchased first.

(B) The coalition, because it purchased first and the neighbor had notice of its deed.

(C) The neighbor, because he recorded first.

(D) The neighbor, because he is a bona fide purchaser.

A

(B) The coalition, because it purchased first and the neighbor had notice of its deed.

270
Q

A tenant agrees to rent an apartment from a landlord on a month-to-month basis. No written agreement is made.

Which of the following is an implied obligation of the tenant at the end of the lease?

(A) To repaint the apartment and clean the carpets.

(B) To give the landlord proper advance notice of the tenant’s intent to terminate the lease.

(C) The tenant has no duty to give advance notice to the landlord because either party may terminate the lease at will.

(D) Not to vacate the premises at the end of the tenancy until the tenant receives the security deposit back.

A

(B) To give the landlord proper advance notice of the tenant’s intent to terminate the lease.

271
Q

A tenant leased a house in a suburban neighborhood from a landlord. The lease was silent as to which party bore the responsibility to make repairs. One day, a group of neighborhood kids were playing baseball in the street when an errant ball went through a second-floor window of the tenant’s house. The tenant replaced the broken window pane at a cost of $300 and deducted the amount from the next month’s rental payment. The landlord then sued to recover the $300 from the tenant.

Will the landlord prevail in recovering the $300 against the tenant?

A No, because deduction of the cost of repairs was the appropriate remedy for breach of the warranty of habitability.

B No, because the broken window constitutes a partial, constructive eviction.

C Yes, because the lease did not specify that the landlord had the burden of making repairs.

D Yes, because the tenant did not notify the landlord of the damage and give the landlord a reasonable time to make repairs.

A

C Yes, because the lease did not specify that the landlord had the burden of making repairs.

Discussion of correct answer:Under the common law rule, in the absence of contractual terms to the contrary, a tenant has a duty to make minor repairs to keep the leased premises wind and water tight. This responsibility arises from the tenant’s duty not to commit waste. In this case, given that the lease did not state which party would be responsible for making repairs, the default common law rule applies, and the tenant is responsible for repairing the window. Because liability for the broken window lies with the tenant, the tenant may not deduct the cost of making the repair from the rent payment.

Discussion of incorrect answers:

Incorrect. No, because deduction of the cost of repairs was the appropriate remedy for breach of the warranty of habitability. A broken window does not make a house uninhabitable. Consequently, the warranty of habitability was not breached, and the tenant cannot deduct the cost of the replacement.

Incorrect. No, because the broken window constitutes a partial, constructive eviction. The broken window does not constitute a constructive eviction because it is not a substantial and permanent interference with the tenant’s enjoyment of the property, forcing him to move out.

Incorrect. Yes, because the tenant did not notify the landlord of the damage and give the landlord a reasonable time to make repairs. Responsibility for repairing the window lies with the tenant, not the landlord. Under the common law rule, a tenant has a duty not to commit waste, which includes a duty to make minor repairs to keep the leased premises wind and water tight. Therefore, it is the tenant’s duty, rather than the landlord’s, to repair the broken window.

272
Q

In 2050, a developer purchased a parcel of land from the government. He intended to develop the area as a ski resort. However, before he could make any improvements on the land, the developer suffered a head injury and was declared incompetent. In 2055, an investor purchased the parcel next to the developer’s. Again, the seller was the government. The investor built a large successful ski resort and, in 2065, she inadvertently expanded her resort onto the developer’s land. The developer died in 2080. His daughter inherited his estate. In 2090, the investor was killed when she skied into a tree, and her two sons inherited the resort. In 2104, the developer’s daughter sought to have the investor’s sons ejected from that part of their resort that was on her land. The statutory period for adverse possession in this jurisdiction is 20 years.

Who will prevail in a suit by the developer’s daughter against the investor’s sons, and why?

The developer’s daughter will succeed, because the statute was tolled during the incompetency of the developer.

The developer’s daughter will succeed, because the investor’s sons have not been in possession for the statutory period of 20 years.

The investor’s sons will succeed, because the developer’s daughter cannot claim the statute was tolled during her father’s incompetency.

The investor’s sons will succeed because, by tacking their possession with their mother’s possession, they have been in possession of the land for the required statutory period.

A

The correct answer is: The investor’s sons will succeed because, by tacking their possession with their mother’s possession, they have been in possession of the land for the required statutory period.

Discussion of correct answer: In order to fulfill the statutory period for adverse possession, there need not be continuous possession by the same person. The period of adverse possession of one possessor can be tacked to the period of adverse possession of another possessor when there is privity between the two. Because the investor’s sons took possession after a voluntary transfer from the investor, they are in privity and can tack their period of possession to her period of possession. In addition, they can claim open, hostile, and continuous possession for the statutory period of 20 years, satisfying all the requirements for adverse possession.

273
Q

A property owner has mortgages on his property with two banks. Bank No. 1 has the first mortgage in the amount of $75,000. Bank No. 2 has the second mortgage in the amount of $35,000. The property owner obtained the second mortgage to finance a new addition to the property, but the owner ran out of funds to complete the addition and left it unfinished. The property owner has accumulated other large debts relating to the new addition and decides he cannot afford to continue making payments to the banks. Bank No. 1 files a foreclosure action after the owner defaults on the first mortgage. If there is a foreclosure sale, what is the relation between Bank No. 1 and Bank No. 2?

(D) Bank No. 1’s interest is junior to Bank No. 2’s interest.

(A) Bank No. 1 can extinguish Bank No. 2’s mortgage.

(B) Bank No. 1 does not need to do anything about Bank No. 2’s mortgage.

(C) Bank No. 1 should obtain a judgment in foreclosure but should not authorize a foreclosure sale.

A

(A) Bank No. 1 can extinguish Bank No. 2’s mortgage.

274
Q

A widow executed a deed naming her bridge partner as the grantee of her summer home. The widow gave the deed to her attorney, with written instructions to give the deed to the bridge partner upon his return from a foreign cruise. Several weeks later, the widow learned that the bridge partner had committed horrible crimes while on the cruise. The widow therefore asked her attorney to return the deed; the attorney refused. The widow brought an action to recover the deed from the attorney.

Is the widow likely to be successful in her suit against her attorney?

No, because the widow manifested a present intent to create a future interest when she gave the deed to the attorney.

No, because title passed to the bridge partner at the moment the widow gave the deed to her attorney.

Yes, because the attorney is the widow’s agent.

Yes, because the widow did not have the intent to transfer title when she delivered the deed to the attorney.

A

The correct answer is: No, because the widow manifested a present intent to create a future interest when she gave the deed to the attorney.

Discussion of correct answer: An escrow involves the deposit of a properly executed deed by the grantor with a third party who is to deliver the deed to the grantee upon the happening or performance of a named event. When the widow gave the deed to the attorney with the instructions to deliver the deed to her bridge partner upon his return from his cruise, the widow (the grantor) manifested a present intent to create a future interest in her bridge partner (the grantee). Once the widow gave the deed to her attorney (a third party), she could not retrieve the deed to prevent the transfer to the bridge partner upon the occurrence of the stated condition. In cases such as this, the courts have held that the grantor’s intent at the time of the transfer to the third party was to create a “springing future interest” in the grantee, making delivery irrevocable.

275
Q

Before they married, a husband and wife purchased a home as their future residence. They took title as joint tenants and subsequently moved onto the property to begin their married life. The husband got into the habit of trading stock online after work each evening. He soon began to spend five to seven hours at the computer each night, and lost a great deal of money from bad investments. He was ashamed to tell his wife the extent of his losses, so to cover those losses, the husband conveyed his interest in the marital home to an investor, who saw the property as a good investment, but had no interest in taking up occupancy. The wife knew nothing of the conveyance. The husband entered psychotherapy to control his trading habit, which had only worsened after the conveyance to the investor. Eventually, the husband drank himself to death, in remorse for squandering his savings. The husband left a will by which he bequeathed all his personal property to his wife and all his real property to his son from a former marriage. The wife filed a quiet title action to determine the ownership of the marital home.

How should the court rule?

(A) In favor of the wife and the husband’s son from his prior marriage.

(B) In favor of the wife and the investor as joint tenants.

(C) In favor of the wife and the investor as tenants in common.

(D) In favor of the wife, because the husband could not sever the tenancy by the entirety that arose upon their marriage.

A

(C) In favor of the wife and the investor as tenants in common.

276
Q

A landowner owned a one-acre lakefront lot on which he had built a large mountain-cabin style residence. His gravel driveway ran from the public street on the side of his lot opposite the lake, along the boundary between his lot and the lot owned by his neighbor, then angled inward so that it permitted entry to the garage attached to the landowner’s cabin. The gravel driveway continued past the landowner’s garage, alongside the cabin to the boat-launching ramp the landowner had constructed alongside the small dock where he moored his ski boat and his catamaran.

The landowner agreed to sell the lakefront lot to a purchaser for $200,000 and executed a written contract to that effect. The contract specified that the landowner would convey “good and marketable title,” subject to “any conditions, covenants, and restrictions; easements; or other encumbrances of record.” In the five years the landowner had lived in the lakefront residence, he had allowed the neighbor to use his (the landowner’s) driveway and boat-launching ramp to get the neighbor’s fishing boat onto the lake, since the neighbor had no comparable facilities, and the crowded public boat-launching ramp was 25 miles away on the far shore of the lake.

After execution of the land sale contract but before the time set for closing, the neighbor asked the landowner to put his permission in writing, so that the neighbor could continue to use the driveway and ramp after the purchaser took possession. The landowner executed a valid deed of easement granting the neighbor the right to use the driveway and ramp, which the neighbor promptly recorded. The purchaser had already performed a public records search prior to the time the easement was recorded, so she had no knowledge of the easement. At the closing, the purchaser learned of the easement and refused to consummate the contract of sale. The landowner brought an appropriate action against the purchaser for specific performance of the land sale contract, and the purchaser counter-sued for the return of her money from escrow.

Who will prevail?

The purchaser, because the easement to the neighbor was created after the contract of sale was executed.

The purchaser, because the neighbor was not a bona fide purchaser for value of the deed of easement.

The landowner, but with a rebate to the purchaser of a portion of the purchase price equal to the value of the easement granted to the neighbor.

The landowner, because the contract specified that the purchase was subject to easements and the neighbor had been using the boat launch for five years.

A

The correct answer is: The purchaser, because the easement to the neighbor was created after the contract of sale was executed.

Discussion of correct answer: When a contract for the sale of land specifies that the land is to be conveyed “subject to easements, covenants, and restrictions of record,” unless there is a contrary provision, it is reasonable to imply that the parties are referring to such encumbrances as exist at the time the contract is entered into. It is generally held that the seller need not possess marketable title until the time for closing, which means that an encumbrance existing on the property after formation but before closing does not constitute a breach of the contract; the seller has until closing to remove the offending encumbrance. In this question, however, the facts provide that the seller created a new encumbrance after the contract had been formed. Thus, when the landowner tendered the deed at closing, the lakefront property was subject to an encumbrance that did not exist at the time the contract was formed and could not be discovered by the purchaser during a search of the records. The easement was not “of record” at the time the parties executed the contract, and it is the type of easement that reduces the value of the property. The existence of such an encumbrance renders the landowner’s title unmarketable, a violation of the express contract provision that marketable title will be conveyed. The purchaser will be able to rescind the contract.

277
Q

A landowner died, devising the land that he owned in fee simple absolute to his sister and daughter “as joint tenants with right of survivorship.” The eastern half of the property extended into the mountains, was heavily forested, and had no roads leading into it. The western half of the property was in the valley and had ready access to two major highways. The sister and daughter disagreed on how to use the property. The sister wanted to develop the mountainous portion as a ski resort, while the daughter wanted to subdivide and sell the entire property and invest the proceeds in the stock market. They agreed to compromise: The sister would build a ski resort on the mountainous half of the property. When completed, the daughter could sell the western, flat portion, which would have appreciated in value by virtue of the proximity of the resort, and keep the proceeds. Their agreement was never written. The sister built the ski resort in the mountainous part of the property. Shortly after the resort went into operation, the sister died, devising all of her property to her granddaughter. The daughter started the process of subdividing the western part of the land, but the sister’s granddaughter sued for an injunction forbidding her from doing so.

Will the sister’s granddaughter be able to enjoin the daughter from subdividing and selling the western part?

Yes, because the close family relationship between the daughter and sister removed the necessity of complying with the Statute of Frauds.

Yes, because the daughter’s and the sister’s conduct during the sister’s lifetime severed the joint tenancy.

No, because the daughter had the sister’s consent to subdivide half of the land.

No, because the sister’s granddaughter has no interest in the land.

A

The correct answer is: No, because the sister’s granddaughter has no interest in the land.

Discussion of correct answer: The sister and the daughter received the property as joint tenants with right of survivorship. The conveyance to them had the four unities of time, title, interest (equal shares), and possession (each having the right to possess the whole). As a result, the death of one of them caused the other to have sole ownership of the whole. The sister did not have the right to devise any part of the property to her granddaughter.

278
Q

With retirement approaching, a school teacher decided to sell his apartment in the city. His first offer came from a sailor, who also planned to retire and who offered to exchange his boat for the apartment. The teacher agreed, and the two agreed to meet in a week to close the deal. The next day, however, a businesswoman offered the teacher nearly twice the value of the apartment in cash. The teacher agreed, and they exchanged the deed for the cash on the spot. Six days later, the sailor came to the apartment to close the deal and the teacher, seeing the boat as the perfect getaway, gave the sailor a deed as well, in exchange for the keys as planned. The sailor promptly recorded the deed. A couple days later, the businesswoman recorded her deed. Neither purchaser was aware of the other’s deed until the businesswoman showed up at the apartment the following week and discovered that the sailor had just moved in. The businesswoman filed suit seeking to quiet title. Assume these events took place in a race-notice jurisdiction.

How should the court rule?

A. The businesswoman owns the property, because her offer was accepted first.

B. The businesswoman owns the property, because the Statute of Frauds requires all contracts for the sale of real property be in writing.

C. The sailor owns the property, because he recorded first without notice of the businesswoman’s agreement.

D. The sailor owns the property, because the recording statute protects those who purchase for greater value.

A

C. The sailor owns the property, because he recorded first without notice of the businesswoman’s agreement.

Discussion of correct answer: In a race-notice jurisdiction, the person who records first without notice of a prior claim on the property is the rightful owner of that property. Here, because he had no notice of any prior claim on the property, paid value, and recorded first, the sailor owns the property. The businesswoman’s earlier agreement with the teacher is irrelevant.

279
Q

A buyer and seller executed a contract that called for the buyer to purchase the seller’s house and the small business he operated out of the basement. The buyer expressly stated to the seller that she intended to live in the house and continue to operate the business. At the closing, the buyer learned that the business was in violation of local zoning ordinances. The buyer informed the seller that she wanted her down payment returned and that she would not complete the closing. The seller, who had already used the buyer’s down payment to make a down payment of his own on a condominium, sued for specific performance. The buyer defended on the grounds that the seller could not convey marketable title.

Who will prevail and why?

The buyer, because the seller misrepresented the legality of the business.

The buyer, because the zoning violation renders the title unmarketable.

The seller, because the buyer could have easily learned of the zoning restrictions before she signed the purchase- and-sale agreement.

The seller, because the zoning ordinance does not render the title unmarketable.

A

The correct answer is:The buyer, because the zoning violation renders the title unmarketable.

Discussion of correct answer:Title to property currently in violation of zoning regulations is unmarketable. It is true that it is the business that is in violation of zoning regulations here, and not the property. However, the seller knows that the buyer intends to continue the business after purchasing the property, and that the business is currently in violation. Because the seller sought to sell property currently in violation, which he knew the buyer intended to continue to use in the same manner, he did not have marketable title to convey. The seller is essentially selling what he knows will be a lawsuit for the buyer, because he knows the business violates zoning regulations and knows that the buyer intends to continue the business. Therefore, the buyer will be entitled to rescission of the contract and a return of her down payment.

280
Q

An entrepreneur was having breakfast at a restaurant and overheard a warehouse owner, sitting at the next table, tell the owner’s companion, “I’ve got to get rid of the warehouse; the district’s been rezoned C-1 and the property taxes are killing me!” The entrepreneur went to the owner’s table. The entrepreneur told the owner that the entrepreneur was familiar with the warehouse property, and the entrepreneur offered the owner $100,000 for it, “on the spot.” When the warehouse owner hesitated, the entrepreneur wrote the owner a personal check for $5,000 and said, “This will show I mean business; you give me a deed here at breakfast tomorrow, I’ll have the rest in cash. Let’s shake on the deal.” The owner shook the entrepreneur’s hand and said, “OK, it’s a deal.” The entrepreneur gave the owner the $5,000 check and returned to his own table. The next morning the entrepreneur was at the restaurant with a cashier’s check for $95,000. When the owner appeared, the entrepreneur offered the owner the check and asked for the deed. Although the owner had prepared and executed a warranty deed in the entrepreneur’s favor, instead of delivering the deed, the owner returned the entrepreneur’s $5,000 personal check, stating that the owner had received another offer for the warehouse for $150,000. The entrepreneur refused to accept return of his personal check and sued the owner for specific performance.

Who will prevail?

A The owner, because the transaction with the entrepreneur was not in writing.
B The owner, because a cashier’s check is not the equivalent of cash.
C The entrepreneur, because the entrepreneur has partly performed the oral agreement.
D The entrepreneur, because the doctrine of equitable conversion treats the entrepreneur as owner of the warehouse.

A

A The owner, because the transaction with the entrepreneur was not in writing.

Discussion of correct answer: The Statute of Frauds requires that a contract for the sale of real property be evidenced by a writing, signed by the party to be charged thereby, and containing all the essential terms of the transaction, such as identification of the parties, a description of the property, and the terms and conditions of the deal. Therefore, the oral agreement to sell the warehouse is unenforceable.

281
Q

A skier had skied for many years. The skier was proceeding down a relatively steep hill on an advanced course by a series of traverses. He was crossing the slope a number of times to diminish the angle of descent the skiing equivalent of switchbacks in hiking. At the same time a ski instructor, who was also the owner of her own ski school and a member of the ski patrol, came straight down the hill, saw the skier midway across the hill in one of his traverses, and attempted to ski behind the skier. The ski instructor miscalculated the speed at which the skier was traveling, and ran directly into him, fracturing the skier’s knee.

The ski instructor’s conduct will be judged against which of the following standards?

A. The conduct of a reasonably prudent person.

B. The conduct of a reasonably prudent person in an emergency.

C. The conduct of a reasonably prudent person with superior skiing knowledge and expertise.

D. The conduct of a reasonably prudent person with the same age, knowledge, and experience.

A

C. The conduct of a reasonably prudent person with superior skiing knowledge and expertise.

282
Q

A first-time home buyer financed the purchase of a house with a $100,000 mortgage she took out with a bank. The mortgage was recorded. A few years later she borrowed $5,000 from a finance company to pay for a foreign trip, using her house as security. The finance company promptly and properly recorded its mortgage on the property. One year after that, she borrowed $40,000 from an equity company to pay for an addition on the house. The equity company promptly and properly recorded the mortgage it took on the property. Shortly thereafter, she lost her job and was unable to make payments on either the finance company’s or the equity company’s mortgages, but she was able to make payments on the bank’s mortgage. The finance company filed foreclosure of its mortgage and included the equity company in the action, and a purchaser bought the property at the foreclosure sale.

What is the purchaser’s obligation regarding the bank’s mortgage and the equity company’s mortgage?

(D) The purchaser takes the property subject to the bank’s mortgage, but not subject to the equity company’s mortgage.

(C) The purchaser takes the property subject to the equity company’s mortgage, but not subject to the bank’s mortgage.

(A) The purchaser takes the property subject to both mortgages.

(B) The purchaser takes the property subject to neither mortgage.

A

(D) The purchaser takes the property subject to the bank’s mortgage, but not subject to the equity company’s mortgage.

B1, can foreclose on the house, after the foreclosure sale
F2-defaults, foreclosure, notified E3
E3-defaults, because notice provided, exterminated

283
Q

A tenant represents to an investor that he owns Cityacre. On May 1, the tenant and the investor enter into a written land-sale contract, the terms of which provide that, on September 1, the investor will pay the tenant $75,000, and the tenant will deliver to the investor a general warranty deed to Cityacre. On June 1, the investor and a purchaser enter into a written land-sale contract. According to the terms of the contract, on October 1, the purchaser will pay the investor $100,000, and the investor will deliver to the purchaser a general warranty deed to Cityacre. Unbeknownst to the investor or the purchaser, the tenant is only leasing Cityacre from the owner. On September 1, the investor tenders $75,000 to the tenant. In exchange, the tenant delivers to the investor a purported general warranty deed to Cityacre. Then, on October 1, the purchaser pays the investor $100,000, and the investor delivers to the purchaser a purported general warranty deed to Cityacre. On October 15, the investor discovers that the owner, not the tenant, was and is the owner of Cityacre. On October 30, the owner gives the investor a general warranty deed to Cityacre in exchange for $85,000. When the purchaser discovers that the investor had not had title to Cityacre on either June 1 or October 1, he brings an action to recover damages from the investor. Should the purchaser recover?

(A) Yes, because the investor conveyed Cityacre to him by general warranty deed.

(B) Yes, because the investor breached the warranty of marketable title implied in his land-sale contract with the purchaser. (every land contract, implied)

(C) No, because under the doctrine of estoppel by deed, the purchaser received title to Cityacre on October 30.

(D) No, because the investor is protected from liability by the doctrine of equitable conversion.

A

(A) Yes, because the investor conveyed Cityacre to him by general warranty deed.

284
Q

A landlord owns an upscale apartment building. An optician rented Apartment A for five years. The lease stated the rent would be paid in installments of $2,000 a month and prohibited subleases. The optician paid his rent on time for the first year of his lease, then got married and decided to move out. The optician signed an agreement with an editor for the editor to move into Apartment A and to pay the rent for the rest of the lease. The editor moved into Apartment A and paid the rent on time for the next three years. At that time, the editor lost his job, and vacated the apartment without paying the last two months’ rent on the lease.

Who is liable for the last two months’ rent?

(A) The optician, because subleases were prohibited.

(B) The optician is liable for $2,000, and the editor is liable for $2,000.

(C) The optician and the editor are jointly and severally liable for $4,000.

(D) The editor, because he agreed to pay the rent.

A

(C) The optician and the editor are jointly and severally liable for $4,000.

285
Q

A landlord and a soft drink manufacturing company entered into a two-year commercial lease agreement for the lease of a factory owned by the landlord. When the soft drink manufacturing company took possession of the factory at the beginning of the lease term, its engineers discovered that there was a high concentration of toxic chemicals in the factory’s air due to the manufacturing activities of the previous tenant. The landlord, who was not aware of these toxic chemicals until notified by the soft drink manufacturing company, refused to pay to have the chemicals removed from the air. The soft drink manufacturing company’s lawyer wrote a threatening letter to the landlord asserting that it had breached the lease’s implied warranty of habitability.

In most jurisdictions, would the lawyer’s contention be successful in a court of law?

(A) No, because the landlord did not know about the chemicals at the time it entered into the lease with the soft drink manufacturing company.

(B) No, because the lease is a commercial lease.

(C) Yes, because the tenant notified the landlord of the toxic chemicals within a reasonable time.

(D) Yes, because the lease is not a long-term lease.

A

(B) No, because the lease is a commercial lease.

286
Q

A car had a flawed transmission design. The car would go into reverse without being shifted, first hesitating and moving in a jerky way. An owner of this model knew of this issue. Her car also had defective brakes; too much heat would cause the brakes to fail, but she was not aware of this flaw. One day, the owner’s car started hesitating and jerking, but she decided to keep driving because it was dark and rainy, and she was only a few miles from home. After two miles, her car started to go into reverse. She applied the brakes, but the friction caused the pads to overheat and the brakes failed. The car slammed into a tree and the owner was injured. She brought an action in strict liability against the car manufacturer. The jurisdiction adheres to a comparative negligence theory.

Will the manufacturer be held completely liable for the owner’s injuries?

(A) No, because the owner was comparatively negligent.
(B) No, because she assumed the risk by continuing to drive the car.
(C) Yes, because the design defect in the transmission caused her injuries.
(D) Yes, because the manufacturing defect in the brakes caused her injuries.

A

(D) Yes, because the manufacturing defect in the brakes caused her injuries.

287
Q

A man’s dream was to open a movie theater that would show nothing but the classics of the 1930s and 1940s - especially musicals. He began looking for an appropriate building, erected during that era, in the heart of downtown. The man soon received a call from a theater owner offering to lease a 150-seat theater downtown to the man. The owner informed the man from the outset that he was willing to lease the building at a discount because some of the overhead lighting was in need of repair. It was not adequately secured; in fact, some had fallen during a recent earthquake, and the owner knew that it would take at least two months to make the necessary repairs. The man signed a 1-month lease, hired a local contractor to repair the lighting, and began an advertising campaign with plans to open for business one month later. Response to the man’s concept was overwhelming, with the first ten showings sold out before the opening. The morning of opening day, the contractor informed the man that the lighting would not be completed for at least an additional two days. The man reluctantly proceeded with the opening anyway.

The day after the opening, a 5.0 aftershock hit the city. A section of the lighting hanging from the balcony fell, causing head injuries to a customer at one of the showings.

If the customer sues the owner, who will prevail?

A. The owner, because the man became solely liable for the condition of the premises as of the date he took possession.

B. The owner, because he disclosed the condition of the lighting to the man at the time of the lease.

C. The customer, because she was an invitee.

D. The customer, because the owner knew the man planned to use the building as a theater.

A

D. The customer, because the owner knew the man planned to use the building as a theater.

Discussion of correct answer: Generally, the lessor of real property is not liable for injuries resulting from conditions on the land. The lease is considered a transfer of all rights and responsibilities with regard to conditions on the land. There are a number of exceptions to this rule, however. One exception involves dangerous conditions existing at the time of transfer of possession of the property when the lessor knows the lessee will be holding the property open to the general public and knows that the lessee will not fix the dangerous conditions before the opening. The interest in protecting the public is considered so important that the lessor continues to be liable. The intent is to motivate lessors to transfer property in safe condition. Here, the owner knew the man intended to use the premises as a theater, open to the general public, and knew that the man would not have the necessary two months to make the necessary repairs before he opened the theater during his one month lease. Therefore the owner remains liable for the condition of the lighting.

288
Q

A landlord owned a two-story apartment building containing 10 apartments. The landlord leased one of the apartments to a new student attending a local college for a period of four years. At the end of the first year, the student failed all of her classes and was expelled from the college. The student decided to move back home and live with her parents, so she arranged (with the landlord’s consent) to transfer the remaining three years on the lease to a mechanic. At the student’s request, the landlord and mechanic signed a letter releasing the student from any further liability on the lease.
Six months later, the mechanic lost his job. For the next three months, the mechanic was unable to find a new job, and therefore failed to pay rent. At the end of this period, the landlord decided to sue for the rent due.

From whom, if anyone, should the landlord be able to recover for the unpaid rent?

(A) The mechanic only.

(B) The student only.

(C) The mechanic and the student jointly.

(D) Neither is liable for the unpaid rent.

A

(A) The mechanic only.

289
Q

An architect and a developer were long-time companions who had summered in the developer’s cottage on the seacoast for 30 years. When the developer was confined to an assisted living facility because of poor health, she conveyed the deed to the cottage to the architect, who continued to summer there without her friend. The developer’s health continued to decline, and she soon died, leaving a will in which she bequeathed her personal property to the architect and devised her summer cottage to a favorite nephew who had spent time in the cottage with the developer and architect in the past. When the architect learned of the devise to the nephew, she immediately recorded the deed conveying the cottage to her. Assume that the jurisdiction in which the summer cottage is located has the following statute: “A conveyance of an estate in land, other than a lease for less than one year, shall not be valid as against any subsequent purchaser for value, except such persons having notice of it, unless the conveyance is properly recorded.”

If the architect files an action to quiet title to the cottage, what is the most likely outcome of the suit?

(A) The architect will prevail, because the nephew had knowledge of her residence in the cottage.

(B) The architect will prevail, because she acquired title to the cottage before the nephew.

(C) The nephew will prevail, because the architect failed to record before the conveyance by will to him

(D) The nephew will prevail, because the architect acquired title to the cottage by gift.

A

(B) The architect will prevail, because she acquired title to the cottage before the nephew.

290
Q

A homeowner took out a purchase money mortgage to buy a larger house for his family. Over time, he made several improvements to the home, using various contractors, as recommended by his new neighbors. The homeowner was unable to settle a dispute with one of the contractors after the homeowner alleged that the contractor had damaged his wrought iron fence while installing a pool. The homeowner refused to pay the pool contractor and the pool contractor placed a lien on the property while pursuing compensation in the courts.

Around that same time, when property values were at their highest, the homeowner took out a second mortgage on his residence to fund a massive renovation. The homeowner enlarged his master suite by merging his bedroom with one of the guest bedrooms, completely refitted the kitchen, and added a sun room on the back of the house. An unexpected recession in the economy caused the homeowner’s property value to fall precipitously, and he lost his job. Unable to pay both mortgages, the homeowner paid on the first mortgage, but let the second mortgage slide. Eventually, the second mortgagee initiated judicial foreclosure proceedings against the homeowner. During the judicial proceedings leading up to the foreclosure sale, the pool contractor and the second mortgagee participated, but the first mortgagee, with the original purchase money mortgage, declined to participate. The home was sold and immediately there was controversy over distribution of the proceeds, which were not enough to satisfy all the debts.

Where will the proceeds of the foreclosure sale be distributed first?

A The proceeds will be distributed first to the second mortgagee, because the first mortgagee did not participate in the foreclosure proceedings.
B The proceeds will be distributed first to the first mortgagee, because the first mortgagee’s interest is senior to the second mortgagee’s and the pool contractor’s interest.
C The proceeds will be distributed first to cover the costs of the foreclosure sale.
D The proceeds will be distributed first to the pool contractor, because contractor liens on real property are always given priority over mortgages.

A

C The proceeds will be distributed first to cover the costs of the foreclosure sale.

291
Q

A technician rented an apartment from a lumberjack. The technician said he would probably stick around for at least a year. The lumberjack did not ask the technician to sign a lease agreement. The technician decided to leave after three months.

Which of the following is not an implied duty of the technician?

A Not to commit waste.
B To find a new tenant.
C To leave at the end of the tenancy.
D To leave the premises in the same condition as when it was rented, excepting reasonable wear and tear.

A

B To find a new tenant.

292
Q

A skier owned a 10-acre ski resort located in a popular resort area. Already an excellent skier, she had recently taken up mountain climbing. Her dream was to scale Mount Everest, and she arranged to make this momentous climb. Prior to departing, the skier executed a deed granting the resort to the resort’s ski instructor. The skier told the instructor that he was to have the resort “only if she didn’t return from her climb alive.” The instructor, believing that the skier was a much better skier than mountain climber, was confident that she would never return from the trip. As such, he promptly recorded the deed. Then, hoping to increase the resort’s profits, the instructor began construction of a new ski lift on the property. The skier successfully climbed Mount Everest and returned home safely from her expedition. However, desiring to pursue mountain climbing full-time, she decided to sell the resort. The skier entered into a land-sale contract to sell the resort to an investor. Upon learning of the instructor’s claim of title, the investor filed suit to rescind the land-sale contract. If the court finds against the investor and enforces the land-sale contract, which of the following best explains why?

(A) The instructor was not a purchaser for value, mortgagee, or judgment creditor.

(B) The investor is protected by the recording act, since he purchased the resort without notice of the earlier conveyance.

(C) The skier automatically revoked the conveyance to the instructor when she entered into the land-sale contract with the investor.

(D) There was no valid delivery of the deed conveying title to the instructor.

A

(D) There was no valid delivery of the deed conveying title to the instructor.

293
Q

A former professional ballerina rented commercial space to open and operate a ballet studio. She installed floor-to-ceiling mirrors on all four walls, and operated the studio successfully for many years. In her 15th year, she decided to close the studio upon her lease’s termination at the end of the calendar year. The owner of the building began showing the space to several other potential lessees. One interested lessee planned to open a retail clothing store, and thought the mirrors around the perimeter of the store would be perfect for customers looking at and trying on clothes. The ballerina’s lease terminated on December 31, and the interested lessee returned on January 2. Thrilled to find the mirrors still there, she signed a five-year lease on the spot. The next day, the ballerina came to remove her mirrors and take them with her to use in her home studio, but was turned away by the new lessee.

Under the modern view, does the ballerina or current lessee have a right to the mirrors?

A. The ballerina, because she is permitted a reasonable period of time in which to remove any fixtures she installed.

B. The ballerina, because she installed the mirrors.

C. The current lessee, because ballerina did not remove the mirrors before the termination of her lease.

D. The current lessee, because ballerina did not provide notice to the current lessee that she would remove the mirrors after the lease’s termination.

A

????A. The ballerina, because she is permitted a reasonable period of time in which to remove any fixtures she installed.

Discussion of correct answer: A trade fixture is a chattel that is annexed to land by a tenant to further his business or trade during his tenancy, and is treated differently than fixtures annexed to property by the property owner himself. Under the modern law approach, if a tenancy is for a certain term and ends on a certain day, the tenant need not remove the trade fixtures by the lease’s expiration or risk forfeiting the item. Instead, the tenant is permitted a reasonable amount of time after the lease’s termination to remove the fixtures. Here, the ballerina installed the mirrors to help her ballet studio business, and as such they are trade fixtures. Her lease was for a certain term and expired on a specific day, and she therefore has a reasonable amount of time after the lease’s expiration to remove the mirrors. She arrived at the studio two days after the lease’s expiration, which is certainly a reasonable period of time. Therefore, the ballerina is entitled to the mirrors. Test tip: Very tricky. A tenant running a business and wanting to remove an item indicates that the question is asking about trade fixtures. The common law rule is the tenant can take the trade fixture away, but only before the new tenant’s lease begins. That seems fair because it is the tenant’s fault for not removing the item before its removal would inconvenience the new tenant. However, the question is asking the outcome under the modern rule. If you do not know the modern rule, make an educated guess that is the opposite of the outcome of the common law rule.

294
Q

A farmer owned a small farm near a town. The water to all outlets on the farmer’s farm other than his house was supplied by a well drilled by a previous owner a few hundred feet behind the farmer’s barn on the property. This well water became contaminated with arsenic. The congresswoman for that district who lived in the town was out hiking on the rural roads one very hot summer day when she got tired and decided to take a shortcut back to town over the farm fields. As she passed through the farmer’s farm, she noticed a water faucet next to the barn and stopped to take a long drink of the cool water. The arsenic content of the water made the congresswoman immediately very sick. The farmer discovered her unconscious body a few hours later and rushed her to the local hospital. After a long period of painful convalescence, the congresswoman recovered with minor long-term damage to her health. The congresswoman brought an action for personal injuries against the farmer arising from her drinking of the arsenic-contaminated water.

What will be the probable outcome?

A. Judgment for the farmer, if the congresswoman was the first person ever to enter upon his farm without permission.

B. Judgment for the farmer, unless the water faucet could be said to be an “attractive nuisance” under these circumstances.

C. Judgment for the congresswoman, if the farmer knew that the well water was tainted with arsenic.

D. Judgment for the congresswoman, if the farmer reasonably should have known that the well water was tainted with arsenic.

A

A. Judgment for the farmer, if the congresswoman was the first person ever to enter upon his farm without permission.

Discussion of correct answer: One who enters the defendant’s land without his permission or without a privilege to do so is a trespasser. The defendant is not liable to a trespasser whose presence is unknown to him, nor has he any duty to inspect his land to attempt to discover such trespassers. The defendant has a duty to exercise reasonable care to warn a known trespasser of hidden dangers known to the defendant and unknown to the trespasser. Where the defendant knows or reasonably should know that a portion of his land is frequently used by trespassers, he must take the same precautions to avoid injury to persons utilizing that portion of his land as if they were known trespassers, even if a particular plaintiff was not known to defendant and that plaintiff had never trespassed before. In this question, if no one had ever taken a shortcut across the farmer’s farm before the congresswoman, there was no basis to classify her as anything other than an unknown trespasser. As such, the farmer has no duty to warn her of any dangers.

295
Q

A tenant leased two floors of an office building from a landlord for a 10-year period, with rent due at the start of each month. According to the terms of the lease, the landlord agreed to perform routine maintenance, including but not limited to repairing and painting the walls, replacing carpeting, and maintaining the air conditioning and electrical systems, at the start of each year, after the tenant submitted an itemized list of requests. At the start of the third year of the lease, the tenant submitted the list to the landlord. However, the landlord, after seeing the costs incurred in doing the repairs in the second year, refused to do them.

What are the tenant’s remedies?

A The tenant may make the repairs himself and offset his subsequent rental payments by the costs incurred.

B The tenant may not make the repairs, because doing so would constitute ameliorative waste.

C The tenant may refuse to pay rent due to the landlord’s breach of covenant.

D The tenant may treat the lease as terminated if he vacates the premises.

A

Discussion of correct answer:A tenant may seek money damages for a landlord’s breach of a term or covenant in the lease. Traditionally, a tenant may vacate the premises and terminate the lease if the tenant has been evicted by the landlord, actually or constructively. Today, statutes in many jurisdictions provide the tenant with the following statutory remedies: (1) withholding rent; or (2) repairing the premises and deducting the cost of repair from subsequent rent payments. Therefore, under the circumstances, this is the best answer choice.

Discussion of incorrect answers:

Incorrect. The tenant may not make the repairs, because doing so would constitute ameliorative waste. Ameliorative waste occurs if an act of a life tenant increases the value of the premises by permanently altering it. Traditionally, ameliorative waste was prohibited, but this is no longer the case under the modern rule. A tenant’s duties in respect to waste are similar to those owed by life tenants. A tenant (except possibly a tenant for a long term) may not commit ameliorative waste (which a life tenant may do). Here, making the repairs would probably not rise to the level of waste, because it does not increase the property value, nor is it a permanent alteration (as, for example, building a pool would be). Thus, this is not the best choice.

Incorrect. The tenant may refuse to pay rent due to the landlord’s breach of covenant. A tenant may treat the lease as terminated and withhold rent if the landlord, or someone claiming through him, breaches the covenant of quiet enjoyment by actual eviction of the tenant. The breach of the landlord’s covenant to make annual repairs would likely not rise to the level of evicting the tenant, either actually or constructively. Rather, the tenant can seek money damages as his remedy.

Incorrect. The tenant may treat the lease as terminated if he vacates the premises. A tenant may treat the lease as terminated and withhold rent if the landlord breaches the covenant of quiet enjoyment by constructively evicting the tenant. For a constructive eviction to exist: (1) the landlord’s act must substantially and permanently interfere with the tenant’s use and enjoyment of the premises; and (2) the tenant must move out. The landlord’s failure to perform annual maintenance and repairs would likely not rise to the level of constructive eviction, unless the property has fallen into such disrepair that the tenant’s use and enjoyment was affected, which is not clear on these facts. Therefore, this is not the best answer.

296
Q

A landowner was the fee simple owner of a 25-acre parcel of land. Adjoining the property was a smaller 15-acre tract owner by a rancher. In 1985, the landowner conveyed 10 acres of his property to the rancher by a deed that contained the following provision: “In the event that the rancher obtains a bona fide offer for the purchase of the 10-acre parcel which he wishes to accept, he shall submit such offer to the landowner, who shall have a ‘right of first refusal’ within 30 days of such submission to purchase said property on the same terms contained in such offer. The option shall extend for 25 years and be in effect until 2010.”

In 2007, the rancher received a bona fide offer for the 10-acre parcel. The rancher, who was willing to accept the offer, refused to offer the 10 acres to the landowner, who was still alive and was the owner of the remaining 15 acres. There was no applicable statute in the jurisdiction. The landowner brought an appropriate action to enforce the option.

If the court rules in favor of the rancher, it will be because of what reason?

(A) The option is unenforceable as an illegal restraint against alienation.

(B) The option is violative of the Rule Against Perpetuities.

(C) The option was not supported by separate consideration.

(D) The landowner’s proper remedy was at law for money damages.

A

(B) The option is violative of the Rule Against Perpetuities.

297
Q

A professor owned a townhouse that he wanted to rent to a tenant. The professor posted an advertisement online seeking renters for the townhouse. The advertisement included pictures, but the professor used pictures of his neighbor’s townhouse, which had the same layout as the professor’s townhouse, but the neighbor’s townhouse had nicer, luxury finishes and was kept in much better repair. A student, who was moving from another state, saw the online advertisement and contacted the professor. The professor told the student that the townhouse looked just like the pictures, and the student would not be disappointed. Without seeing the property, the student signed a two year lease of the townhouse with the landlord. When the student arrived at the townhouse, the student was surprised to see the condition of the premises and that it did not look like the pictures. The student decided not to move into the townhouse and never paid the landlord any rent or security deposit.

Was the student’s conduct proper?

A Yes, because the professor breached the implied warranty of habitability.
B Yes, because the professor fraudulently misrepresented the condition of the townhouse.
C No, because the student did not give the professor notice and an opportunity to cure.
D No, because the student waived the right to inspect the premises by signing the lease without seeing the property.

A

B Yes, because the professor fraudulently misrepresented the condition of the townhouse.

298
Q

A homeowner gave a mortgage on a property to a third-party lender for $4 million, $2.5 million to buy the property and $1.5 million to make improvements on it.

Which of the following is true?

(D) This mortgage is a purchase-money mortgage, and as such, it has priority over all prior liens that attach to the property through the actions of the homeowner.

(A) This mortgage is not a purchase-money mortgage, because a portion was used to make improvements on the land.

(B) This mortgage is not a purchase-money mortgage, because the mortgage was given as part of the same transaction in which the homeowner acquired title to the property.

(C) This mortgage is a purchase-money mortgage, and as such, it does not have priority over any pre-existing liens on the property.

A

(D) This mortgage is a purchase-money mortgage, and as such, it has priority over all prior liens that attach to the property through the actions of the homeowner.

299
Q

A scientist encouraged all three of his children (two boys and one girl) to pursue scientific endeavors for the good of humankind. His two sons earned doctorates and engaged in medical research and went on to prestigious and noble professions. However, to the scientist’s chagrin, his daughter became an actress. The scientist found acting a frivolous and self-indulgent profession. Eventually, to the scientist’s infinite embarrassment, the daughter became a soft-porn star, who was well known in the industry. The scientist threatened the daughter with disinheritance if she persisted in her “career.” She insisted that the films were art and that she would not give up her true calling. The scientist died at the age of 90. By will, he created a trust, with income to be distributed for life to the two sons. The remainder was to be distributed among the scientist’s grandchildren, including the daughter’s progeny. The daughter was specifically disinherited. At the time of the scientist’s death, one son is married but has no children; the other has two sons; and the daughter has no children.

Which of the following best describes the interests that the scientist’s grandchildren hold in the testamentary trust?

Vested remainders subject to open.

Shifting executory interests.

Contingent remainders subject to express condition.

Contingent remainders.

A

The correct answer is: Vested remainders subject to open.

Discussion of correct answer: A remainder is a future interest created in a third party following a life estate. Here, the scientist’s testamentary trust created a life estate in his sons, with the remainder to his grandchildren upon the termination of the life estate by death. The interests of the scientist’s existing grandchildren (the son’s two sons) are vested remainders, because the interests were created in ascertained persons and are not subject to a condition precedent (other than the death of the sons). The grandchildren’s vested remainders are subject to open because if more grandchildren are born, the existing grandchildren’s vested remainders will be reduced by the subsequent vesting of the interests of other class members. Thus, the grandchildren’s interests are future interests created in ascertained persons who are in a class that may change with the addition of class members.

300
Q

A landlady believed that her tenants were like her family, and for this reason, all of her leases contained a prohibition on assignments. An executive signed a lease on Apartment 3 for five years at $3,000 per month rent. The executive paid her rent on time for four years. She then agreed to spend most of the next year setting up her company’s latest headquarters in London. The executive entered into an agreement with a dentist for the dentist to move into the executive’s apartment and pay the rent for 10 of the remaining 12 months on the lease. The executive intended to return to and resume living in the apartment for the last two months. The dentist moved into the apartment but never paid any rent to the landlady. When the executive returned home, she discovered a note from the landlady demanding the back rent.

Who is liable to the landlady for the rent owed?

(A) The executive, because assignments were prohibited.

(B) The executive, because the dentist was a subtenant.

(C) The executive and the dentist are jointly and severally liable.

(D) The dentist, because he agreed to pay the rent.

A

(B) The executive, because the dentist was a subtenant.

301
Q

A man owned Blackacre in fee simple. In 1982, the man conveyed Blackacre to a development consortium “on condition that a ski lodge and resort area be built thereon and the land be used solely for skiing purposes and, in the event that said property is not used as a ski resort, the property shall revert to myself, the grantor, my heirs or assigns.” Subsequently, a ski lodge was built, and the land was continuously used as a ski resort ever since. In 1987, a neighbor who owned Whiteacre, a property abutting the northeast corner of Blackacre, began to use a portion of Blackacre in order to gain access to Whiteacre for ingress and egress. He used this access road openly, visibly, and notoriously until 2008. The period of prescription in this particular jurisdiction was 20 years.

In 2008, the neighbor entered into a contract to sell Whiteacre to a buyer. A month before closing, the consortium’s president was approached by the buyer with the request that a written easement be given over the access road and that an appropriate instrument be delivered to him concurrently with his closing with the neighbor. Following this meeting, the development consortium voted unanimously to authorize its trustees to grant such an easement. Two weeks later, the man died, survived only by his wife. Upon his death, she immediately notified the consortium that if it granted the easement to the buyer, Blackacre would revert to her. The buyer then brought an appropriate action, joining the neighbor, the wife, and the development consortium, seeking a declaratory judgment that the neighbor has a perpetual easement appurtenant to Whiteacre.

In this action, judgment should be in favor of which of the following parties?

The wife, because the buyer could not obtain rights against the development consortium by prescription.

The wife, because the easement would violate the negative restriction that the man originally sought to impose.

The buyer, because the neighbor had already obtained an easement by prescription.

The buyer, because the language of the conveyance to the development consortium was too vague to support any claim by the wife.

A

The correct answer is: The buyer, because the neighbor had already obtained an easement by prescription.

Discussion of correct answer: An easement by prescription arises by adverse use of the servient tenement by the dominant tenant for the period of the statute of limitations. To mature such an easement against a landowner the use must be: (1) adverse as distinct from permissive in derogation of right, rather than in subordination to the rights of the landowner; (2) open and notorious; (3) continuous and without interruption; and (4) for the period of prescription. Because the neighbor openly and notoriously used the northeast portion of Blackacre to gain access to Whiteacre for the 20-year period, as required by statute, he obtained an easement by prescription. This easement will pass on to the buyer, who was in privity with the neighbor by means of the land-sale contract. Therefore, the buyer’s rights are enforceable against the whole world, and he will prevail against the wife.

302
Q

At 10 a.m., an owner sold his oil field to an oil investor for $800,000. The owner gave the investor a general warranty deed. The investor immediately sent the deed to her attorney with instructions to record it the following morning. On her way out of her meeting with the owner, the investor passed a famous financier in the hall. The financier was on his way in to meet with the owner as well. Noticing the oil investor, the financier was curious as to what business she could have with the owner, but he decided not to let it trouble him. At 11 a.m., the owner gave the financier a general warranty deed to the same oil field he had just sold to the investor, in exchange for $1 million. The financier immediately signed the property over to his wife, in order to make up for having forgotten their anniversary. The financier’s wife immediately recorded the deed. The property is located in a race-notice jurisdiction.

In an action by the financier’s wife to quiet title to the property, how will the court rule, and why?

The investor owns the property, and the financier’s wife is entitled to damages of $1 million from the original owner.

The investor owns the property, and the financier’s wife is entitled to no damages.

The financier’s wife owns the property and is entitled to damages of $1 million.

The financier’s wife owns the property, but is not entitled to money damages.

A

The correct answer is: The investor owns the property, and the financier’s wife is entitled to damages of $1 million from the original owner.

Discussion of correct answer: A purchaser is charged with inquiry notice of any prior claims to the property if he is aware of facts or circumstances that would lead a reasonable person to inquire further. In this case, the financier was on notice that the investor had just met with the owner, and his suspicions were raised; as such, the financier was obligated to make further inquiries regarding prior claims to the property. Furthermore, because the financier never had title to the property, he could not pass title to his wife. As such, the investor, not the financier’s wife, owns the property. Nevertheless, the financier’s wife is entitled to damages, because the owner breached the covenant of quiet enjoyment when he purported to sell the property to the financier. The covenant of quiet enjoyment runs with the property, permitting remote grantees to sue for its breach.

303
Q

A developer built a community with alcohol-free principles. A restaurant owner wanted to build a restaurant in the community, so the developer conveyed one parcel “to [restaurant owner], provided that no alcoholic beverages of any kind are served on the premises.” Years later, the restaurant owner obtained a license to serve beer and wine. The developer claimed that this violated the restriction in the deed, and demanded that the restaurant owner quit the premises.

Is the real estate developer entitled to reclaim the property?

(A) No, because the restriction violated the Rule Against Perpetuities.

(B) No, because no power of termination was reserved.

(C) Yes, because the conveyance to the restaurant owner was of a fee simple subject to an executory interest.

(D) Yes, because the conveyance to the restaurant owner was of a fee simple subject to a condition subsequent.

A

(B) No, because no power of termination was reserved.

304
Q

A writer and an artist were neighboring homeowners. The writer’s property was bordered by a public street, while the artist’s property was located off of the street, behind the writer’s property. The artist’s only means of egress to the public street that ran in front of the writer’s house was an unpaved driveway across the writer’s lot. The writer wanted to build a swimming pool that would block the artist’s use of the driveway, so the writer offered to build a new driveway on the other side of his property to reach the artist’s lot, on the condition that the artist would agree to help with the paving costs. The artist agreed, and the writer built the new driveway. The writer then gave the artist a deed of easement for the use of the new driveway for ingress and egress to his property. Some time later, the writer sold his property to a pharmacist, and the artist sold his to a waitress. The deeds from the writer and the artist did not mention the easement. The pharmacist obstructed the new driveway by parking his collection of antique vehicles on it, and the waitress brought an action to remove the obstruction and allow her continued use of the driveway.

Is the waitress likely to prevail?

No, because the artist’s, and, thus, the waitress’, continued use of the easement was permissive, and permission could be withdrawn at any time.

No, because the conditions for the creation of an easement by necessity cannot be met.

Yes, because the deed from the writer to the artist created an easement in gross.

Yes, because she purchased the dominant estate and acquired all the rights that belonged to the artist.

A

The correct answer is: Yes, because she purchased the dominant estate and acquired all the rights that belonged to the artist.

Discussion of correct answer: The artist had an express easement appurtenant by grant, which transferred to the waitress. An express easement appurtenant requires a dominant and servient tenement. The artist gained an easement for purposes of ingress and egress across the writer’s yard. Artist’s easement was the dominant estate because the easement benefited the artist in the use of his property. Conversely, the writer had the servient estate because his land was burdened by the easement. Transfer of title to a dominant estate includes the transfer of any easements appurtenant attached to the estate, regardless of whether the instrument of transfer expressly mentions the easement. A driveway for ingress and egress to and from the parcel is clearly appurtenant, since it is attached to a piece of land and benefits the owner of such land in her use and enjoyment thereof. As the purchaser of the dominant estate, the waitress has acquired the appurtenant easement and will prevail in her effort to compel the pharmacist to remove the obstruction.

305
Q

The owner of an apartment complex containing 50 units decides to convert his building into a condominium and sell the units individually. The owner realizes that a condominium association will have to be formed and it will have to collect pro rata monthly fees from the individual owners to maintain the common areas of the property. Some of the renters agree to buy their units, but others opt to move when their current leases expire. The owner wants to ensure that all future purchasers and assigns will be bound by the association fee requirement.

Which of the following will offer him the best chance of binding future owners to the fee restriction?

Covenant.

Easement.

Mortgage.

Personal contractual obligation by each purchaser.

A

The correct answer is: Covenant.

Discussion of correct answer: A covenant is a contractual obligation that is created in connection with the transfer of real property. If done properly, the covenant will bind and benefit not only the original obligor and obligee, but will “run with the land” to bind and benefit successors to the original parties. A covenant may be either negative (a promise not to do something, such as not to use land for industrial purposes) or affirmative (a promise to do something, such as make payments toward an annual operating deficit). Thus, a covenant is the best way to assure that each condominium owner, even those who did not buy directly from the owner, will have to make payments toward the maintenance of the common areas.

306
Q

Tom devised Blackacre to Larry and Dahlia in equal shares. A year later, Larry conveyed his interest in Blackacre to Paul. Larry subsequently died, and Paul brought an action to partition Blackacre. The jurisdiction has no statute directly applicable to the relevant issues.

What is the most likely reason why Dahlia would be declared the sole owner of all of Blackacre?

(A) The estate created in Larry and Dahlia by Tom’s will was a joint tenancy.

(B) The estate created in Larry and Dahlia by Tom’s will was a tenancy by the entirety.

(C) The estate created in Larry and Dahlia by Tom’s will included the right of survivorship.

(D) The estate created in Larry and Dahlia by Tom’s will was a fee rather than a life estate.

A

(B) The estate created in Larry and Dahlia by Tom’s will was a tenancy by the entirety.

307
Q

A development corporation subdivided desirable land into lots numbered 1 through 75. The corporation placed an ad in the local newspaper advertising the subdivision as “a beautiful residential neighborhood containing only single-family residences” and showed this ad to every prospective buyer. The corporation subsequently conveyed Lots 1 through 74 by deeds containing the restriction that the grantees were to use their lots exclusively for residential purposes.

Furthermore, the deeds stated that the restrictions “shall run with the land, and shall bind the purchasers, their heirs, executors, administrators, and assigns.” The subdivision had become a popular neighborhood full of single-family homes by the time a businesswoman bought Lot 75 and decided to open a small grocery store on her lot. A neighbor filed an injunction to prevent the businesswoman from opening the store.

Is the court likely to grant the injunction?

Yes, because both the neighbor and the businesswoman received their deeds from the development corporation, and, thus, the neighbor and the businesswoman are in privity of estate.

Yes, because the development corporation showed the newspaper ad to every prospective buyer.

No, because the businesswoman did not have actual notice of the restrictive covenants.

No, because the neighbor is only entitled to money damages.

A

The correct answer is: Yes, because the development corporation showed the newspaper ad to every prospective buyer.

Discussion of correct answer: When a landowner owns a large parcel of land and subsequently divides it into smaller parcels for purposes of development (e.g., a subdivision), that landowner may place restrictions in the deed of the parcels. Occasionally, the landowner will place the restrictions in the deed of some parcels, but not in all. However, where the developer intends a common scheme for the entire parcel of land, including all of the plots, a landowner whose deed does not contain the restriction may be bound by the restriction if the other deeds of the adjacent properties contain the restriction. Here, the development corporation intended a common scheme, as manifested by the newspaper advertisement shown to all of the prospective buyers, including the businesswoman. Thus, the businesswoman had constructive notice of the single-family residence requirement, and the court will likely enjoin her from opening the store.

308
Q

A museum director and a buyer entered into a written contract for the sale of a building owned by the museum, constructed of marble, and situated on a half-acre lot. After the contract had been signed but before the date set for closing, a powerful earthquake struck the area and totally destroyed the building. The museum has no insurance to compensate for the loss of the building. The contract made no provision for the destruction of the building, and the jurisdiction has no applicable statute governing the rights of the parties. When the buyer refused to deposit the purchase price into escrow, the museum director brought an appropriate action to enforce the contract via specific performance.

Which of the following best supports a judgment in favor of the museum?

The doctrine of equitable conversion means that the buyer is regarded in equity as the owner of the building.

Since the buyer could have avoided loss by insuring his interest in the building, he cannot complain about having to perform under the contract.

Specific performance is available to either party in a land sale contract since each parcel of land is considered unique, the loss of which is not compensable in damages.

Since the museum’s action sounds in equity, the buyer is precluded from raising the circumstances of the building’s destruction as a defense.

A

The correct answer is: The doctrine of equitable conversion means that the buyer is regarded in equity as the owner of the building.

Discussion of correct answer: The doctrine of equitable conversion allocates the risk of loss in a land sale contract to the purchaser. Thus, if damage or destruction occurs to the real property after formation but before closing, the purchaser (whom equity considers the owner of the property) bears the loss. In circumstances such as those presented by this question, the buyer is considered the equitable owner of the building from the moment the contract of sale is formed, and the subsequent destruction of the building by the earthquake is the buyer’s loss. The buyer must pay the full purchase price to the museum even though the value of the property has been substantially reduced by the loss of the marble structure. A number of jurisdictions have altered the doctrine of equitable conversion so that the risk of loss remains on the seller until closing unless the purchaser takes possession. The Uniform Vendor and Purchaser Risk Act generally places the risk of loss upon the seller until the buyer has either taken possession or obtained legal title, at least as to physical damage to the property. The facts here indicate that this jurisdiction had not adopted any statute on the subject.

309
Q

An owner has a home valued at $100,000 and owes Bank No. 1 $50,000 for a first mortgage lien. The owner owes Bank No. 2 $40,000 for a second mortgage. The owner applies to Bank No. 1 for additional funds to purchase a motor home, and the bank provides the funds. Bank No. 1’s mortgage is increased to $100,000 after it increases the homeowner’s mortgage.

Is Bank No. 2 materially prejudiced as the result of Bank No. 1’s increasing the amount added to the homeowner’s mortgage?

(B) Yes, because the homeowner increased his loan obligation by $50,000.

(C) No, because the homeowner increased the principal amount owed to Bank No. 1.

(D) No, because the homeowner will have larger installment payments as the result of the additional monies loaned.

(A) Yes, because Bank No. 2 should have been required to authorize the additional loan.

A

(B) Yes, because the homeowner increased his loan obligation by $50,000.

B1 $50,000; modifies the principal balance by $50,000, B1, $100,000
B2 $40,000 B2 is now prejudiced by B1

Priority: B1 first $50,000; B2 $40,000; B2 is a judgment $10,000 (not a deficiency judgment), and B1 has a deficiency judgement because they foreclosed of $50,000.

If instead of increasing principal amount, increased interest rate, the entire B1 mortgage moves into 2nd priority place.

310
Q

A father conveyed land to his daughter with the following conveyance: “to daughter while the land is used for farming.”
What kind of estate does the daughter have in the land?

(A) A fee simple subject to condition subsequent.

(B) A fee simple absolute.

(C) A fee simple subject to executory interest.

(D) A fee simple determinable.

A

(D) A fee simple determinable.

311
Q

A landlord owned a home in a neighborhood that was zoned for mixed residential/commercial use, and which had been leased out for several years as a doctor’s office. Following the doctor’s retirement, the landlord rented the house to a family of four, who moved in immediately while the father began renovating the interior. A couple of months later, both of the family’s children began getting sick. A subsequent inspection discovered that the doctor and his staff had allowed hazardous chemicals to seep into the floors and walls, and which were slowly evaporating over time. The landlord, who was not aware of the situation until notified by the father, refused to pay to have the floors and walls scrubbed clean of the chemicals. The family sued the landlord, claiming that he had breached the lease’s implied warranty of habitability.

Will the family prevail?

A No, because the landlord did not know about the chemicals at the time it entered into the lease with the family.

B No, because the doctor’s lease was a commercial lease.

C Yes, because the family notified the landlord of the hazardous chemicals within a reasonable time.

D Yes, because the lease is not a long-term lease.

A

C Yes, because the family notified the landlord of the hazardous chemicals within a reasonable time.

Discussion of correct answer:In order to have a valid claim against a landlord for breach of an implied warranty of habitability, the tenant must notify the landlord of defect, and the landlord must fail to repair the defect within a reasonable time. Only residential leases contain an implied warranty of habitability. While the doctor’s lease was a commercial one, the family has a residential lease, and is therefore owed the implied warranty. Because the family notified the landlord of the defect and the landlord refused to make the necessary repairs, the family will prevail.

Discussion of incorrect answers:

Incorrect. No, because the landlord did not know about the chemicals at the time it entered into the lease with the family. Whether the landlord knew of the latent defect or not may have a bearing on other tort liabilities, but not in regard to the implied warranty of habitability. The landlord’s knowledge of the latent defect is irrelevant for a determination of whether the landlord breached the implied warranty of habitability.

Incorrect. No, because the doctor’s lease was a commercial lease. It is true that the doctor had a commercial lease, and as such, the landlord did not owe the doctor an implied warranty of habitability. However, the family has a residential lease, and as such is owed the implied warranty.

Incorrect. Yes, because the lease is not a long-term lease. This answer reaches the correct conclusion, but for the wrong reason. Some jurisdictions exclude long-term leases from having an implied warranty of habitability, which is the opposite of the reason stated here. Additionally, there is nothing in the facts to indicate whether this lease is a long- or short-term lease.

312
Q

An owner had a big house located on 500 acres on a mountaintop in the Rocky Mountains. He became tired of the snow and ice and wanted to move to the tropics. He was also a charter member of an environmental organization dedicated to saving the endangered Rocky Mountain spotted bobcat. The owner conveyed the house and property “to my nephew, but if hunting is allowed on the premises, then to my niece and her heirs.” Thirteen years after the grant, his nephew opened a hunting lodge on the property.
Which of the following is correct?

(A) The nephew has a fee simple absolute.

(B) The nephew has a fee simple subject to a springing executory interest.

(C) The niece has a fee simple subject to a springing executory interest.

(D) The niece has a shifting executory interest.

A

(A) The nephew has a fee simple absolute.

313
Q

A landowner entered into a written contract by which a contractor agreed to construct a daycare center on the landowner’s land within 10 months in exchange for $100,000. During construction of the daycare center, a dispute between Canadian and American timber interests increased the cost of lumber significantly. The contractor discovered that her costs for lumber to complete the daycare center would be $20,000 higher than she had expected. The contractor and landowner subsequently executed a written modification of the contract that provided that the square footage of the daycare center could be reduced from 2,000 to 1,800 square feet and that the contractor would still be entitled to the full contract price of $100,000. When the contractor had completed the building, the landowner refused to pay $100,000 as demanded, claiming that the 10% reduction in the size of the daycare center was a breach of the original contract, entitling her to damages reducing the amount owed on the contract.

In an appropriate action to determine the rights of the parties, applying traditional rules of contract law unmodified by statute, for whom should the court award judgment?

A The contractor, because the written agreement reducing the size of the daycare center was an enforceable novation.

B The contractor, because the written agreement reducing the size of the daycare center was an enforceable modification requiring no consideration.

C The contractor, because the written agreement reducing the size of the daycare center was an enforceable modification supported by the consideration of the contractor’s continued performance in the face of changed conditions.

D The landowner, because the written agreement reducing the size of the daycare center was not supported by consideration.

A

D The landowner, because the written agreement reducing the size of the daycare center was not supported by consideration.

314
Q

A real estate developer purchased a piece of real property in a lien theory jurisdiction on which he planned to construct low-income housing. The developer obtained a $5 million loan from a bank, which recorded a lien against the property. The bank loan documents specified that installment payments would start within six months of construction. A few days later, the developer obtained a second loan from the cmillion to ity for $1construct the low-income housing. The city subsequently filed a lien against the property.

Five months later, the developer was still in the middle of construction and asked the bank to reschedule the installment payments for its loan. The bank agreed to reschedule the installment payments and modified its senior lien. The city found out about this modification and claimed that its junior mortgage now had priority over the bank’s senior mortgage.

Is the city’s claim correct?

A.No, because the bank may make any modifications to its loan without losing its senior mortgage status.

B. No, because the rescheduling of installment payments did not materially prejudice the city.

C. Yes, because the rescheduling of loan payments is a modification.

D. Yes, because the rescheduling of loan payments materially prejudiced the city.

A

B. No, because the rescheduling of installment payments did not materially prejudice the city.

315
Q

A girl was browsing through a case of used CDs when she came across a rare boxed set of songs by her favorite singer. She did not have enough money to buy the set, so she slipped the CDs into her purse and started to leave the store. The owner approached the girl just as she reached the door. The girl saw the owner, tossed the CDs back into the store, and ran out the door.

In an suit by the owner against the girl for conversion, will the owner prevail?

(A) No, because the girl did not permanently remove the CDs from the store.
(B) No, because the girl did not harm the CDs.
(C) Yes, because the girl exercised dominion and control over the CDs.
(D) Yes, because the girl intended to deprive the owner of the CDs.

A

(C) Yes, because the girl exercised dominion and control over the CDs.

316
Q

A commercial landlord leased a two-story office building to an investment firm for a lease term of two years beginning on September 1. A heavy rainstorm in late October resulted in several leaks from the second floor ceiling. The tenant did not report the leaks because there was no real damage and the problem disappeared when the rain stopped. In early January the first snowfall of the season resulted in more leaks and in a sizable crack in the ceiling. This time the tenant informed the landlord, but nothing was done to correct the problem. The following month another heavy snow caused the roof of the building to collapse, and the offices on the second floor of the building were covered in wet plaster and other debris. The investment firm vacated the building and refused to make any further rental payments, claiming constructive eviction.

If the landlord is successful in a suit against the tenant for breaching the lease, it will most likely be for what reason?

A. The tenant failed to report the initial leak.

B. The tenant did not give the landlord adequate time to address the problem.

C. The defense of constructive eviction applies only to residential leases.

D. The defense of constructive eviction is not available to a tenant who abandons the premises.

A

C. The defense of constructive eviction applies only to residential leases.

317
Q

A young man gave his girlfriend, to whom he was engaged, a deed to a parcel of undeveloped land. The girlfriend framed the deed, but did not record it. Two days before the wedding, the young man broke off the engagement. The girlfriend took the deed from its frame, shredded it, and sent the pieces to the young man. The young man left town and later committed suicide. The court ordered the distribution of his estate. His mother claimed that she was entitled to the parcel of land. The girlfriend challenged the claim. She had placed a shrine on the land in order to try to expiate her guilt. The young man’s mother brought suit to quiet title.

Which of the following is the most likely outcome of the suit?

(A) The girlfriend will be awarded title, because she never transferred title back to the young man.

(B) The girlfriend will be awarded title, because she had made improvements to the land.

(C) The mother will be awarded title as the young man’s heir.

(D) The mother will be awarded title, because the girlfriend destroyed the deed, thereby rejecting the young man’s gift.

A

(A) The girlfriend will be awarded title, because she never transferred title back to the young man.

318
Q

An investor purchased a 100-acre piece of land 30 years ago. Ten years later, the investor built a house on a different residential property and moved in. The investor instructed an accountant to pay all property taxes on both the 100-acre piece of land and the residence and to inspect the 100-acre piece of land once every five years. Shortly thereafter, a trespasser cleared a five-acre section in the northwest corner of the 100-acre piece of land. The trespasser built a house, a barn, and a store and posted a sign, “Welcome to my town.” The trespasser never entered the other 95 acres of the land. Twenty years after the trespasser entered the property, the investor sold the 100 acres to a developer. The developer announced plans to build a mall and proceeded to clear the remainder of the property. The trespasser sought an injunction to stop the developer. The relevant statute of limitations is 10 years.

Will the trespasser succeed in proving that the trespasser has acquired title to the whole of the 100-acre property?

No, because the property was inspected every five years.

No, because the trespasser acquired title to only the five acres of land that he had cleared and used.

Yes, because the trespasser obtained title to all of the 100-acre property by adverse possession.

Yes, because the developer should have known the trespasser was in possession of part of the property.

A

The correct answer is: No, because the trespasser acquired title to only the five acres of land that he had cleared and used.

Discussion of correct answer: Under the doctrine of adverse possession, the occupation of only a portion of the property will confer title to the whole only if the claimant enters under color of title (constructive adverse possession). In this case, the trespasser did not enter under color of title. However, the trespasser will still be able to claim possession of the five-acre portion of the property that he inhabited and used, because his possession was open, visible, and notorious; actual; exclusive; hostile and under a claim of right; and continuous for the required statutory period.

319
Q

A teacher and a yoga instructor bought adjoining lots in a new residential subdivision. Each had custom homes constructed on their respective lots. The teacher also built a swimming pool with an extensive deck surrounding it. Two years later, the yoga instructor was doing some landscaping and struck concrete where she didn’t expect it. She hired a surveyor and learned that the concrete footings of the swimming pool extended across the property line into her lot, covering a triangular area 75’ long and varying in depth from a maximum of six inches at one end down to zero at the other. The yoga instructor brought an action for trespass against the teacher, seeking damages caused by the intruding concrete footings.

How is the court likely to rule?

Against the yoga instructor, because the minimal intrusion does not touch or concern any present or contemplated use of the yoga instructor’s land.

Against the yoga instructor, if the teacher acted in good faith and was unaware of the intrusion until after the swimming pool was in place.

In favor of the yoga instructor, despite the fact that the yoga instructor has suffered no actual damages to her land.

In favor of the yoga instructor, because the teacher did not meet the time requirement for adverse possession.

A

The correct answer is: In favor of the yoga instructor, because the teacher did not meet the time requirement for adverse possession.

Discussion of correct answer: It is true that the teacher did not meet the time requirement for adverse possession. The facts do not state the time period used in the jurisdiction, but the common law requirement is generally 20 years, with statutory periods of five, seven, or 10 years being common. Therefore, it is not necessary to examine whether the teacher intended to claim the land as her own, making possession sufficiently hostile to meet the standards of adverse possession under the majority rule.

320
Q

A hotelier owned an acre of land on an ocean bluff. He obtained the necessary government approvals to subdivide the parcel into two half-acre lots, one on the bluff and the other away from it. The hotelier sold the lot away from the ocean to a jeweler and retained the lot on the edge of the bluff for himself. The jeweler started building a large house. Its main feature was a second-story deck outside of a glass-walled entertainment room, facing the ocean and providing a panoramic view. Six months after the jeweler’s house was completed, the hotelier started building a three-story house on his lot that will substantially obstruct the view of the ocean from the entertainment room of the jeweler’s residence. There are no applicable zoning ordinances.

If the jeweler seeks to enjoin the hotelier from building his three-story residence so as to obstruct the view from the jeweler’s second story deck and entertainment room, what will be the probable outcome?

The hotelier will win, because there is no easement for light, air, and view benefiting the jeweler’s lot.

The hotelier will win, so long as he informed the jeweler at the time of the sale of the lot that he (the hotelier) intended to construct a multi-story residence on the retained lot.

The jeweler will win, because the hotelier’s residence would interfere with his (the jeweler’s) natural easement for light, air, and view.

The jeweler will win, because the hotelier is estopped from contesting the jeweler’s right to an ocean view by the hotelier’s failure to object when the jeweler constructed his house with the deck and entertainment room.

A

The correct answer is: The hotelier will win, because there is no easement for light, air, and view benefiting the jeweler’s lot.

Discussion of correct answer: To preclude an adjacent landowner from constructing a building that will interfere with his view, the jeweler must have some right or interest, such as an easement for light, air, or view, or an equitable restriction governing the maximum height of buildings on which to base his claim for judicial intervention. The hotelier granted the jeweler no such easement when he sold the lot, and it is not possible to gain an easement for light, air, or view by prescription, since the “use” of the supposed servient estate is not sufficiently actual, open, and notorious to put the owner of the servient estate on notice. Since the jeweler has no legal basis upon which to compel the hotelier not to build the large house, the jeweler will not obtain an injunction.

321
Q

A ski instructor rented a house by entering into a written lease agreement for a term of five years with a monthly rental fee of $2,000. The instructor resided in the house for two years. The instructor, in writing, then transferred his “entire interest for two years” to a chef. According to the written instrument, the chef was obligated to directly pay the ski instructor $2,200 per month for the term of his occupancy. For the next four months, the chef paid the tenant $2,200 each month, of which the ski instructor paid the landlord $8,000. During months five and six, the chef made his $2,200 payments to the ski instructor, but the ski instructor did not make any rental payments to the landlord for those two months. After not receiving his usual rental payments, the landlord went to the apartment and found the chef in possession. The landlord then sued the chef for $4,000.

Who should prevail?

A The chef, because he is not obligated to pay the landlord.
B The chef, because he made his monthly rental payments to the ski instructor.
C The landlord, because there was privity of contract between the landlord and the chef.
D The landlord, because the ski instructor transferred his entire interest to the chef.

A

A The chef, because he is not obligated to pay the landlord.

Discussion of correct answer: This is a straightforward property question dealing with subleases. A subtenancy is where the tenant transfers his right of possession for a time shorter than the balance of the leasehold, so that he retains a reversion within the leasehold. Therefore, the subtenant is neither in privity of estate nor privity of contract with the landlord. Lacking privity, a subtenant is not liable to the landlord to pay rent. There was a five year lease. After two years, the instructor transferred his “entire interest for two years.” Since there is still a year remaining on the lease, the transfer amounts to a sublease, not an assignment.

322
Q

Tom devised Blackacre to Larry and Dahlia in equal shares. A year later, Larry conveyed his interest in Blackacre to Paul. Larry subsequently died, and Paul brought an action to partition Blackacre. The jurisdiction has no statute directly applicable to the relevant issues.

What is the most likely reason why Dahlia would be declared the sole owner of all of Blackacre?

The estate created in Larry and Dahlia by Tom’s will was a joint tenancy.

The estate created in Larry and Dahlia by Tom’s will was a tenancy by the entirety.

The estate created in Larry and Dahlia by Tom’s will included the right of survivorship.

The estate created in Larry and Dahlia by Tom’s will was a fee rather than a life estate.

A

The correct answer is: The estate created in Larry and Dahlia by Tom’s will was a tenancy by the entirety.

Discussion of correct answer: The circumstances of this problem require that the estate devised to Larry and Dahlia be one that includes a right of survivorship and cannot be unilaterally severed by one of the co-tenants. There are two common law estates that include among their characteristics a right of survivorship: the joint tenancy and the tenancy by the entirety. The latter estate may only be held by a married couple. A joint tenancy may be unilaterally severed (and a tenancy in common created thereby) by one joint tenant conveying his interest in the property to a third person. A tenancy by the entirety may not be severed by unilateral conveyance; the tenants must mutually agree to convey to a third party or one must convey to the other. A tenancy by the entirety may also be severed by execution on the property by a joint creditor and by divorce. Because Dahlia was adjudged sole owner of Blackacre even after Larry had purportedly conveyed his interest to Paul, the Larry-Paul conveyance must have been void; that is possible on these facts only if Blackacre was held as a tenancy by the entirety.

323
Q

A merchant owned a large ranch in the western United States. The merchant promised a ranch foreman that he would receive the ranch when the merchant died. However, the merchant’s will left everything to the merchant’s niece. The niece lived in New York and never visited the ranch. She refused to respond personally to a letter from the foreman. The niece had the executor of the merchant’s estate shut down the ranch, fire the ranch foreman, and inform the ranch foreman that the ranch would never be his. Two years later, the ranch foreman returned to the land and reopened the ranch. The ranch foreman paid the property taxes and otherwise held himself out as the owner of the land for the next 21 years.

The ranch foreman finally decided that he was getting too old to work the ranch, and he contracted for the sale of the land to a conservation group. Their contract required that the ranch foreman convey “good and marketable title.” Before delivering the purchase price, an attorney for the conservation group discovered that the title to the land was in the niece’s name, not the foreman’s name, and the conservation group refused to go forward with the purchase. The ranch foreman sued the conservation group for breach of contract. The relevant statutory period is 20 years.

Will the ranch foreman prevail against the conservation group?

Yes, because the ranch foreman is entitled to specific performance of the purchase and sale contract.

Yes, because the ranch foreman is entitled to damages from the conservation group for breach of contract.

No, because the niece, as the owner of the ranch, is entitled to enforce the contract and receive payment from the conservation group.

No, because although the ranch foreman is the owner of the ranch, the foreman’s title is not marketable.

A

The correct answer is: No, because although the ranch foreman is the owner of the ranch, the foreman’s title is not marketable.

Discussion of correct answer: The ranch foreman is the owner of the ranch by adverse possession because his possession was open, visible, and notorious; actual; exclusive; hostile; and continuous for the statutory period. Title acquired by adverse possession may become marketable if the adverse possessor quiets title before attempting to convey it. However, because there is no evidence that the ranch foreman had brought an action to prove his adverse possession claim and quiet title, the ranch foreman’s title was not marketable. Modern courts might still find that title acquired by adverse possession is marketable if the adverse possession has been for a very long time; the record owner is unlikely to sue; and the record owner is unlikely to prevail if she did. The facts do not suggest that the niece, as title holder of record, would be unlikely to sue. Therefore, the ranch foreman would not prevail.

324
Q

A landlord owned an apartment building near a college campus. The landlord rented apartments primarily to students attending the college. She used a form lease for each apartment she rented which stipulated that rent was due on the first day of each month, and that the lease could not be assigned.
The landlord leased an apartment to a college student for a term of four years using her standard form lease. After occupying the premises for one year, the college student informed the landlord that he had been dismissed from college and could no longer afford to pay rent. The college student requested permission to assign the lease to a visiting professor. The landlord agreed to a waiver of the provision in the lease which prohibited assignments, stating that she would do so “this one time only because of the circumstances,” but would hold the college student liable for any unpaid rent. One year later, the visiting professor assigned the remainder of the lease to an athlete without requesting or receiving permission from the landlord. The athlete subsequently defaulted on the rent.

Who should the landlord sue for the unpaid rent?

(A) The college student only.

(B) The college student and the visiting professor.

(C) The visiting professor and the athlete.

(D) The athlete only.

A

(B) The college student and the visiting professor.

325
Q

A miner lives in a city where copper mining is prominent. The miner purchased a property by obtaining a mortgage from a bank. The property has a copper mine on the land, but the copper mine has not been excavated for a year. Without making physical changes to the appearance of the property, the miner started excavating the copper mine, and continued the mining operation to the point of extracting all available copper. The bank discovered that the miner was mining on the property and asked the miner to stop, because it would reduce the value of the property. The miner refused, and the bank filed an injunction against the miner.

Will the court grant the bank’s injunction?

(A) Yes, because the miner’s actions constitute waste and will reduce the property value.

(B) Yes, because the copper mine had not been excavated for a year when the miner began excavating it.

(C) No, because the copper mine was on the property originally and excavating it would not constitute waste.

(D) No, because an injunction is not a proper remedy in situations involving mortgage-related waste.

A

(C) No, because the copper mine was on the property originally and excavating it would not constitute waste.

326
Q

One afternoon, a dog owner returned home from work to find a maintenance man in the apartment and her dog locked in a back bedroom. The maintenance man told the owner that he’d come to fix the plumbing and the dog had bitten him in the leg. The owner was astonished and assured the man that her dog must have been very upset to do so, because she was ordinarily such a well-behaved little dog. Four days later, the owner’s sister-in-law and her two children came to visit. The children were young and not used to dogs. The little boy came up to the dog and yelled, “Doggie!” The poodle immediately jumped on him and bit his ear. The owner told her sister-in-law that it was the boy’s fault for yelling and asked them to leave. When they boy turned out to need extensive cosmetic surgery to repair the injury, the owner’s sister-in-law filed a strict liability action against the owner for the boy’s injuries.

To prevail in her strict liability claim, what must the sister-in-law prove?

(A) That the owner was negligent in not leashing the dog.
(B) That the boy did not yell at the dog.
(C) That the owner knew that the dog had a tendency to bite.
(D) That the boy and sister-in-law were invited, guests.

A

(C) That the owner knew that the dog had a tendency to bite.

327
Q

At the end of a long and enjoyable day at the amusement park, a boyfriend and girlfriend decided to end the day by walking through the fun house. However, the two discovered that they only had enough money left to purchase one admission ticket. The couple came up with a plan for the girlfriend to buy a ticket, go into the fun house, and let the boyfriend in through the emergency exit.

The girlfriend entered the fun house after purchasing her admission ticket, found the emergency exit, opened it, and let her boyfriend in. Nearby, a maintenance worker finished repairing an animated goblin character that leaped out at passersby. He activated it just as the girlfriend and boyfriend approached. The goblin malfunctioned and leaped out more than normal, hitting the boyfriend and causing a bruise. The boyfriend sued the owners of the fun house for the injury.

Which of the following, if true, would be most helpful to the boyfriend?

A The boyfriend’s presence had been discovered by the maintenance worker before he was injured by the goblin.
B Although the boyfriend was a trespasser, no one had ever been found to have improperly entered the fun house before.
C An inspection by the maintenance worker would have revealed that the goblin could malfunction and injure passersby.
D The boyfriend had visited the fun house on a previous trip to the amusement park.

A

A The boyfriend’s presence had been discovered by the maintenance worker before he was injured by the goblin.

Think Like a Lawyer
A land-occupier has no duty of care as to a trespasser whose presence is unknown to him.

Step by Step Walkthrough
Step 1: A landowner’s duty to known trespassers is to exercise reasonable care to protect them from injuries deriving from activities conducted on the land and to warn of hidden dangers.

Step 2: A defendant has no duty of care as to a trespasser whose presence is unknown to him. The defendant also has no duty to inspect his land to attempt to discover unknown trespassers.

Step 3: If the worker had discovered the boyfriend’s presence, then the boyfriend would be a known trespasser, rather than an unknown trespasser to whom a much lesser duty of care is owed. This would help his case. As an unknown trespasser, he will collect nothing.

Step 4: Select the answer stating that the argument that would be most helpful to the boyfriend is that the boyfriend’s presence had been discovered by the maintenance worker before he was injured by the goblin.

Step 5: Notice that the other answer choices do not provide any duty of care for an unknown trespasser. The boyfriend must try to become a known trespasser to have any chance of success in his suit.

328
Q

A Saleswoman and an architect entered into an agreement whereby the architect would lease an apartment owned by the saleswoman. The architect moved into the apartment as provided under the lease agreement. However, during the first several months of the lease term, and despite numerous requests from the architect, the saleswoman refused to repair a leaky faucet in the apartment’s bathroom. The lease did not address whose responsibility it was to make such repairs.
Under the common law, is the saleswoman required to make the repairs?

(A) Yes, because under the common law, the landlord has a duty to repair without exception.

(B) Yes, because a leaky faucet constitutes ordinary wear and tear.

(C) No, because the failure to repair a leaky faucet does not constitute waste.

(D) No, because the saleswoman has no duty to make any repairs to the premises.

A

(D) No, because the saleswoman has no duty to make any repairs to the premises.

Explanation
The correct answer is:No because the saleswoman has no duty to make any repairs to the premises.

Discussion of correct answer: Under the common law estate theory, the landlord had no duty to make any repairs to the premises during the lease term. By contrast, the tenant had a duty not to commit waste, which included a duty to repair the premises (including minor repairs to keep the building wind tight and watertight).

Discussion of incorrect answers:

Incorrect. Yes, because under the common law, the landlord has a duty to repair without exception. It is not true that under common law the landlord had a duty to repair without exception. In fact, at common law, the landlord had no duty to repair the premises during the course of the lease. The tenant, on the other hand, had a duty not to commit waste, which included a duty to repair the premises.

Incorrect. Yes, because a leaky faucet constitutes ordinary wear and tear. It is questionable whether a leaky faucet constitutes ordinary wear and tear. Even if a leaky faucet did constitute ordinary wear and tear, there was no rule at common law that required a landlord to make ordinary wear-and-tear repairs; in fact, the landlord had no duty to make any repairs whatsoever at common law. The tenant, however, had a duty not to commit waste, which included the duty to repair the premises.

Incorrect. No, because the failure to repair a leaky faucet does not constitute waste. Waste is an act that adversely affects a future interest in the estate. Allowing a leaky faucet to go unrepaired would constitute waste, because it would adversely affect the value of the rental property.

329
Q

A seller sold his house to a buyer subject to the existing mortgage on the house. The parties did not obtain the mortgagee’s permission for the transaction. After making payments on the mortgage for a few years, the buyer lost his job and was unable to make additional mortgage payments. The mortgagee, initiated foreclosure proceedings, and the property was sold at a foreclosure sale. The sale price realized was less than the amount still owed on the mortgage, so the mortgagee pursued the buyer for the deficiency.

Can the mortgagee pursue a deficiency judgment against the buyer?

(A) No, because the buyer did not assume the mortgage.

(B) No, because the seller is personally liable for the deficiency.

(C) Yes, because the buyer assumed the mortgage.

(D) Yes, because the buyer is jointly liable with the seller.

A

(B) No, because the seller is personally liable for the deficiency.

330
Q

A photographer purchases a home in Blackacre for $500,000 and obtains a loan from the bank to purchase the home. Every month, the photographer pays the bank $5,000. Two years later, the photographer transfers the property, subject to the mortgage, to his best friend, a well-known soap opera star. The soap opera star takes possession of the property and makes the first few monthly payments of $5,000 due to the bank. After several months, the soap opera star learns his character has been killed off and he is asked to leave the show. The soap opera star stops making mortgage payments.

Who may the bank pursue for the missing mortgage payments?

A The photographer, because the law prohibits the photographer from transferring the property subject to the mortgage.
B The photographer, because he is personally liable for the mortgage.
C The soap opera star, because he was responsible for making the monthly mortgage payments.
D The soap opera star, because he took the property subject to the mortgage.

A

B The photographer, because he is personally liable for the mortgage.

Discussion of correct answer: This is the correct answer choice. If a mortgagor transfers his interest in a property “subject to the mortgage,” the mortgagor still remains personally liable for the mortgage; the original mortgage remains intact, and the transferee does not assume personal liability for the obligation. Here, the photographer transferred the property to the soap opera star “subject to the mortgage.” When the soap opera star defaulted on the mortgage payments, the bank may attempt to collect the debt from photographer either by initiating foreclosure proceedings or by any other lawful means.

331
Q

A landowner granted Blackacre “to my nephew, his heirs and assigns, provided that he is survived by issue of himself and his wife, Sally, but if he dies without issue of Sally and himself, then to the county hospital.” The nephew took possession of Blackacre and used it wisely for many years. Unfortunately, Sally died without having any children. The nephew remarried, and his new wife, a geologist, discovered huge deposits of gold in the northeastern corner of Blackacre.

Does the nephew have the legal right to open a mine to extract gold from Blackacre?

No, because after Sally died childless, the nephew has no further ownership interest in Blackacre.

No, because after Sally died childless, the nephew is only a life tenant of Blackacre.

Yes, because the nephew holds a fee simple determinable.

Yes, because the nephew holds a fee simple defeasible.

A

The correct answer is: Yes, because the nephew holds a fee simple defeasible.

Discussion of correct answer: As holder of a fee simple defeasible, the nephew may use Blackacre as he pleases. Upon Sally’s death without children, the nephew can no longer avoid the condition that will cause ownership of Blackacre to shift to the county hospital. However, the grant specified that the county hospital would take upon the nephew’s death, so the nephew retains ownership of Blackacre during his life. While this might technically be termed a life estate, most commentators characterize it as a fee simple subject to an executory limitation, because the original grantor obviously intended that the nephew should have the use and enjoyment of Blackacre commensurate with a fee ownership. As owner of a fee, even a defeasible one, the nephew is not subject to the doctrine of waste, and possesses the legal right to use the property as he pleases.

332
Q

A chef owned a small, exclusive restaurant downtown. He owned both the building and the property, via a fifteen-year mortgage with an old, established mortgage company. The mortgage company recorded the mortgage. The chef planned to completely renovate the kitchen and took out a second mortgage from a new bank in town to cover the costs. The new bank recorded the mortgage. The contractor he hired for the renovation convinced him to upgrade on most of his choices, resulting in a cost overrun. The contractor placed a lien on the property for what was owed. After the renovations were completed, the chef hosted a grand re-opening party. Unfortunately, the oysters that the chef served were contaminated and most of the party guests became ill. Several people sued the restaurant as a result. Naturally, business declined after the incident, which was reported in the local paper. Struggling to make his mortgage payments, the chef went back to the mortgage company that held his original mortgage and renegotiated it into a thirty-year mortgage. He would ultimately pay a lot more in interest over time, but the renegotiation allowed him to better manage his money in the short run. The jurisdiction has a race-notice recording statute.

If the chef sells the restaurant and property, in what order will the various interests be satisfied?

(C) The first mortgagee will recover based on the newly negotiated thirty-year mortgage; then the second mortgagee will recover on the second mortgage; the lienholder will recover thereafter, and the plaintiffs will recover later if they win judgment.

(A) The first mortgagee will recover based on the original fifteen-year mortgage; then the second mortgagee will recover on the second mortgage; then the lienholder will recover, and finally, the first mortgagee will recover on the mortgage extension, and the plaintiffs will recover later if they obtain judgments.

(B) The first mortgagee will recover based on the original fifteen-year mortgage; then the second mortgagee will recover based on the second mortgage; then the first mortgagee will recover on the mortgage extension; the lienholder will recover thereafter, and the plaintiffs will recover later if they prevail in their lawsuits.

(D) The chef will be estopped from selling the property until the lawsuit is completed.

A

(C) The first mortgagee will recover based on the newly negotiated thirty-year mortgage; then the second mortgagee will recover on the second mortgage; the lienholder will recover thereafter, and the plaintiffs will recover later if they win judgment.

333
Q

After a gallery purchased one of her original paintings for an enormous sum of money, an artist decided to sell her studio, which was located in a rather dangerous part of town, and move to a nicer area. As a result of the studio’s less than desirable location, hardly anyone expressed any interest in purchasing the studio. Finally, a young newscaster saw an ad for the studio and came to view the property. The newscaster liked the studio very much, and thought that living in a “high-risk” area might give him access to breaking news stories. Sensing the artist’s eagerness to sell the studio, the newscaster offered to purchase the studio for $120,000, a price $50,000 below market value. The artist accepted the offer, and the newscaster gave the artist a check for $12,000 as a down payment on the studio, and the artist gave the newscaster the deed to the property. The parties agreed that the newscaster would deliver the remainder of the purchase price the following day.

However, the artist’s next-door neighbor overhead the communication between the artist and the newscaster. Less than an hour after the newscaster had left, the neighbor approached the artist with an offer to purchase the studio for the full market value of $170,000. Delighted at the opportunity, the artist immediately accepted the neighbor’s offer. That same night, the neighbor gave the artist a cashier’s check for the full amount of the purchase price, and the artist conveyed the deed to the neighbor. The neighbor did not immediately record the deed. The next day, the newscaster returned to the studio to give the artist a check for the remainder of the purchase price. Too embarrassed to admit that she had sold the studio to someone else, the artist accepted the newscaster’s check. Shortly thereafter, the newscaster recorded his deed to the studio. One week later, the neighbor recorded his deed. The jurisdiction in which the studio was located was a “pure notice” jurisdiction.

Under the jurisdiction’s “pure notice” recording statute, who owns the studio?

A. The newscaster, because he recorded his deed first.

B. The newscaster, because the neighbor had notice of the conveyance to the newscaster.

C. The neighbor, because he took subsequent in time to the newscaster.

D. The neighbor, because he was a bona fide purchaser.

A

B. The newscaster, because the neighbor had notice of the conveyance to the newscaster.

Discussion of correct answer: Under a “pure notice” recording statute, an unrecorded conveyance or other instrument is invalid as against a subsequent bona fide purchase for value and without notice.

To prevail under a notice statute, a claimant of real property must prove the following three elements:

1) that the claimant took subsequent in time to another person claiming ownership of the real property in question;
2) that the claimant was a bona fide purchaser for value; and
3) that the claimant took the property without actual, constructive, or inquiry notice of a prior claim to the property.

In this case, the neighbor’s purchase of the studio did occur subsequent in time to the transaction between the artist and the newscaster, and the neighbor paid value for the studio. However, given that the neighbor overhead the transaction between the artist and newscaster and knew that the artist had delivered to the newscaster a deed to the studio, the neighbor had actual notice of the newscaster’s prior claim to the property, and will not be protected under the pure notice recording statute. Therefore, the newscaster is the legal owner of the studio.

334
Q

A boyfriend and girlfriend lived in an apartment while the boyfriend worked in a repair shop. The couple signed the lease as married because the landlord refused to permit cohabitation by unmarried couples. The owner of the repair shop devised the repair shop to the boyfriend and girlfriend as “husband and wife, as tenants by the entirety.” The next year, the owner died and the boyfriend took over management of the repair shop. However, a month after the shop owner’s death, the girlfriend left the boyfriend. The jurisdiction does not recognize common-law marriage.

In an action by the boyfriend against the girlfriend for partition of the repair shop, is the court likely to grant partition?

(A) No, because a tenant by the entirety has no right to partition.

(B) No, because the boyfriend has absolute title to the property and need not seek partition.

(C) Yes, because the tenancy by the entirety was severed when the girlfriend abandoned the boyfriend.

(D) Yes, because the estate created by the deed was not a tenancy by the entirety.

A

(D) Yes, because the estate created by the deed was not a tenancy by the entirety.

335
Q

A homeowner borrowed $50,000 from a bank, secured by a mortgage on his home. Shortly thereafter, the homeowner sold his home to a buyer for $70,000 by a deed containing a recital signed by both parties that title passed “subject to” the bank’s mortgage, “which obligation grantee expressly assumes.” The buyer paid the homeowner $20,000, took possession of the house, and began making monthly payments of principal and interest to the bank. A few years later, a chemical manufacturing firm built a huge sulfur processing plant just down the road from the home, which cause the house to immediately decline in value to $35,000. Subsequently, the buyer stopped making the monthly payments to the bank. The bank exercised its contractual right of nonjudicial foreclosure and sold the house at a public auction for $34,000. The bank then brought suit against the homeowner and the buyer for $14,000, the difference between the proceeds of the foreclosure sale and the $48,000 principal remaining due on the original loan to the homeowner. This jurisdiction does not bar deficiency judgments.

Against whom should the bank be granted a judgment for the $14,000?

A) Both the homeowner and the buyer.

B) Only the homeowner.

C) Only the buyer.

D) Neither the homeowner or the buyer.

A

A) Both the homeowner and the buyer.

336
Q

An owner purchased a home in a new subdivision, paying 20% of the purchase price as a down payment and financing the rest of her purchase through a mortgage with a lender. The owner lived in her home for three years and always made her mortgage payments promptly. She then decided to put her house on the market. While the house was being marketed, the owner continued to make all mortgage payments promptly. She sold the house to a buyer, who purchased the property subject to the mortgage. After the buyer took possession, the lender received no further mortgage payments from either the owner or the buyer.

In most states, which of the following best describes the remedy or remedies available to the lender?

A) The lender may foreclose on the land, but may not sue either the owner or the buyer on the underlying debt.

B) The lender may foreclose on the land and it may sue the owner on the underlying debt.

C) The lender may foreclose on the land or it may sue the buyer on the underlying debt.

D) The lender may foreclose on the land and it may elect to sue either the owner or the buyer on the underlying debt.

A

B) The lender may foreclose on the land and it may sue the owner on the underlying debt.